Bovine Flashcards

1
Q

You are called out to a dairy herd that has recently been experiencing reproductive problems. The cows were all acquired 1 year ago from an unknown source and have no known vaccinations or tattoos. The dairyman reports that several cows have had late abortions (6-7 months gestation) and weak or stillborn calves in the past year and he’s never had this problem before. Several of the cows that had abortions developed placental retention and/or metritis.

None of the younger pre-pubescent heifers are displaying any clinical signs. You perform a necropsy on two recently aborted fetuses and find lung consolidation in one but no other obvious abnormalities. Which of the following should you recommend?

  • Serologic testing for Neospora
  • Test for Trichomoniasis
  • Test for Campylobacter fetus subsp venerealis
  • Serologic testing for Brucellosis
A

Answer: Serologic testing for Brucellosis

Explanation
You should be most suspicious of Brucellosis based on the assortment of signs (abortions, retained placenta, metritis and lack of signs in younger animals), the timing of abortions (last half of pregnancy), and the relatively normal appearance of the examined fetuses. Plus the cattle are not tattooed as they should be if they were given calfhood brucella vaccine.

Tritrichomonas fetus abortions usually occur in the first half of gestation but do have placentitis and pneumonia in the fetus as does Brucella. Campylobacter fetus subsp venerealis usually causes early embryonic death. Neospora abortions are usually mid gestation and cause necrosis of the cotyledons, with fetal lesions including myocarditis, hepatitis, myositis and encephalitis.

How well did you know this?
1
Not at all
2
3
4
5
Perfectly
2
Q

When performing a necropsy on a cow, you find multiple abnormalities including abomasal ulcers, interstitial pneumonia, pericardial effusion, and mastitis. You also see that the liver has a brown- (tan) and red-mottled lobular pattern described as a nutmeg liver. Which of these conditions in the cow could lead to the necropsy findings in the liver?

  • Pericardial effusion
  • Abomasal ulcers
  • Interstitial pneumonia
  • Mastitis
A

Answer: Pericardial effusion.

Explanation
The appearance of a nutmeg liver occurs because there are red central veins and sinusoids between tan areas of swollen hepatocytes. This occurs with right sided heart failure from passive congestion of the sinusoids of hepatocellular hypoxia. Pericardial effusion causes right sided heart failure and is the cause of the liver’s appearance.

How well did you know this?
1
Not at all
2
3
4
5
Perfectly
3
Q

You perform a joint tap on a swollen carpus of this bull in the picture and obtain the following results: total protein 4.7 g/dl, total nucleated cell count 26,900/ul, 84% neutrophils. What is your interpretation of these results?

  • Normal
  • Unlikely to be infected
  • Likely to be immune mediated joint disease
  • Likely infected
A

Answer: Likely infected.

Explanation
The correct answer is likely infected.

Guidelines for an infected joint are as follows:

  • total protein greater than 4.5 g/di
  • total nucleated count greater than 25,000/ul
  • polymorphonuclear cell count greater than 20,000/ul
How well did you know this?
1
Not at all
2
3
4
5
Perfectly
4
Q

A group of yearling steers were recently fed moldy sweet potatoes two days ago. Yesterday, they began to exhibit tachypnea, dyspnea, grunting, frothing at the mouth, deep cough, and respiratory distress. Today there are several dead. The lungs are wet, firm, and fail to collapse. Lobules are dark red and firm, with hemorrhages, edema, emphysema and bullae throughout the interstitial pneumonia. What is the cause of this disorder?

  • 4-ipomeanol
  • Bovine virus diarrhea
  • 3-methylindole
  • Mannheimia hemolytica
  • Bovine respiratory syncytial virus
A

Answer: 4-ipomeanol

Explanation
A metabolite of 4-ipomeanol is a pneumotoxic compound that is produced by the fungus Fusarium solani acting on the sweet potato compound called 4-hydroxymyoporone. This metabolite is toxic to pulmonary tissues. Lesions are similar to those found with acute bovine pulmonary edema and emphysema (ABPE) caused by conversion of ingested L-tryptophan in lush pastures to pneumotoxic 3-methylindole.

The disorder in the steers is caused by 4-ipomeanol, a pneumotoxic compound produced by Fusarium solani acting on sweet potato 4-hydroxymyoporone. This toxin affects pulmonary tissues, causing lesions like those in acute bovine pulmonary edema and emphysema (ABPE) from L-tryptophan conversion to 3-methylindole in lush pastures. Symptoms include respiratory distress and lung pathology.

CGPT: 4-Ipomeanol Toxicosis (Moldy Sweet Potato) and Perilla Ketone Toxicosis (Purple Mint Toxicosis) in Cattle

Definitions and Causative Agents:

•	4-Ipomeanol Toxicosis: Caused by ingestion of moldy sweet potatoes infested with Fusarium solani, producing the pneumotoxin 4-ipomeanol.
•	Perilla Ketone Toxicosis: Caused by ingestion of leaves and seeds of Perilla frutescens (purple mint), containing a pneumotoxin.

Clinical Changes and Symptoms:

•	Similar to acute bovine pulmonary emphysema and edema (ABPEE).
•	Symptoms: Severe dyspnea, frothy nasal discharge, coughing, tachypnea, and death in severe cases.

Diagnosis:

•	Tests: History of ingestion, clinical signs, necropsy findings.
•	Findings: Pulmonary edema, emphysema.

Treatment:

•	Medications:
•	Corticosteroids: Dexamethasone to reduce inflammation.
•	Diuretics: Furosemide to manage pulmonary edema.
•	Supportive Care: Oxygen therapy, maintaining hydration.

Prevention:

•	Avoid feeding moldy sweet potatoes or exposure to purple mint.

https://www.merckvetmanual.com/respiratory-system/non-infectious-respiratory-system-diseases-in-cattle/4-ipomeanol-toxicosis-moldy-sweet-potato-and-perilla-ketone-toxicosis-purple-mint-toxicosis-in-cattle?redirectid=4694?ruleredirectid=30

How well did you know this?
1
Not at all
2
3
4
5
Perfectly
5
Q

A clinician calls you about a heifer he examined and mentions that he heard a murmur over what he thought was the aortic region. Over which intercostal space would an aortic valve murmur be heard best?

  • Left 3rd intercostal space
  • Right 3rd intercostal space
  • Left 4th intercostal space
  • Right 4th intercostal space
  • Left 5th intercostal space
A

Answer: Left 4th intercostal space

Explanation
The correct answer is the left 4th intercostal space. We at VetPrep always think of our left-handed ex-girlfriend PAM. That stands for Pulmonic, Aortic and Mitral and then you just work your way from the 3rd intercostal space (pulmonic) to the 5th intercostal space (mitral). So the point of maximal intensity for an aortic murmur will be on the 4th intercostal space. Keep in mind that this is all on the left side. On the right side the only valve you can appreciate is the tricuspid valve which is audible best between the 3rd and 4th intercostal space.

The aortic valve murmur is best heard at the left 4th intercostal space. Remember “PAM” (Pulmonic, Aortic, Mitral), moving from the 3rd (pulmonic) to the 5th (mitral) intercostal space on the left side. The tricuspid valve is heard on the right side between the 3rd and 4th intercostal spaces.

How well did you know this?
1
Not at all
2
3
4
5
Perfectly
6
Q

A cow presents to you for decreased milk production. On physical exam, you determine the cow has a much stronger pulse on expiration than she does on inspiration. What is this pulse associated with?

  • Pericardial effusion
  • Atrial premature contractions
  • Patent ductus arteriosus
  • Atrial fibrillation
A

Answer: Pericardial effusion

Explanation
The correct answer is pericardial effusion. Pulsus paradoxus is an exaggeration of normal variations in the pulse quality during the respiratory cycle; the pulse becomes weaker on inhalation and stronger on exhalation. It is characteristic of constrictive pericarditis or pericardial effusion. Atrial premature contractions are more likely to result in pulse deficits.

Atrial fibrillation is likely to result in pulsus alternans, in which you see two quick normal pulses in a row followed by no pulse.

A patent ductus arteriosus usually is found only in very young animals and results in waterhammer pulses which is a large bounding pulse with a sharp peak and very rapid decline.

Pericardial effusion is associated with a stronger pulse on expiration than on inspiration, known as pulsus paradoxus. This exaggerated variation in pulse quality during the respiratory cycle is characteristic of pericardial effusion or constrictive pericarditis. In contrast, atrial premature contractions cause pulse deficits, and atrial fibrillation leads to pulsus alternans.

How well did you know this?
1
Not at all
2
3
4
5
Perfectly
7
Q

A very valuable dairy cow has died after a long illness with fever that the owner had treated with potent antimicrobial drugs for a week, then had given up. He asks you to perform a post-mortem to tell him the cause of death so he can collect insurance money. You find edematous lungs, edema in the ventral thoracic subcutaneous tissues, and this heart lesion. **What is the diagnosis?*

Valvular endocarditis
Pulmonary stenosis
White muscle disease
Tetralogy of Fallot
Ventricular septal defect

A

Answer: Valvular endocarditis

Explanation
Bacteria can lodge on heart valves and collect platelets and do damage to valves, causing murmurs and dysfunction and in some cases heart failure. The bacteria usually enter via a septic site such as a hoof abscess.

Valuable cows should be treated with penicillin or ceftiofur for several days if they develop hoof abscesses or similar problem.

The diagnosis is valvular endocarditis. Bacteria from a septic site, such as a hoof abscess, can lodge on heart valves, causing damage, murmurs, and potential heart failure. Valvular endocarditis is associated with edematous lungs and subcutaneous edema. Treatment for valuable cows with hoof abscesses includes penicillin or ceftiofur for several days.

How well did you know this?
1
Not at all
2
3
4
5
Perfectly
8
Q

What cardiac abnormality is commonly associated with hypocalcemia and milk fever in the cow?

Tachycardia
Atrial fibrillation
Ventricular premature contractions
Sinoatrial node block

A

Answer: Tachycardia

Explanation
The correct answer is tachycardia. Calcium administration may cause all the other abnormalities if you give too much too fast. You may also see a sinus node arrest. Calcium administration will cause the heart to beat more slowly and more strongly.

The cardiac abnormality commonly associated with hypocalcemia and milk fever in cows is tachycardia. Rapid calcium administration can cause other abnormalities, such as ventricular premature contractions, sinoatrial node block, or sinus node arrest, leading to a slower and stronger heartbeat.

** Parturient Paresis in Cows - Comprehensive Information**
Definitions and Causative Agents:
• Parturient Paresis (Milk Fever): Acute hypocalcemia causing flaccid paralysis around parturition.
• Causative Agents: Low blood calcium due to high calcium demand at lactation onset.

Clinical Findings:
• Stage 1: Hyperexcitability, ataxia, tremors.
• Stage 2: Sternal recumbency, cold extremities, tachycardia, dry muzzle.
• Stage 3: Lateral recumbency, coma, severe muscle flaccidity, potential death.

Diagnosis:
• Clinical Signs: Recumbency, muscle tremors, decreased heart sounds.
• Blood Tests: Hypocalcemia confirmation.

Treatment:
• Stage 1: Oral calcium supplements (40-55 g elemental calcium).
• Stage 2 & 3: IV calcium gluconate (500 mL of 23% solution), slow infusion.

Prevention:
• Dietary Management: Acidogenic diets prepartum (DCAD of -50 to -150 meq/kg).
• Monitoring: Urinary pH to maintain optimal acidification.
• Prophylactic Calcium: Oral calcium around calving.

How well did you know this?
1
Not at all
2
3
4
5
Perfectly
9
Q

A rancher has just found a big 2 month-old calf dead in the field and asks you to perform a post-mortem to determine the cause of death. He is worried about a contagious disease in his herd. You find the lungs are edematous and there is mild bronchopneumonia, but this lesion is found in the heart. Which diagnosis best fits?

Pulmonary stenosis
Tetralogy of Fallot
Ventricular septal defect
Patent ductus arteriosus
Bacterial endocarditis

A

Answer: Ventricular Septal Defect

Explanation
Congenital ventricular septal defect is one of the most commonly encountered congenital cardiac defects in cattle. It may be a relatively benign of cause death, depending on size. If the rancher sees several calves with this defect, he should discontinue using the bull responsible.

The diagnosis is ventricular septal defect, a common congenital cardiac defect in cattle. Its severity can vary, sometimes leading to death. If multiple calves are affected, the rancher should stop using the responsible bull to prevent further cases. Other diagnoses are less likely based on the findings.

How well did you know this?
1
Not at all
2
3
4
5
Perfectly
10
Q

A 7 year old Jersey cow presents to you with anorexia, bottle jaw, brisket edema, and a true jugular pulse. She has a heart rate of 90 bpm, rapid respiration, and is depressed. What is your primary differential?

Generalized lymphangitis
Right heart failure
Salt poisoning
Anaplasmosis

A

Answer: Right heart failure

Explanation
The correct answer is right heart failure. As a result of an inability to pump blood, it backs up and edema and a true jugular pulse occur.

The primary differential is right heart failure. The inability of the heart to pump blood effectively leads to backup, resulting in edema, bottle jaw, brisket edema, and a true jugular pulse. This is consistent with the cow’s clinical signs.

How well did you know this?
1
Not at all
2
3
4
5
Perfectly
11
Q

What is the most common supraventricular arrhythmia in cattle?

Atrial fibrillation
Ventricular fibrillation
Sinoatrial node arrest
Atrial standstill

A

Answer: Atrial fibrillation

Explanation
The correct answer is atrial fibrillation. This usually occurs as a result of digestive disturbances. It is associated with vagal tone, hypokalemia, hyperkalemia, and calcium therapy. You may be able to palpate a pulse deficit and/or pulsus alternans in these animals. On ECG, there will be no P waves because the sinoatrial node is not working. You will see random high frequency, low amplitude depolarizations called F waves. A QRS wave that occurs at irregular intervals will be present.

The most common supraventricular arrhythmia in cattle is atrial fibrillation, often resulting from digestive disturbances. It is linked to vagal tone, electrolyte imbalances, and calcium therapy. On ECG, there are no P waves, with irregular QRS waves and F waves due to sinoatrial node dysfunction.

How well did you know this?
1
Not at all
2
3
4
5
Perfectly
12
Q

You are in the process of performing an ECG on a Charolais cow with chronic pulmonary disease and notice that the P wave is tall and slender and measures 0.48mV (normal <0.4 mV), a condition known as P pulmonale. What conclusion as to the nature of the cardiac abnormality can you make from this finding?

Left atrial enlargement
Right atrial enlargement
Congestive heart failure
Left ventricular enlargement

A

Answer: Right atrial enlargement

Explanation
The correct answer is right atrial enlargement. This ECG finding is known as P pulmonale and is characterized by tall slender peaked P waves greater than 0.4mV. Many times these patients have a history of chronic pulmonary disease.

Left atrial enlargement on an ECG is known as P mitrale. In this case, an increase in duration of the P wave is seen. Usually they will last at least 0.05 seconds. The reason you see these electrical changes is because of the resulting vectors that are produced by having a certain portion of the heart enlarged.

The conclusion is right atrial enlargement, indicated by the P pulmonale finding on the ECG, characterized by tall, slender P waves over 0.4mV. This is commonly seen in patients with chronic pulmonary disease. Left atrial enlargement, or P mitrale, shows prolonged P wave duration instead of increased height.

How well did you know this?
1
Not at all
2
3
4
5
Perfectly
13
Q

A rancher who lives at 8000 ft. elevation in Colorado is planning to buy good bulls and wants you to test them for susceptibility to high mountain disease (pulmonary hypertension and cor pulmonale). See image for what high mountain disease looks like. You recommend doing pulmonary artery pressure (PAP) testing in the squeeze chute to measure their susceptibility to pulmonary hypertension. Which of the following PAP measurements, taken at 6000 ft. elevation, would be satisfactory for this ranch to purchase?

  • 45 to 50 mmHg pressure
  • None, the testing must be done below 5000 ft. elevation to be reliable
  • None, there is no reliable way to predict susceptibility to high mountain disease
  • Less than 40 mmHg pressure
  • 55 to 60 mmHg pressure
A

Answer: Less than 40mmHg pressure

Explanation
PAP testing done at elevations 6000 ft. or above on animals over 12 months of age is a reliable predictor of genetic susceptibility to high mountain disease (the BMPR gene is involved). The lowest pulmonary artery pressure readings are the least susceptible animals. Using local anesthesia, a catheter is inserted into the jugular vein and threaded into the right heart and pulmonary artery where the pressure is taken while the animal is restrained in a squeeze chute. The condition is caused by a genetic predisposition to developing pulmonary hypertension when oxygen tension is reduced.

The pulmonary hypertension leads to pulmonary artery hypertrophy, cor pulmonale, and right heart failure. The hallmarks are lethargy, distended jugular vein, and ventral edema (see image), as well as diarrhea, pulmonary edema, and pleural effusion.

Ingestion of toxic plants such as locoweed worsen the disease by causing additional pathology.

Cold weather, pregnancy and other stresses can also predispose animals and cause borderline susceptible animals to develop the condition.

A satisfactory pulmonary artery pressure (PAP) measurement for bulls at 6000 ft. elevation to predict susceptibility to high mountain disease is less than 40 mmHg. PAP testing at elevations of 6000 ft. or above is reliable for detecting genetic susceptibility. Animals with lower pressures are less likely to develop pulmonary hypertension, cor pulmonale, and right heart failure.

Bovine High-Mountain Disease (BHMD): NAVLE Study Guide

Definitions and Etiology
• BHMD: Also known as “Brisket Disease,” it occurs in cattle at high altitudes due to hypoxia-induced pulmonary hypertension.
• Causative Factors: Genetic predisposition, high altitude, pulmonary vascular shunting, and locoweed poisoning (Oxytropis and Astragalus spp.).

Pathophysiology
• Mechanism: Hypoxia causes pulmonary arterial vasoconstriction and hypertrophy, leading to pulmonary hypertension, right ventricular hypertrophy, and eventual congestive heart failure.
• Vascular Remodeling: Chronic exposure results in loss of peripheral pulmonary arteries and increased pulmonary resistance.

Clinical Signs
• Symptoms: Brisket edema, pleural effusion, jugular distension, cyanosis, labored breathing, and sometimes exophthalmos. Animals may become recumbent and show signs of right heart failure.
• Advanced Signs: Ascites, profuse diarrhea, and decreased appetite due to intestinal venous hypertension.

Diagnosis
• Pulmonary Arterial Pressure (PAP): Elevated PAP (>50 mmHg) and visible signs of right ventricular failure. Radiographic and ultrasonographic findings support the diagnosis.
• Differential Diagnosis: Includes other causes of congestive heart failure, such as pericarditis, lymphosarcoma, and myocarditis.

Treatment
• Primary Treatment: Moving affected cattle to lower altitudes, supportive care, diuretics, and thoracocentesis. Affected cattle should not be returned to high altitudes.
• Prevention: Selective breeding for animals with lower PAP measurements, minimizing exposure to locoweed, and addressing concurrent diseases.

Key Points for Veterinary Professionals
• Genetic Selection: Breeding programs should focus on selecting cattle resistant to hypoxia.
• Environmental Management: Control exposure to high-altitude stressors and toxic plants like locoweed.
• Ongoing Research: Studies on genetic markers and improving diagnostic techniques for BHMD are critical for future prevention and management strategies.

How well did you know this?
1
Not at all
2
3
4
5
Perfectly
14
Q

In this type of arrhythmia, the P-R interval progressively gets larger until there is a dropped QRS complex (P wave without a QRS complex).

First degree heart block
Third degree heart block
Second degree Mobitz type I heart block
Second degree Mobitz type Il heart block

A

Answer: Second degree Mobitz type I heart block

Explanation
The correct answer is Second degree Mobitz type I heart block, also known as the Wenckebach beat. In a Mobitz type Il block, the P-R interval duration is unchanged and occasionally you will observe a P wave without a QRS complex. These blocks are thought to be caused by a refractory AV node which is not responding to a delayed atrial depolarization. In a 1st degree heart block, you will observe an increased P-R interval and no dropped QRS. In 3rd degree heart blocks, the P waves are unassociated from the QRS complex.

The arrhythmia described is a Second degree Mobitz type I heart block (Wenckebach beat), where the P-R interval progressively lengthens until a QRS complex is dropped. In contrast, Mobitz type II shows no P-R interval change but occasional dropped QRS complexes. First-degree heart block has an increased P-R interval without dropped QRS, and third-degree heart blocks show no association between P waves and QRS complexes.

How well did you know this?
1
Not at all
2
3
4
5
Perfectly
15
Q

A cow presents due to a decrease in milk production. On physical exam, you were unable to hear the heart clearly (it is muffled). You decide to perform an ECG and immediately recognize that this cow has electrical alternans. What is most likely to be occurring?

Right atrial enlargement
Pericardial effusion
Left ventricular enlargement
Right ventricular enlargement
Left atrial enlargement

A

Answer: Pericardial effusion

Explanation
The correct answer is pericardial effusion. Electrical alternans is a classic finding in cases where there is pericardial effusion. Electrical alternans is characterized by different amplitudes of multiple successive R waves in the QRS complex. This is thought to be a result of the heart shifting around within the fluid filled compartment of the pericardial sac.

The most likely condition is pericardial effusion. Electrical alternans, seen as varying R wave amplitudes in the QRS complex, is a classic sign of pericardial effusion. This occurs due to the heart shifting within the fluid-filled pericardial sac, leading to muffled heart sounds.

How well did you know this?
1
Not at all
2
3
4
5
Perfectly
16
Q

A 3-year old Jersey heifer is not doing well and is euthanized with a post-mortem performed in the field. Below is an image of her heart, with the right atrium and right ventricle opened. What is your diagnosis?

Tricuspid valve endocarditis
Ventricular septal defect
Cardiac neoplasia
No significant findings
Tricuspid valve dysplasia

A

Answer: Tricuspid valve endocarditis

Explanation
The correct answer is tricuspid valve endocarditis. Hemorrhagic and white masses are seen on all leaflets of the tricuspid valve which makes endocarditis the most likely diagnosis in a young heifer. Neoplasia is unlikely on the valve and at this age; tricuspid valve dysplasia would be seen as short, thick chordae but no masses; a VSD would be seen as a hole between the right and left ventricle which is not apparent here.

The diagnosis is tricuspid valve endocarditis. Hemorrhagic and white masses on all tricuspid valve leaflets are indicative of endocarditis, especially in a young heifer. Neoplasia is unlikely at this age and location. Tricuspid valve dysplasia would show thickened chordae without masses, and a ventricular septal defect would appear as a hole between ventricles, which is not present here.

Bovine Cardiac Diseases Overview:
• Cardiac diseases in bovine primarily affect the pericardium, myocardium, and endocardium. Congenital heart diseases, such as ventricular septal defects, are also noted but rare.

Pericardial Diseases:
• Common Diseases: Traumatic pericarditis, pericarditis secondary to pleural or lung infections, neoplastic effusions due to lymphoma or mesothelioma, and idiopathic aseptic pericarditis.
• Pathophysiology: Inflammation of the pericardium due to trauma or infection leads to the accumulation of serous or fibrinous products. The proximity of the reticulum to the pericardium in cattle increases the risk of traumatic pericarditis, particularly due to hardware disease.
• Symptoms: Tachycardia, muffled heart sounds, jugular vein distension, ventral edema, anorexia, decreased milk production, and pyrexia. In advanced cases, there may be diarrhea, congested mucous membranes, and increased capillary refill time.
• Diagnostics: Clinical signs, radiographic findings (enlarged cardiac silhouette), ultrasonography, echocardiography (gold standard), and electrocardiography (base-apex lead system).
• Treatment: Digoxin to improve venous return and reduce afterload, antibiotics like amoxicillin or aminoglycosides, pericardiocentesis for fluid drainage, and in severe cases, surgical pericardiotomy or pericardiectomy.
• Prognosis: Generally poor, with treatments aimed at short-term survival for calving or superovulation.

Myocardial Diseases:
• Common Diseases: Dilated cardiomyopathy (DCM) is the most common myocardial disease in cattle, often affecting Holstein-Friesian cattle due to possible genetic factors.
• Pathophysiology: DCM involves myocardial dysfunction due to various etiologies, including immune dysfunction, viral infections, and genetic predispositions. Other causes include toxic effects from ionophores or selenium deficiency.
• Symptoms: Subcutaneous edema, jugular vein distension, ascites, heart murmurs, diarrhea, and muffled heart sounds.
• Diagnostics: Clinical signs, echocardiography, and biochemical analysis (elevated liver enzymes, creatinine, and transferrin).
• Treatment: No specific treatment for DCM. Supportive care includes correcting electrolyte imbalances and treating underlying causes.
• Prognosis: Fatal in primary DCM; prognosis improves with early detection and resolution of secondary causes.

Endocardial Diseases:
• Common Diseases: Bacterial endocarditis, often secondary to chronic infections (e.g., mastitis, metritis, or abscesses).
• Pathophysiology: Persistent bacteremia leads to endocardial infection, with common bacteria being Arcanobacterium pyogenes and streptococci.
• Symptoms: Weight loss, decreased milk production, tachycardia, murmurs, jugular distension, fever, and edema.
• Diagnostics: Hematological findings (anemia, leukocytosis), echocardiography (vegetative lesions), and blood cultures.
• Treatment: Long-term antibiotic therapy (minimum of four weeks) to arrest valve destruction and eliminate sepsis.
• Prognosis: Dependent on early diagnosis; prognosis worsens with delayed treatment.

Congenital Heart Diseases:
• Common Conditions: Ventricular septal defects, atrial septal defects, and patent ductus arteriosus.
• Symptoms: Ill-thrift, poor growth, respiratory disease, and sudden death.
• Diagnostics: Echocardiography and radiography for assessing congenital defects.
• Treatment: No surgical or medicinal treatments available for congenital heart disease in cattle.
• Prognosis: Varies from guarded to poor.

Cardiac Neoplasia:
• Common Tumors: Lymphoma is the most common, along with others like hemangiosarcoma, fibrosarcoma, and mesothelioma.
• Symptoms: Similar to pericarditis or congestive heart failure, with additional signs of neoplasia like lymphadenopathy.
• Diagnostics: Peripheral lymph node aspirates, echocardiography, and pericardiocentesis.
• Treatment: Palliative treatments like thoracoscopic pericardiotomy or repeated pericardiocentesis; antibiotics and diuretics for symptom management.
• Prognosis: Generally poor, with short-term survival following palliative care.

Congestive Heart Failure (CHF):
• Pathophysiology: Result of progressive heart disease leading to fluid retention and edema.
• Symptoms: Syncope, exercise intolerance, weakness, effusion, and edema.
• Diagnostics: Clinical signs, echocardiography, and blood gas analysis.
• Treatment: Digoxin for initial therapy, potassium supplementation, and cautious use of furosemide to resolve edema.
• Prognosis: Short-term improvement possible with medicinal therapy; long-term outcomes are generally poor.

https://www.researchgate.net/profile/Ashwani-Kumar-54/publication/336121748_An_overview_on_the_diagnostic_and_therapeutic_aspects_of_cardiac_diseases_in_bovine/links/5d8f9b26299bf10cff17f7b4/An-overview-on-the-diagnostic-and-therapeutic-aspects-of-cardiac-diseases-in-bovine.pdf

How well did you know this?
1
Not at all
2
3
4
5
Perfectly
17
Q

This 3 1/2 month old calf presents for weakness and failure to thrive. On physical exam you find normal temperature but rapid heart and respiratory rates. A loud holosystolic murmur is heard on both sides of the thorax with maximal intensity in the region of the tricuspid valve. It is also clearly heard on the left side with maximal intensity near the pulmonic valve (far forward). What is the most likely diagnosis?

Endocarditis
Ventricular septal defect
Patent ductus arteriosus
Patent foramen ovale
Traumatic reticulopericarditis

A

Answer: Ventricular septal defect

Explanation
The correct answer is ventricular septal defect. In order to answer this question, you need to “play the odds” and know that VSDs are the most common congenital heart defect in cattle, and they usually present at about 2-3 months of age because they are failing to thrive and/or are developing pulmonary edema and secondary pneumonia. This defect results in oxygenated blood being pushed back into the right atrium and ventricle. In addition, you will see right ventricular dilation and hypertrophy. The increased flow through the right side creates a relative pulmonic stenosis, and thus the murmur is also heard on the left side in the area of the pulmonic valve. VSD is the most common congenital defect in cattle.

The most likely diagnosis is a ventricular septal defect (VSD). VSDs are the most common congenital heart defect in cattle, usually presenting at 2-3 months of age with failure to thrive. The defect causes oxygenated blood to flow back into the right atrium and ventricle, leading to right ventricular dilation, hypertrophy, and a murmur heard on both sides of the thorax.

How well did you know this?
1
Not at all
2
3
4
5
Perfectly
18
Q

Which of the following is not a feature of Tetralogy of Fallot?

Pulmonic valve stenosis
Aortic valve stenosis
Ventricular septal defect
Right ventricular hypertrophy
Dextropositioned aorta

A

Answer: Aortic Valve Stenosis

Explanation
The correct answer is aortic valve stenosis. Excellent job if you remembered this one! In case you don’t remember the four features of Tetralogy of Fallot, they are: Right ventricular hypertrophy, ventricular septal defect, dextropositioned aorta (over-riding aorta), and pulmonic valve stenosis. Clinical signs include a bilateral basilar murmur, right sided heart failure, cyanosis, and secondary bacterial endocarditis.

***PowerLecture: Cardiology Disease Specific Pearls of Wisdom

* Tetralogy of Fallot in Animals

Definition:

•	Congenital heart defect with four components: pulmonic stenosis, ventricular septal defect, right ventricular hypertrophy, and overriding aorta.

Affected Species:

•	Dogs (e.g., Keeshonds, English Bulldogs), cats.

Clinical Signs:

•	Cyanosis, exercise intolerance, stunted growth, collapse, seizures.
•	Loud ejection heart murmur, precordial thrill.

Diagnosis:

•	Echocardiography: Confirms structural defects.
•	Radiographs: Right heart enlargement.
•	ECG: Right ventricular enlargement.

Treatment:

•	Beta-adrenergic blockers to manage outflow obstruction.
•	Periodic phlebotomy for polycythemia.
•	Surgical options: Palliative techniques, valvuloplasty.

Control:

•	Manage based on severity, supportive care, and specific surgical interventions.
How well did you know this?
1
Not at all
2
3
4
5
Perfectly
19
Q

While performing a physical exam on a cow, you notice a rapid irregular heart rate with no atrial sounds or compensatory pauses. There is no regular rhythm to the irregularity. What is this?

Sinus arrhythmia
Ventricular tachycardia
Bradyarrhythmia
Atrial fibrillation

A

Answer: Atrial fibrillation

Explanation
This is descriptive for atrial fibrillation. If you ran an ECG, you would find no P waves and variable intervals between QRS complexes. Atrial fibrillation is the most common arrhythmia in cattle, often due to Gl or electrolyte abnormalities.

The condition described is atrial fibrillation. It presents as a rapid, irregular heart rate without atrial sounds or compensatory pauses, and an irregular rhythm. On ECG, you’d find no P waves and variable intervals between QRS complexes. Atrial fibrillation is common in cattle, often linked to GI or electrolyte abnormalities.

How well did you know this?
1
Not at all
2
3
4
5
Perfectly
20
Q

You examine a valuable beef bull that was recently purchased in Texas and taken to the mountains of Colorado. The pasture is a mountain meadow at 10,000 feet elevation. The bull has developed edema of the brisket and ventral thorax, submandibular edema, dyspnea and tachypnea. Rectal temperature is normal. The HR is 90, and heart sounds are clearly audible without murmurs. What is your diagnosis?

Ventricular septal defect
Valvular endocarditis
Cardiomyopathy
High mountain disease
Traumatic pericarditis

A

Answer: High mountain disease

Explanation
The key to this question is the altitude mentioned in the question and accompanying clinical signs. Also called brisket disease and high altitude disease, this condition can be fatal and is brought on by elevations above 6000 feet. Hypoxic vasoconstriction (worse in some lines of cattle) causes pulmonary hypertension, which leads to cor pulmonale, which is secondary cardiac disease including right heart enlargement and failure.

The diagnosis is high mountain disease, also known as brisket disease or high altitude disease. This condition occurs at elevations above 6000 feet, causing hypoxic vasoconstriction, leading to pulmonary hypertension, cor pulmonale, right heart enlargement, and failure. The clinical signs include brisket edema, submandibular edema, dyspnea, and tachypnea, with normal rectal temperature and clear heart sounds.

Bovine High-Mountain Disease (BHMD): NAVLE Study Guide
Definitions and Etiology
• BHMD: Also known as “Brisket Disease,” it occurs in cattle at high altitudes due to hypoxia-induced pulmonary hypertension.
• Causative Factors: Genetic predisposition, high altitude, pulmonary vascular shunting, and locoweed poisoning (Oxytropis and Astragalus spp.).

Pathophysiology
• Mechanism: Hypoxia causes pulmonary arterial vasoconstriction and hypertrophy, leading to pulmonary hypertension, right ventricular hypertrophy, and eventual congestive heart failure.
• Vascular Remodeling: Chronic exposure results in loss of peripheral pulmonary arteries and increased pulmonary resistance.

Clinical Signs
• Symptoms: Brisket edema, pleural effusion, jugular distension, cyanosis, labored breathing, and sometimes exophthalmos. Animals may become recumbent and show signs of right heart failure.
• Advanced Signs: Ascites, profuse diarrhea, and decreased appetite due to intestinal venous hypertension.

Diagnosis
• Pulmonary Arterial Pressure (PAP): Elevated PAP (>50 mmHg) and visible signs of right ventricular failure. Radiographic and ultrasonographic findings support the diagnosis.
• Differential Diagnosis: Includes other causes of congestive heart failure, such as pericarditis, lymphosarcoma, and myocarditis.

Treatment

•	Primary Treatment: Moving affected cattle to lower altitudes, supportive care, diuretics, and thoracocentesis. Affected cattle should not be returned to high altitudes.
•	Prevention: Selective breeding for animals with lower PAP measurements, minimizing exposure to locoweed, and addressing concurrent diseases.

Key Points for Veterinary Professionals
• Genetic Selection: Breeding programs should focus on selecting cattle resistant to hypoxia.
• Environmental Management: Control exposure to high-altitude stressors and toxic plants like locoweed.
• Ongoing Research: Studies on genetic markers and improving diagnostic techniques for BHMD are critical for future prevention and management strategies.

How well did you know this?
1
Not at all
2
3
4
5
Perfectly
21
Q

When performing a necropsy on a cow, you find multiple abnormalities including abomasal ulcers, interstitial pneumonia, pericardial effusion, and mastitis. You also see that the liver has a brown- (tan) and red-mottled lobular pattern described as a nutmeg liver. Which of these conditions in the cow could lead to the necropsy findings in the liver?

Pericardial effusion
Abomasal ulcers
Interstitial pneumonia
Mastitis

A

Answer: Pericardial effusion

Explanation
The appearance of a nutmeg liver occurs because there are red central veins and sinusoids between tan areas of swollen hepatocytes. This occurs with right sided heart failure from passive congestion of the sinusoids of hepatocellular hypoxia. Pericardial effusion causes right sided heart failure and is the cause of the liver’s appearance.

The condition that could lead to the nutmeg liver appearance in the cow is pericardial effusion. This occurs due to right-sided heart failure, resulting in passive congestion of the liver’s sinusoids and hepatocellular hypoxia, causing the characteristic red and tan mottled pattern seen on necropsy.

How well did you know this?
1
Not at all
2
3
4
5
Perfectly
22
Q

Bovine growth hormone is also called bovine somatotropin (BST). For which approved purpose is BST sometimes given to cattle?

-Produce larger cows capable of avoiding dystocia
-Increase the rate of growth in feedlot cattle
-Prevent milk fever in dairy cattle
-Produce twinning in dairy cows
-Promote milk production

A

Answer: Promote milk production

Explanation
BST or rBST is given by injection to lactating dairy cows every two weeks to increase the amount of milk produced and to prolong lactation.

Bovine growth hormone, or bovine somatotropin (BST), is given to lactating dairy cows to promote milk production. It is typically administered by injection every two weeks to increase milk yield and extend the lactation period.

How well did you know this?
1
Not at all
2
3
4
5
Perfectly
23
Q

Bovine growth hormone is also called bovine somatotropin (BST). For which approved purpose is BST sometimes given to cattle?

-Produce larger cows capable of avoiding dystocia
-Increase the rate of growth in feedlot cattle
-Prevent milk fever in dairy cattle
-Produce twinning in dairy cows
-Promote milk production

A

Answer: Promote milk production

Explanation
BST or rBST is given by injection to lactating dairy cows every two weeks to increase the amount of milk produced and to prolong lactation.

Bovine growth hormone, or bovine somatotropin (BST), is given to lactating dairy cows to promote milk production. It is typically administered by injection every two weeks to increase milk yield and extend the lactation period.

How well did you know this?
1
Not at all
2
3
4
5
Perfectly
24
Q

You examine a 3-year old beef cow which has been acting depressed for the last 24 hours, and has failed to remain with the herd. The cow’s temperature is 104F, (40 C), heart rate is 100 bpm and is pounding loudly, and the respiratory rate is elevated at 42 breaths per minute. The mucous membranes are pale. You take a blood sample back to your nearby clinic and observe the blood smear (see image). The PCV is 20% (24-46%). Of the following, what is the single best, most cost-effective treatment for this cow?

  • Long acting tetracycline injection
  • Whole blood transfusion
  • Procaine penicillin injection
  • Vaccinate against parainfluenza-3
  • Imidocarb dipropionate injection
A

Answer: Long acting tetracycline injection

Explanation
The blood smear shows multiple round Anaplasma marginale along the outer membrane of red blood cells.

Anaplasma marginale is an obligate intracellular bacterium. It is the most prevalent tick-borne pathogen of cattle worldwide and is endemic in tropical and sub-tropical regions. Canada is Anaplasma-free.
Infection causes large scale destruction of red blood cells and fever, weight loss, respiratory distress, abortion, and often death.

Although some cattle recover, they can become lifetime carriers as reservoirs. Anaplasma ovis can occur in sheep and goats.

Anaplasma can often be treated successfully with a single injection of long acting tetracycline (20 mg/kg).
When the PCV is below 12%, a blood transfusion may also be needed. In addition, the animal should be given ready access to feed and water.

Even with treatment, animals may remain carriers after all clinical signs resolve.

The best and most cost-effective treatment for this cow with Anaplasma marginale infection is a single injection of long-acting tetracycline (20 mg/kg). Anaplasma marginale is a tick-borne intracellular bacterium that causes red blood cell destruction, leading to fever, weight loss, respiratory distress, and anemia. A blood transfusion may be necessary if the PCV drops below 12%.

Anaplasmosis in Ruminants - Comprehensive Veterinary Information

Definitions and Terminology:

•	Anaplasmosis: A tick-borne disease of ruminants caused by intracellular bacteria infecting red blood cells, leading to severe anemia and fever.

Causative Agents:

•	Pathogens: Anaplasma marginale, A. centrale, A. ovis, A. phagocytophilum.

Physiopathology:

•	Transmission: Ticks (Dermacentor, Rhipicephalus), mechanical via biting dipterans, contaminated needles.
•	Pathogenesis: Bacteria infect erythrocytes, causing their destruction by the immune system, leading to anemia and jaundice.

Clinical Findings:

•	Symptoms: Progressive anemia, fever, weight loss, decreased milk production, icterus. Severe cases may lead to death.

Diagnosis:

•	Tests: Blood smears (Giemsa-stain), serologic tests (ELISA), PCR.
•	Microscopic Findings: Anaplasma organisms in erythrocytes.

Treatment:

•	Antibiotics: Tetracyclines (e.g., oxytetracycline 20 mg/kg IM), imidocarb (1.5 mg/kg SC).
•	Supportive Care: Blood transfusions for severely affected animals.

Control and Prevention:

•	Vaccination: Use of live vaccines (e.g., A. centrale) in endemic areas.
•	Tick Control: Acaricides and environmental management.
How well did you know this?
1
Not at all
2
3
4
5
Perfectly
25
Q

Nine out of 100 adult cattle are found weak, depressed, and staring into space. On physical exam, they are found to be pale, icteric, and febrile. None of them have hemoglobinuria. The nine cattle all arrived at the ranch approximately one month ago. What is your most likely diagnosis?

  • Leptospirosis
  • Anthrax
  • Bacillary hemoglobinuria
  • Anaplasmosis
A

Answer: Anaplasmosis

Explanation
The correct answer is Anaplasmosis. The causative agent is Anaplasma marginale. The clinical signs are fairly straight forward, and you need to pick up on a couple key features. The fact that new adults are sick is one feature because cattle are resistant to clinical Anaplasmosis as calves. Also, it should be remembered that hemoglobinuria with Anaplasmosis is never seen because it is all extravascular hemolysis. Since there is no hemoglobinuria, leptospirosis, bacillary hemoglobinuria, and anthrax can be ruled out. Additionally, icterus with anthrax is not seen.

The most likely diagnosis is Anaplasmosis, caused by Anaplasma marginale. Key signs include weakness, depression, pallor, icterus, and fever, especially in newly arrived adult cattle. Unlike other conditions, hemoglobinuria is absent due to extravascular hemolysis. This rules out leptospirosis, bacillary hemoglobinuria, and anthrax, where hemoglobinuria and different signs occur.

How well did you know this?
1
Not at all
2
3
4
5
Perfectly
26
Q

You examine a 7-year-old dairy cow with the presenting complaint of poor milk production and weight loss. You find a normal TPR, but the cow is in poor body condition compared to others in the string, and has multiple enlarged lymph nodes at widely scattered locations (see image). You suspect bovine leukosis virus (BLV) has caused clinical lymphoma. Which of the following tests would be the most definitive in making the diagnosis of lymphoma?

  • WBC count
  • Aspirate lymph node and make slide
  • Lymphocyte count
  • AGID for BLV antibodies
  • ELISA for BLV antibodies
A

Answer: ELISA for BLV antibodies

Explanation
The best answer to this question is ELISA for BLV antibodies. A biopsy is definitive if you can demonstrate antigen with immunohistochemistry, but many diagnostic labs do not have this capability, so this answer option was not included for this question. The ELISA for BLV only reveals that the cow has antibodies to BLV virus, but not that she has clinical lymphoma. However, multiple enlarged lymph nodes and a positive BLV antibody test are strongly suggestive of lymphoma.

A negative ELISA test would rule out BLV and associated lymphoma.

Aspiration of lymph nodes can lead to false positives, as many inflamed reactive nodes in cattle can have cells that look neoplastic.

The AGID for BLV antibodies is less sensitive than ELISA. The WBC and lymphocyte count lacks sensitivity and specificity.

The most definitive test for diagnosing lymphoma in a cow with suspected bovine leukosis virus (BLV) is the ELISA for BLV antibodies. While it indicates BLV exposure, it strongly suggests lymphoma when paired with multiple enlarged lymph nodes. A negative ELISA rules out BLV and associated lymphoma. Other methods, such as lymph node aspiration, can lead to false positives, and AGID is less sensitive than ELISA.

Bovine Leukosis: Comprehensive Information

Definition

•	Bovine Leukosis (Bovine Lymphosarcoma): A cancerous disease caused by Bovine Leukemia Virus (BLV), an oncogenic retrovirus.

Etiology and Transmission

•	Causative Agent: Bovine Leukemia Virus (BLV), a C-type oncogenic retrovirus.
•	Transmission: Primarily horizontal through blood transfer (e.g., contaminated needles, dehorning tools), and rarely vertical (transplacental or via colostrum).

Epidemiology

•	Prevalence: High in US dairy herds (~46.5%) and lower in beef herds (~10.3%).

Pathogenesis

•	Infection Outcomes:
•	Asymptomatic: Most animals show no signs.
•	Persistent Lymphocytosis: ~29% develop this benign condition.
•	Lymphosarcoma: <5% develop cancerous tumors.

Clinical Findings

•	Juvenile Lymphosarcoma: Weight loss, fever, dyspnea, bloat, and posterior paresis.
•	Thymic Lymphosarcoma: Cervical swelling, dyspnea, bloat, jugular distention, tachycardia.
•	Cutaneous Lymphosarcoma: Cutaneous plaques, enlarged lymph nodes.

Diagnosis

•	Serology: ELISA and PCR for BLV detection.
•	Cytology/Histology: Biopsy for diagnosing lymphosarcoma.

Treatment and Control

•	No Treatment: No effective treatment; symptomatic care with corticosteroids.
•	Control Measures: Testing and culling infected animals, disinfection of equipment, using single-use needles.

Prevention

•	Blood Precautions: Avoiding exposure to blood from infected animals.
•	Colostrum Management: Using colostrum from seronegative cows.
•	Farm Practices: Cleaning equipment and facilities, controlling flies.

Zoonotic Risk

•	Human Risk: Potential, but not well-established, zoonotic risk through ingestion of unpasteurized dairy products.
How well did you know this?
1
Not at all
2
3
4
5
Perfectly
27
Q

You are called by a rancher and asked to examine a 3-year old beef cow he purchased 4 weeks ago and turned out in the hills of California. The complaint is that she appears weak and disoriented, and even a little belligerent. On exam, you find T=105F or 40.6 C, HR=110, and RR=45. The heart is pounding very loudly and you note a number of ticks on her. Mucous membranes are pale (see image of her vulva), but urine and feces appear normal. Other parameters are not notably abnormal. Which of the following diseases best fits this cow’s clinical signs?

  • Piroplasmosis (Babesiosis)
  • Purpura hemorrhagica
  • Anaplasmosis
  • Porphyria
  • Bacillary hemoglobinuria (Clostridium novyi type D)
A

Answer: Anaplasmosis

Explanation
This tick-borne disease has an incubation period of 2 to 4 weeks, and causes these exact signs. Tetracycline is the best treatment.

With piroplasmosis or Clostridium novyi type D infection (bacillary hemoglobinuria), one would expect intravascular hemolysis and dark urine.

How well did you know this?
1
Not at all
2
3
4
5
Perfectly
28
Q

You are in the process of giving a cow a blood transfusion and begin to notice tachycardia, dyspnea, ptyalism, and rough lung sounds. Which of the following is NOT a treatment option?

  • Epinephrine
  • Diazepam
  • Dexamethasone phosphate
  • Antihistamine
  • Oxygen
A

Answer: Diazepam

Explanation
The correct answer is diazepam. The clinical signs that are described are those of an anaphylactic reaction to the transfusion. Diazepam will not help you in this situation. Epinephrine, antihistamines, dexamethasone, IV fluids and oxygen are all possible treatment options.

How well did you know this?
1
Not at all
2
3
4
5
Perfectly
29
Q

You are consulting on a shipment of imported cattle that have arrived in Canada from South America. Ticks have been found on several cattle while they are in quarantine and yesterday one became ill with fever and hemolysis. Hemoglobinuria was observed. A blood smear from this animal is sent to you (see image). Given their origin, the ticks and hemoglobinuria observed, and what you see in the smear, which of the following is the most likely diagnosis?

  • Transport stress induced hemolytic syndrome
  • Babesiosis
  • Copper toxicity
  • Bacillary hemoglobinuria
  • Anaplasmosis
A

Answer: Babesiosis

Explanation
Babesia bigemina and Babesia bovis are red cell parasites of cattle found in the Americas and spread mainly by Boophilus ticks. Other species of Babesia can infect man and other animals and are found on most continents.

The disease they cause is characterized by hemolytic anemia, hemoglobinuria (which differentiates it from anaplasmosis), icterus, and death. Bovine babesiosis is also called piroplasmosis, red water, Texas fever, or tick fever. Eradication of Boophilus ticks has been an effective means of control in the United States.

The most likely diagnosis is babesiosis, caused by Babesia bigemina or Babesia bovis, red cell parasites spread by Boophilus ticks in the Americas. Babesiosis leads to hemolytic anemia, hemoglobinuria (distinguishing it from anaplasmosis), icterus, and death. Known as piroplasmosis, red water, Texas fever, or tick fever, eradication of Boophilus ticks controls the disease.

Babesiosis in Animals - Comprehensive Veterinary Information

Definitions and Terminology:

•	Babesiosis: A disease caused by Babesia spp., transmitted by ixodid ticks, affecting various mammals, including cattle, horses, sheep, goats, pigs, dogs, and cats.

Causative Agents:

•	Pathogens: Various Babesia species, e.g., B. bovis, B. bigemina in cattle; B. canis, B. gibsoni in dogs.

Physiopathology:

•	Transmission: Tick bites, transovarial transmission in ticks.
•	Pathogenesis: Intraerythrocytic protozoan parasites cause erythrocyte destruction, leading to anemia, hemolysis, and potential organ damage.

Clinical Findings:

•	Symptoms: Fever, anemia, jaundice, hemoglobinuria, lethargy, weight loss, and in severe cases, CNS involvement.
•	Postmortem Lesions: Enlarged spleen, swollen liver, congested kidneys, and general anemia.

Diagnosis:

•	Tests: Blood smear microscopy, PCR assays, serologic tests.
•	Microscopic Examination: Identification of parasites in erythrocytes.

Treatment:

•	Babesiacides: Imidocarb dipropionate (1.2 mg/kg SC), diminazene aceturate (3.5 mg/kg IM).
•	Supportive Care: Anti-inflammatories, corticosteroids, fluid therapy, blood transfusions.

Control and Prevention:

•	Tick Control: Acaricides, resistant breeds, management practices.
•	Vaccination: Live attenuated vaccines in some countries.
How well did you know this?
1
Not at all
2
3
4
5
Perfectly
30
Q

A beef cow presents for decreased milk production. On physical exam, the cow has a fever of 104.5F. Her urine is yellow, and the dipstick tests are normal. You make a blood smear and see an organism at the margin of the red blood cells. What is your treatment of choice for this animal?

  • Procaine Penicillin
  • Oxytetracycline
  • Chloramphenicol
  • Corticosteroids
A

Answer: Oxytetracycline

Explanation
The correct answer is oxytetracycline. The cow has Anaplasmosis, a rickettsial organism that is transmitted by ticks. Infected calves have a low mortality rate, but adult cattle have a 20-50% mortality rate with this disease.

How well did you know this?
1
Not at all
2
3
4
5
Perfectly
31
Q

You are working with a farmer who is having trouble with calves between the age of 2 weeks and 6 months. They are alert, but weak, dyspneic and die suddenly. On necropsy they have pale cardiac and skeletal muscles. What is the farmer’s problem?

  • Lightning strike
  • Sorghum toxicity
  • Copper deficiency
  • Selenium deficiency
A

Answer: Selenium deficiency

The correct answer is selenium deficiency. The pale muscle and clinical signs are classic for vitamin E and selenium deficiency. This is important to remember! Other things that should be on your differential list for this case include cardiotoxic plants.

Nutritional Myopathies in Ruminants and Pigs: Comprehensive Information

Nutritional Myodegeneration (NMD)

Definition: Acute, degenerative disease of cardiac and skeletal muscle due to selenium or vitamin E deficiency in young, rapidly growing animals.

Causative Agents:

•	Selenium Deficiency: More critical than vitamin E deficiency.
•	Vitamin E Deficiency: Common in animals fed poor-quality hay or straw.

Clinical Signs:

•	Cardiac Muscle: Respiratory distress, arrhythmias, sudden death.
•	Skeletal Muscle: Muscle weakness, stiffness, difficulty rising, respiratory distress, dysphagia.

Diagnosis:

•	Laboratory Tests: Increased CK, AST, LDH levels.
•	Definitive Diagnosis: Low selenium in blood/liver, low vitamin E in plasma.

Treatment:

•	Injectable Selenium: 0.055–0.067 mg/kg IM or SC.
•	Vitamin E Supplements: Oral supplementation, injectable products with 300-500 IU vitamin E/mL.

Prevention:

•	Dietary Management: Selenium supplementation in feed, periodic injections, high-quality forage.

Hypokalemic Myopathy

Definition: Severe muscle weakness due to low serum potassium (< 2.0 mmol/L) in dairy cattle, often associated with isoflupredone acetate administration.

Clinical Signs: Weakness, recumbency, abnormal head/neck position, rumen hypomotility, anorexia, tachycardia.

Diagnosis: Clinical signs, serum potassium < 2.0 mmol/L, increased CK and AST.

Treatment: Potassium chloride supplementation, addressing primary cause of ketosis and anorexia.

Nutritional Myopathy of Pigs

Specific Conditions: Mulberry heart disease, hepatosis dietetica.

Causative Agents: Selenium or vitamin E deficiency, iron dextran administration.

Clinical Findings:

•	Mulberry Heart Disease: Sudden death, pericardial effusion, myocardial necrosis.
•	Hepatosis Dietetica: Subcutaneous edema, serous cavity transudate, liver necrosis.

Diagnosis: History, clinical signs, necropsy, histology, serum/tissue selenium and vitamin E levels.

Treatment: Selenium and vitamin E supplementation, injections to affected pigs and sows in late gestation.

How well did you know this?
1
Not at all
2
3
4
5
Perfectly
32
Q

A 3-year-old beef cow presents with 4 days’ history of poor appetite and rapid weight loss. The cow had calved 10 days previously and had prolapsed her uterus immediately after delivery of a large dead calf that had been “hip-locked”. The cow had received no antibiotics because the uterus was replaced easily and the placenta was already detached from the caruncles. The cow is now weak and depressed. The rectal temperature is 100.8F (38.2 C). The ocular and oral mucous membranes appear slightly congested. The heart rate is 96 beats per minute. The respiratory rate is 30 breaths per minute with a slight abdominal component. The ruminal contractions are reduced and the cow has passed only scant feces. Rectal examination reveals that the uterus is still extending well beyond the pelvic inlet. Vaginal examination reveals approximately 200 mL of foul-smelling brown fluid. Despite treatment with intravenous oxytetracycline and flunixin meglumine for suspected metritis the cow died suddenly 2 days later. Postmortem examination reveals septic metritis and the liver lesions shown here. What condition is affecting the liver?

  • Fasciola hepatica
  • Mycobacterium paratuberculosis
  • Clostridium chauvei
  • Fusobacterium necrophorum
A

Answer: Fusobacterium necrophorum

Explanation
The image shows widespread severe liver abscesses caused by Fusobacterium necrophorum. Fusobacterium necrophorum is a gram-negative, obligate anaerobic bacterium that is a component of normal rumen microflora. In this case, the metritis caused bacteremic spread of the organism which can go to the liver and release local toxins leading to damage and potential abscess formation. Typical Fusobacterium lesions are yellow and spherical with irregular outlines. A diagnosis could be confirmed in this case by culture of the uterine contents and liver. Affected livers are condemned at slaughter. Adhesions to surrounding organs or the diaphragm may require carcass trimming. Liver abscess can also lead to disease associated with thrombosis of the vena cava.

The utility of prophylactic antibiotic therapy in large animal practice is unclear and this practice is often questioned with respect to cost, food safety (milk and meat withdrawal periods), and selection for antibiotic resistance in certain bacterial species. However, in this particular case, prophylactic antibiotics after replacement of the uterine prolapse may have prevented/cleared the uterine infection and/or prevented the bacteremia seeding the liver with fatal consequences.

Fasciola hepatica is the liver fluke of cattle. While Fasciola can cause liver lesions, the lesions typically appear more as tracks and/or hemorrhage. Animals with fasciolosis typically present with abdominal pain or jaundice acutely. Chronically, animals become anemic or anorectic.

Clostridium chauvei is the cause of blackleg in cattle and sheep and causes lameness and fever.

Mycobacterium paratuberculosis is the cause of Johne’s Disease which results in chronic wasting.

The liver condition in this case is caused by Fusobacterium necrophorum, a gram-negative anaerobic bacterium that can spread through bacteremia from metritis, resulting in liver abscesses. These abscesses are typically yellow, spherical, and irregular. Diagnosis is confirmed by culturing uterine contents and liver samples. Prophylactic antibiotics might have prevented this bacteremia after the uterine prolapse. Other liver-affecting conditions like Fasciola hepatica or Clostridium chauvei cause different lesions and clinical signs.

Key Information on Vena Caval Thrombosis and Metastatic Pneumonia in Cattle

Etiology

•	Primary Cause: Ruminal acidosis leading to rumenitis and liver abscessation, with potential for thrombus formation in the caudal vena cava.
•	Bacteria Involved: Fusobacterium necrophorum, Trueperella pyogenes, staphylococci, streptococci, Escherichia coli.

Clinical Findings

•	Acute Signs: Respiratory distress.
•	Chronic Signs: Weight loss, chronic coughing, tachypnea, tachycardia, hemic murmurs, coughing, pale mucous membranes, increased lung sounds, hemoptysis, and epistaxis.

Diagnosis and Lesions

•	Diagnosis: Clinical signs, history, and postmortem examination.
•	Post-Mortem Lesions: Thrombus in the vena cava, hepatic abscesses, suppurative pneumonia with multifocal abscesses and aneurysms.

Treatment and Control

•	Treatment: Antimicrobial therapy is usually ineffective; focus on supportive care.
•	Control: Nutritional management, gradual adaptation to high-grain diets.
How well did you know this?
1
Not at all
2
3
4
5
Perfectly
33
Q

A 2-year old Holstein heifer presents for depression and decreased milk production over the previous week. On physical exam, it is noted her posterior shape is “papple” and she is slightly dehydrated. T=100.2 (37.9 C), P=62, R=28. She does not have episcleral injection. Rumen contractions are not present. Xiphoid pain response is negative and you are able to hear a monotone ping in the rumen on the left side from the 11th rib to the hip. On ballottement and rectal exam a large fluid-filled rumen is palpable. Which of the following is the most likely diagnosis?

  • Grain overload (rumen acidosis)
  • Failure of omasal transport
  • Abomasal torsion or volvulus
  • Left displaced abomasum (LDA)
A

Anwer: Failure of omasal transport

Explanation
The correct answer is failure of omasal transport, a form of vagal indigestion, which leads to an accumulation of fluid and some dorsal gas in the rumen, lack of motility, and inappetance. TRP is a common cause of vagal indigestion; the negative grunt test for xiphoid pain is the result of chronicity and formed adhesions that are no longer acutely painful. It is difficult to clinically distinguish between omasal transport failure and pyloric outflow failure, except that the latter usually has more profound acid-base and electrolyte disturbances, and the cow will appear sicker as a result.

Abomasal torsion can be immediately ruled out because there is no abomasal ping on the right. In addition, if it were a right displaced abomasum, the cow would be showing more signs of distress and systemic disease such as episcleral injection and an elevated heart rate. Left displaced abomasum can be ruled out because the ping is monotone (LDA ping is variable in pitch) and the ping extends all the way back to the hip. Further, the large fluid filled rumen is rectally palpable, whereas a cow with LDA would have an empty rumen. Grain overload (rumen acidosis) would have a large fluid filled rumen, but the cow would be very sick with scleral injection and rapid heart rate.

***PowerLecture: Vagal Indigestion

The most likely diagnosis for this Holstein heifer is failure of omasal transport, a form of vagal indigestion. This condition leads to fluid accumulation, dorsal gas in the rumen, lack of motility, and inappetence. Omasal transport failure and pyloric outflow failure are hard to distinguish clinically, but the latter usually causes more severe electrolyte imbalances and systemic illness. Abomasal torsion, left displaced abomasum, and grain overload are ruled out due to the monotone ping, fluid-filled rumen, and absence of severe systemic signs.

How well did you know this?
1
Not at all
2
3
4
5
Perfectly
34
Q

You are called to examine and treat a cow which has eaten green chop alfalfa and has bloat. You first pass a stomach tube but little or no gas escapes, and the cow is in respiratory distress due to the tremendous abdominal pressure. You next insert a trocar into the rumen from the left flank and froth exits as shown in the photo. The pH of the froth is 6.2. You should now treat this cow with what drug to further relieve the bloat?

  • Xylazine IV (1 mg/kg)
  • Tetracycline IM or IV (label dose rate)
  • Penicillin orally (10 million units)
  • Oral sodium bicarbonate (1 gm/kg)
  • Poloxalene orally (44 mg/kg)
A

Answer: Poloxalene orally (44 mg/kg)

Explanation
You need to treat with a substance to reduce the surface tension and destabilize the froth. The pH is within normal limits for rumen contents; therefore, oral bicarbonate is not beneficial. In a pinch, one could also use cooking oil to attempt to break down the foam.

To treat the cow’s frothy bloat, administer poloxalene orally at 44 mg/kg. This substance reduces surface tension, helping to destabilize and break down the foam. Since the rumen pH is normal at 6.2, oral bicarbonate is not useful. Cooking oil can be used as an alternative in emergencies.

How well did you know this?
1
Not at all
2
3
4
5
Perfectly
35
Q

A 5-day old dairy calf has decreased appetite and is depressed. On physical exam, you note that she has a temperature of 104F (40 C), HR 120, scleral injection, and one eye has cloudy material in the anterior chamber. Pending lab results, you make a tentative diagnosis of what condition?

  • Chlamydia psittaci ocular infection and sepsis
  • Pinkeye caused by Moraxella bovis
  • Malignant catarrhal fever (MCF)
  • Bovine viral diarrhea (BVD)
  • Failure of passive transfer (FPT) and sepsis, with hypopyon
A

Answer: Failure of passive transfer (FPT) and sepsis, with hypopyon

Explanation
The history of a neonate with these signs and an elevated temperature, scleral injection and hypopyon are indicative of bacterial sepsis, a result of FPT.

How well did you know this?
1
Not at all
2
3
4
5
Perfectly
36
Q

A purebred Holstein breeder wishes to rid his dairy herd of Leptospira interrogans serovar Hardjo (type Hardjo-Bovis), which has been causing reproductive problems in his herd. Which steps are most likely to accomplish this goal?

  • Treat all animals with tetracycline to eliminate carriers and regularly vaccinate all cattle against this serovar
  • Regularly vaccinate all cattle and eradicate rodents, as they are the maintenance hosts of this serovar
  • Regularly vaccinate all cattle and eliminate all dogs from the farm
  • Treat all animals with dihydrostreptomycin
  • Vaccinate all animals in the herd regularly
A

Answer: Treat all animals with tetracycline to eliminate carriers and regularly vaccinate all cattle against this serovar

Explanation
Since Hardjo-Bovis is carried by cattle, he needs to both treat and vaccinate. Vaccination can prevent new carriers, but will not eliminate existing carriers. To be cost effective, he may need to mass treat dry cows and young stock using long acting tetracycline.

Swine (and opossums, skunks and raccoons) are carriers of Pomona, dogs of Canicola, rats of Icterohemorrhagiae, swine, mice and horses of Bratislava, and raccoons, muskrats and squirrels of Grippotyphosa.

How well did you know this?
1
Not at all
2
3
4
5
Perfectly
37
Q

It is the middle of summer, and located in a pasture with a marshy area, several cattle are found dead. The only clinical sign observed by an employee was red urine. You perform a necropsy on one of the affected animals and notice that urine is dark red and all body fluids are icteric and hemorrhagic. There is evidence of hemorrhage in the abomasum and reticulum. Finally, the liver has an anemic infarct near the portal vein. What are these animals infected with?

  • Anaplasma marginale
  • Clostridium hemolyticum (Cl novyi type D)
  • Leptospira pomona
  • Salmonella typhimurium
A

Answer:

Explanation
The correct answer is Clostridium hemolyticum, now more properly called Cl novyi type D, which causes bacillary hemoglobinuria. The organism resides in the liver until anaerobic conditions develop, and then they start replicating and producing toxins. The finding of an anemic infarct on necropsy is classic for red water disease. Liver flukes are often associated with the disease, as they cause the anaerobic tracts needed for the organism to bloom. Marshy areas with snails are part of the liver fluke cycle of transmission.

Anaplasmosis is not a good answer choice because there is no hemoglobinuria associated with the disease. The same goes for Salmonella. Leptospirosis is a good differential given the history; however, necropsy findings are consistent with bacillary hemoglobinuria.

***PowerLecture: Flukes and Bacillary Hemoglobinuria

How well did you know this?
1
Not at all
2
3
4
5
Perfectly
38
Q

You are called to a dairy farm in the western United States because several cows are standing in the corral rather than eating with the others. They have been drooling and champing their mouths. The daughter’s horse is also affected. On physical exam of the cows you find their rectal temperatures to be elevated and that their tongues are ulcerated as shown in the photo, but there are no other lesions you can find on the feet or teats. You call the state and federal veterinarians and say that this looks most like a case of_______

  • Contagious ecthyma (Orf)
  • Foot and mouth disease (FMD)
  • Vesicular stomatitis (VS)
  • Woody tongue (actinobacillosis)
  • Bovine papular stomatitis (BPS)
A

Answer: Vesicular stomatitis (VS)

Explanation
VS occurs as an epidemic about once every 7 to 10 years in the western US. It appears to be spread by insect vectors and then spreads by contact and fomites from cow to cow once clinical disease occurs. Horses can also be affected by this virus.

***PowerLecture: Vesicular Stomatitis

How well did you know this?
1
Not at all
2
3
4
5
Perfectly
39
Q

You visit a dairy that has ongoing problems with mastitis. They ask you for advice on proper milking and maintenance of equipment. Which of the following statements is INCORRECT?

  • Shut off vacuum before removing the claw
  • Replace the liner every 25 milkings
  • Pre dip all teats
  • Post dip all teats
  • Fore strip each quarter
A

Answer: Replace the liner every 25 milkings

Explanation
The incorrect answer is to replace the liner after 25 milkings. This is the best answer because the liner will last for at least 500 milkings. There are three types of liners: synthetic rubber, natural rubber, and silicone. These are designed to last anywhere between 500 and 10,000 milkings. Post-milking teat disinfection is probably the most effective way of reducing the rate of contagious mastitis. Pre-milking dip will help control environmental mastitis.

How well did you know this?
1
Not at all
2
3
4
5
Perfectly
40
Q

A beef ranch in the oak/brush foothills of California has experienced abortions in 40/200 of their new heifers, and the abortions are continuing. These heifers were purchased from Arizona. The fetuses appear to be about 6 to 7 months gestational age. You take several freshly-aborted fetuses (see image) to the diagnostic lab. The lesions they describe include enlarged superficial cervical lymph nodes, spleen, and liver. The thymus is slightly smaller than normal. Based on this information and the history that these animals were not locally raised, you make a tentative diagnosis of epizootic bovine abortion (EBA), also known as foothill abortion. The rancher asks you for information on how best to avoid this scenario again in the future. What is the best preventative strategy?

  • Raise heifers in the foothills and put pregnant heifers in the hills only when 6 months or more pregnant
  • Feed tetracycline to pregnant heifers for 9 months
  • Vaccinate heifers against Chlamydia psittaci
  • Put ear tags on all heifers to keep ticks off before putting the heifers in the foothills
  • Provide hay periodically so that heifers do not eat oak
A

Answer: Raise heifers in the foothills and put pregnant heifers in the hills only when 6 months or more pregnant

Explanation
After decades of research, a novel deltaproteobacterium was identified as the causative agent of EBA in 2005.

The disease is transmitted by a ground-dwelling soft tick, Ornithodoros coriaceus. Abortion occurs more than 3 to 4 months after exposure, so keeping pregnant heifers out of the foothills until they are 6 months pregnant means they calve before fetal lesions occur. Raising heifers in the foothills until breeding does provide premunition immunity. Also, the rancher should keep these heifers that abort, as they will now be immune to EBA. Tick control is very difficult on cattle running in the hills, but the ticks do not inhabit irrigated pastures (they live in the dry ground duff of needles and leaves), so bred heifers can be kept in irrigated pastures for months 1-6 of pregnancy. Feeding tetracycline for 9 months is not practical. As of 2019, there is no commercially available vaccine. However, since 2009 the University of California at Davis has been doing field trials on a live bacteria vaccine that has shown promising efficacy.

How well did you know this?
1
Not at all
2
3
4
5
Perfectly
41
Q

A rancher in the Rocky Mountains has found that many of the beef calves born this season are deformed with cleft palates, arthrogryposis, and crooked spines. The pregnant cows were grazing on pastures that have had a large amount of several species of the plant pictured. What is the problem that caused these congenital anomalies?

  • Locoweed
  • Lupine
  • Tansy ragwort
  • Oleander
  • Larkspur
A

Answer: Lupine

Explanation
Some lupines contain alkaloids called sparteine and anagyrine. Cows eating lupine during early gestation often give birth to calves with cleft palates, crooked legs and distorted/malformed spines. Six of the poisonous species implicated in “crooked calf disease” are silky lupine (L. sericeus), tailcup lupine (L. caudatus), velvet lupine (L. leucophyllus), silvery lupine (L. argenteus), lunara lupine (L. formosus), and yellow lupine (L. sulphureus).

How well did you know this?
1
Not at all
2
3
4
5
Perfectly
42
Q

There are a number of drugs that are used extra label in beef cattle. When working with any food animal, you always have to consider milk and meat withdrawal. What is the meat withdrawal (in days) for xylazine, tolazoline, lidocaine, and meloxicam, respectively?

  • 4, 8, 4, 15
  • 15, 1, 8, 4
  • 7, 18, 21, 15
  • 3, 7, 3, 18
  • 4,1, 18, 21
A

Answer: 4, 8, 4, 15

Explanation
The correct answer is 4, 8, 4, and 15. Remember that xylazine is extremely potent in cattle, taking 10% of the dose that would be given to a horse of equivalent size is sufficient.

Tolazoline is an alpha 2 antagonist. Give the same volume as xylazine. Lidocaine is commonly used during c-sections in cattle. If a producer is doing a c-section as a salvage procedure, it is important to inform that person that the cow cannot go to slaughter for 4 days.

Visit www.farad.org to search for this type of information.

Meloxicam is being used more and more. It is longer acting (~ 4 days) than banamine, but banamine only has 4 days of meat withdrawal. If you plan on going into production medicine, it is important to memorize the numbers for these commonly used drugs so that you can inform the producer.

How well did you know this?
1
Not at all
2
3
4
5
Perfectly
43
Q

A dairy has experienced an increase in abortions from midterm to term, as well as the birth of weak calves with neurologic signs. Based upon histopathologic lesions and herd serology, you have diagnosed Neospora caninum. Which of the following steps to help control the disease is most correct?

  • Keep cats away from cows and cow feed
  • Treat all cows in the herd with long acting tetracycline
  • Depopulate and bring in clean seronegative cows
  • Keep dogs and coyotes away from cows and cow feed Core Answer
  • Keep raccoons away from cows and cow feed
A

Answer: Keep dogs and coyotes away from cows and cow feed

Explanation
The canine is the definitive host and sheds oocysts in the feces, which cause the disease when consumed by cattle. Like cattle, deer may also be infected as intermediate hosts. The dogs are initially infected by eating infected tissues from the cattle, most commonly an aborted fetus.

How well did you know this?
1
Not at all
2
3
4
5
Perfectly
44
Q

Which of the following is a characteristic often associated with bovine pyometra?

  • Diarrhea
  • High fever
  • Polyuria/polydipsia
  • Inappetence
  • Marked abdominal pain
  • Few if any overt signs
A

Answer: Few if any overt signs

Explanation
Unlike the bitch, who shows overt signs, including polydypsia, polyuria, inappetence, and sometimes high fever and diarrhea, the cow shows very few external signs of pyometra.

Cows localize the problem to the uterine lumen. Diagnosis in cows is obtained by transrectal palpation or transrectal ultrasonography.

How well did you know this?
1
Not at all
2
3
4
5
Perfectly
45
Q

You are called to investigate a beef cattle operation where several young adults (1-2 years of age) have developed acute, watery diarrhea (see image). One of the affected cows died and a necropsy reveals worms and petechiae in the abomasum. The wall of the abomasum has a nodular “cobblestone” appearance that is most severe in the fundic region. Which of the following agents is most likely?

  • Trichostrongylus axei
  • Ostertagia ostertagii
  • Haemonchus placei
  • Dictyocaulus viviparous
  • Mecistocirrus digitatus
A

Answer: Ostertagia ostertagii

Explanation
This is a description of type Il ostertagiasis.

The other parasites listed could be differentials with the exception of Dictyocaulus which is a lungworm. The main reason that Ostertagia is the best answer is the description of the abomasum.

The cobblestone appearance which is also sometimes described as having a Moroccon leather appearance is essentially pathognomonic for Ostertagia. This occurs because Ostertagia larva undergo hypobiosis (larval inhibition) and lay dormant in the early fourth larval stage within the glands of the abomasum, forming the nodules.

They resume development and emerge seasonally leading to severe signs.

How well did you know this?
1
Not at all
2
3
4
5
Perfectly
46
Q

You have submitted tissues from a dead dairy cow near a sheep feedlot that died 7 days after exhibiting clinical signs of corneal opacity, high fever, lymphadenopathy, and CNS signs. The lab reports that the brain has lymphocytic perivasculitis. Which of the following disorders is the most likely diagnosis?

  • Polioencephalomalacia
  • Bovine virus diarrhea virus (BVDV)
  • Malignant catarrhal fever (MCF)
  • Histophilus somni infection
  • Grain overload (rumen acidosis)
A

Answer: Malignant catarrhal fever (MCF)

Explanation
Malignant catarrhal fever (MCF) in North America is caused by sheep-associated ovine herpesvirus-2. A very high percentage of sheep and goats in North America are infected with OHV-2 and can transmit the virus to susceptible cattle, bison, deer, and other related species, even if only in close proximity and not in direct contact. The virus causes lymphocytic vasculitis and can be fatal.

BVD does cause fever, but uncommonly results in corneal opacity. The marked lymphadenopathy and lymphocytic vasculitis are not consistent with BVD.

**PowerLecture: Malignant Catarrhal Fever - MCF

How well did you know this?
1
Not at all
2
3
4
5
Perfectly
47
Q

The owner of a new beef ranch on poor volcanic soil asks you to evaluate 2 steers. They are representative of an ongoing herd problem of chronic diarrhea and respiratory disease that is unresponsive to antibiotic treatment. On physical exam, you notice achromotrichia, ill thrift, and a temperature of 103F
(39.4 C) in both steers. You perform a transtracheal wash in one of the steers and it comes back positive for Pasteurella multocida. What is your diagnosis?

  • Selenium deficiency
  • Copper deficiency
  • Pasteurella pneumonia
  • Bovine viral diarrhea
A

Answer: Copper deficiency

Explanation
The correct answer is copper deficiency. The giveaway is achromotrichia or loss of hair color.
Dilution of the coat color is due to dysfunction of tyrosinase which converts L-tyrosine to melanin. In addition, copper deficient animals will have spontaneous fractures, secondary respiratory disease, diarrhea, ill thrift, decreased immunity, anemia, and poor reproduction. Selenium deficiency and BVD are actually pretty good differentials and if achromotrichia was not present either of those two are reasonable choices.

Molybdenum deficiency would not result in achromotrichia. On the other hand, if there had been excess molybdenum, then the Cu:Mo ratio would be off and copper deficiency could be observed.

How well did you know this?
1
Not at all
2
3
4
5
Perfectly
48
Q

Bleeding abomasal ulcers (sometimes called type 2 ulcers) have been diagnosed in a 2-year old dairy cow with melena and anemia. Which of the following treatments is contraindicated?

  • Omeprazole
  • Blood transfusion
  • Flunixin meglumine
  • Ranitidine
A

Answer: Flunixin meglumine

Explanation
Flunixin meglumine is contraindicated in this case. This drug (aka Banamine) is a non-steroidal anti-inflammatory drug (NSAID) and will only promote more ulceration because NSAIDs decrease mucosal blood flow. Blood transfusions are sometimes given if the bleeding is severe. Omeprazole (a hydrogen pump blocker) and ranitidine can be beneficial in calves and raise the abomasal pH, but not useful orally in adults. In adults, IV ranitidine can be used but is very expensive and reserved for high-value animals.

**PowerLecture: Abomasal Ulcers

How well did you know this?
1
Not at all
2
3
4
5
Perfectly
49
Q

You have decided that you need to do an exploratory right flank surgery on a valuable cow because she has colic and abdominal distention, and is passing scant amounts of dark red feces. There are no obvious pings. She is dehydrated (skin turgor is abnormal and eyes sunken) and totally anorectic. The owner reports that she was normal two days ago when he last looked at her closely.

On exploratory surgery under local anesthesia you find a hard 6- to 8- inch long by 4-inch diameter mass in the small intestine, as shown in the photo. It is painful to the cow when it is touched. The bowel proximal to the mass is distended and the bowel distal is empty. How would you treat this?

  • Euthanize the cow as this is an inoperable tumor
  • Surgically reset the intussusception
  • Perform a surgical bypass of the mass but do not remove it
  • Close the cow and give antimicrobial drugs, laxatives and IV fluids
  • Manually reduce the mass and close the cow
A

Answer: Surgically reset the intussusception

Explanation
Hopefully you will be able to recognize that the “mass” visualized in this image is an intussusception.

After removal, an end-to-end anastomosis of the intestine is performed, and the cow is aggressively treated with antimicrobial drugs and IV fluids with a good content of chloride, such as saline, since these animals are usually suffering from hypochloremic hypokalemic metabolic alkalosis.

***PowerLecture: Colic

How well did you know this?
1
Not at all
2
3
4
5
Perfectly
50
Q

This 3-year old Charolais cow in the picture presents with a 2-day history of not eating and looking uncomfortable and hunched up. There are no others in the herd affected. On physical exam, there is decreased rumen motility, a temperature of 105.2F or 40.7C, and a heart rate of 82 bpm. You perform a scooch test on the cow by firmly squeezing down over the withers. The test was positive (she refused to dip her back normally). What is your most likely diagnosis?

  • Left displaced abomasum
  • Bovine spongiform encephalopathy
  • Grain overload
  • Traumatic reticuloperitonitis
A

Answer: ** Traumatic reticuloperitonitis**

Explanation
This question provided a classic description of the presentation for TRP. The clinical signs can be vague and misleading, but the localization of xiphoid pain as shown with the scooch test is a strong indicator of TRP. With a positive scooch test, the cow will be stiff, reluctant to scooch down, and grunts may be heard. Sometimes the stethoscope needs to be placed over the trachea because the grunt may be soft. Additionally, the xiphoid region should be pushed up upon to see whether the cow reacts painfully. There is no mention of pinging that would indicate a displaced abomasum. Xiphoid pain would not be seen as a result of grain overload. Cows with bovine spongiform encephalopathy (mad cow disease) will show neurologic clinical signs such as hypermetric ataxia, hyperexcitability, and hyperesthesia.

**PowerLecture: Traumatic Reticuloperitonitis

How well did you know this?
1
Not at all
2
3
4
5
Perfectly
51
Q

Illness in young calves caused by Eimeria sp. is best treated with which of the following?

  • Levamisole
  • Ivermectin
  • Amprolium
  • Penicillin
  • Tetracycline
A

Answer: Amprolium

Explanation
A number of drugs can be used to treat or prevent coccidiosis. The treatment dosage is generally higher but for fewer days than the preventive dosage. Monensin- and sulfa-containing drugs are also frequently used as treatments.

How well did you know this?
1
Not at all
2
3
4
5
Perfectly
52
Q

Bovine viral diarrhea virus and Border disease virus are small, enveloped RNA viruses that affect cattle and sheep respectively. What is another closely related pestivirus which affects swine and is of worldwide importance as a pathogen?

  • Porcine Parvovirus
  • Pseudorabies
  • Classical swine fever
  • Porcine Cytomegalovirus
  • Swine Pox
A

Answer: Classical swine fever

Explanation
CSF, also known as hog cholera, is very closely related to BVD and Border Disease viruses. It is currently of major importance in Central and South America, Eastern Europe, and Southeast Asia.

***PowerLecture: Foreign Animal Diseases

How well did you know this?
1
Not at all
2
3
4
5
Perfectly
53
Q

You examine a group of sixty 4- to 7-month old dairy calves which appear small and thin. The owner says that in the last few weeks several have developed diarrhea and quit eating, and two have recently died. You note some sunburned white areas on the skin, all are thin, and several are very weak. You do a postmortem examination on the worst and find a small hard liver. The biopsy looks as shown in the photo with fibrosis, bile duct proliferation, and megalocytosis. What is your diagnosis?

  • Black disease
  • Oleander toxicity
  • Mycotoxicosis
  • Pyrrolizidine alkaloid toxicosis
  • Gossypol toxicity
A

Answer: Pyrrolizidine alkaloid toxicosis

Explanation
PAs inhibit mitosis of hepatocytes, so they cannot multiply. As they age, they become large (megalocytes) and when they die they are replaced by fibrous tissue. Bile ducts proliferate as a nonspecific response to hepatic damage. You then look at some of the older alfalfa bales he was feeding to these calves and find loads of Senecio vulgaris, a PA containing plant.

Aflatoxins (a mycotoxin) from Aspergillus and other fungi can cause liver lesions that are similar, but most pathologists think that finding megalocytes is diagnostic for PA toxicity.

***PowerLecture: Hepatic Disorders

The diagnosis is pyrrolizidine alkaloid (PA) toxicosis. PAs inhibit hepatocyte mitosis, leading to large megalocytes, fibrosis, and bile duct proliferation. The calves likely consumed Senecio vulgaris, a PA-containing plant found in their alfalfa feed. While aflatoxins can cause similar liver lesions, megalocytes are considered diagnostic for PA toxicity.

Comprehensive Information on Pyrrolizidine Alkaloidosis in Animals

  1. Definitions:• Pyrrolizidine Alkaloidosis (PA): Chronic toxicosis leading to hepatic failure due to ingestion of toxic plants containing pyrrolizidine alkaloids.
  2. Causative Agents:• Plants: Senecio, Crotalaria, Heliotropium, Amsinckia, Echium, Cynoglossum, Trichodesma.
    • Toxins: Pyrrolizidine alkaloids (e.g., jacobine, retrorsine, seneciphylline, monocrotaline).
  3. Pathophysiology:• Absorption and Distribution: Absorbed via GI tract, distributed to liver via portal vein, also found in fetal tissues, milk, and eggs.
    • Metabolism: Converted to pyrroles in the liver, causing cytotoxic effects, particularly in hepatocytes.
    • Mechanism of Action: Pyrroles create DNA and nucleoprotein adducts, leading to antimitotic effects and megalocytosis, promoting hepatic neoplasms.
  4. Clinical Findings:• Acute: Sudden death from hemorrhagic hepatic necrosis and visceral hemorrhages.
    • Chronic: Loss of condition, anorexia, dullness, diarrhea, icterus, ascites, photosensitization, hepatic encephalopathy (head-pressing, wandering, aggressive behavior), pica.
    • Lesions: Enlarged, hemorrhagic liver in acute cases; atrophied, fibrous, nodular liver in chronic cases.
  5. Diagnosis:• Clinical Signs: Based on exposure history, clinical signs, and biochemical changes (elevated GGT, GDH).
    • Laboratory Tests: Chemical analysis of blood for toxic metabolites, hepatic biopsy showing megalocytosis and fibrosis.
    • Postmortem Examination: Histopathologic changes in liver, pulmonary, or renal tissues, hepatic tissue assays for pyrrolic metabolites.
  6. Treatment:• Supportive Care: Hydration, managing photosensitization.
    • Diet: High-carbohydrate rations to avoid protein-induced clinical signs.
    • Prognosis: Poor for animals showing clinical signs; prevent further intake of toxic plants.
  7. Control and Prevention:• Feed Management: Avoid feeding contaminated hay, silage, or pellets.
    • Biological Control: Use of sheep for grazing control, biological agents like predator moths and flea beetles.
    • Herbicide Application: Annual applications in spring to control toxic plants.
How well did you know this?
1
Not at all
2
3
4
5
Perfectly
54
Q

Which of these treatments is least effective for treating cyanide toxicity?

  • Sodium nitrate
  • Methylene blue
  • Sodium thiosulfate
  • Supplemental oxygen
A

Answer: Supplemental oxygen

Explanation
The answer is supplemental oxygen. Cyanide blocks cellular respiration and blocks oxidative transport. As a result, hemoglobin cannot release oxygen to the tissues and venous blood stays saturated with oxygen and is bright red. Supplemental oxygen is unlikely to change this. The other three options are the usual treatments for cyanide toxicity.

The least effective treatment for cyanide toxicity is supplemental oxygen. Cyanide blocks cellular respiration, preventing hemoglobin from releasing oxygen to tissues, causing venous blood to remain oxygen-saturated and bright red. Supplemental oxygen does not resolve this issue, while other treatments directly counter cyanide’s effects on cellular respiration.

How well did you know this?
1
Not at all
2
3
4
5
Perfectly
55
Q

Which of the following situations is most consistent with a diagnosis of Foot-and-Mouth disease?

  • Cattle are affected with oral and foot lesions and horses are unaffected
  • Horses are affected by foot lesions while cattle are affected with oral and foot lesions
  • Sheep are not affected by the disease
  • Horses and pigs will not be affected by the disease
  • Both cattle and horses are equally affected with oral and foot lesions
A

Answer: Cattle are affected with oral and foot lesions and horses are unaffected

Explanation
The correct answer is cattle are affected with oral and foot lesions and horses are unaffected. Only cloven-hoofed animals are affected by FMD. Therefore, sheep and pigs are susceptible and horses remain unaffected.

If lesions are observed in swine, it is important to differentiate FMD from swine vesicular disease. In cattle, it is important to differentiate FMD from vesicular stomatitis, bovine papular stomatitis, pseudocowpox, and bluetongue. Another point is that sheep will not be affected by vesicular stomatitis.

The most consistent situation for Foot-and-Mouth Disease (FMD) is when cattle show oral and foot lesions while horses remain unaffected. FMD affects only cloven-hoofed animals, including cattle, sheep, and pigs, while horses are not susceptible. Differentiation from vesicular diseases in swine and cattle is essential for accurate diagnosis.

Comprehensive Information on Foot-and-Mouth Disease (FMD) in Animals

  1. Definitions:• Foot-and-Mouth Disease (FMD): A highly contagious viral disease affecting cloven-hoofed animals, causing vesicular lesions.
  2. Causative Agent:• Foot-and-Mouth Disease Virus (FMDV): An Aphthovirus from the family Picornaviridae with 7 serotypes: O, A, C, Asia 1, SAT 1, SAT 2, and SAT 3.
  3. Epidemiology and Transmission:• Hosts: Cattle, pigs, sheep, goats, and over 70 species of wild artiodactyls.
    • Transmission: Direct contact, aerosolized particles, contaminated feed, fomites, and mechanical vectors.
    • Geographical Distribution: Endemic in Africa, the Middle East, and Asia; eradicated from North America and Europe.
  4. Pathophysiology:• Primary Infection Sites: Pharyngeal mucosa, respiratory tract, skin lesions.
    • Viral Spread: Via lymphatic system, causing vesicle formation in the mouth, muzzle, teats, and feet.
  5. Clinical Signs:• Cattle: Fever, vesicles in the mouth and on feet, drooling, lameness, weight loss, decreased milk yield, secondary bacterial infections.
    • Pigs: Fever, lameness, vesicles on the feet and snout, lethargy, loss of appetite.
    • Sheep/Goats: Largely inapparent; lameness, vesicles on feet and mouth.
    • Young Animals: Myocarditis leading to sudden death without vesicular lesions.
  6. Diagnosis:• Laboratory Tests: RT-PCR, virus isolation, serology, antigen ELISA.
    • Samples: Vesicular epithelium or fluid, oropharyngeal fluid, serum.
  7. Treatment:• No Specific Treatment: Supportive care in endemic regions.
  8. Control and Prevention:• Culling: In FMD-free regions to control outbreaks.
    • Vaccination: Inactivated virus vaccines, particularly in endemic areas.
    • Biosecurity Measures: Strict movement controls, disinfection, and rapid reporting of outbreaks.

Medications and Doses:

•	Supportive Care: Fluids, antipyretics, antibiotics for secondary infections as necessary.
•	Vaccination: Regular booster doses based on regional epidemiology and serotype prevalence.

Summary:

•	Causative Agent: Foot-and-Mouth Disease Virus (FMDV).
•	Symptoms: Vesicles, fever, drooling, lameness, myocarditis in young animals.
•	Diagnosis: RT-PCR, virus isolation, serology.
•	Treatment: Supportive care.
•	Control: Vaccination, culling in non-endemic areas, biosecurity measures.
How well did you know this?
1
Not at all
2
3
4
5
Perfectly
56
Q

At what age during gestation is a fetus most likely to become persistently infected with the noncytopathic form of bovine viral diarrhea?

  • Fetus infected before 125 days of gestation
  • Fetus infected at 320 days of gestation
  • Fetus infected between 150-200 days of gestation
  • Fetus infected between 200-270 days of gestation
A

Answer: Fetus infected before 125 days of gestation

Explanation
The correct answer is fetus infected before 125 days of gestation, with most of those occurring before 100 days.

A fetus infected with BVD at any time during gestation can be aborted or be a stillbirth. Most congenital defects associated with BVD occur when a fetus is infected between days 90-150 of gestation. Persistent infection of fetuses is seen when they are infected before 125 days of gestation. This is of importance because they have a potential to affect many other herdmates since they shed virus but may show no clinical signs. About half of the persistently infected calves will die during their first year of life. Clinical signs in BVD carriers vary greatly.

They may range from an asymptomatic animal to one that has diarrhea, fever, pneumonia, and oral ulcerations.

Additionally, you may have infected animals that are just repeat breeders. You may also see abortions, stillbirth, and congenital anomalies, The most co and congenital anomalies. The most common congenital anomaly associated with BVD is cerebellar hypoplasia, but there are many others.

**PowerPage: Bovine Viral Diarrhea
**
PowerLecture: Bovine Viral Diarrhea

A fetus infected with the noncytopathic form of bovine viral diarrhea (BVD) before 125 days of gestation, particularly before 100 days, is most likely to become persistently infected. These fetuses shed the virus and may remain asymptomatic, but can infect herdmates. Persistent infection is significant due to potential transmission within the herd. Common congenital anomalies, like cerebellar hypoplasia, occur when infection happens between 90-150 days.

Comprehensive Information on Bovine Viral Diarrhea (BVD) for NAVLE Preparation

Definition & Etiology:

•	Bovine Viral Diarrhea (BVD): A disease caused by Bovine viral diarrhea virus (BVDV), a Pestivirus in the Flaviviridae family.
•	Transmission: Mainly through direct contact with infected animals or contaminated fomites. Vertical transmission can lead to persistently infected (PI) calves.

Pathophysiology:

•	BVDV can cause immunosuppression, leading to secondary infections.
•	Cerebellar Hypoplasia: In utero infection during mid-gestation can result in cerebellar hypoplasia in calves, characterized by ataxia, tremors, and inability to stand properly.

Clinical Signs:

•	Diarrhea, fever, nasal discharge, immunosuppression, and mucosal disease in PI animals.
•	Neurological signs in calves with cerebellar hypoplasia include ataxia, intention tremors, and hypermetric gait.

Diagnosis:

•	PCR and virus isolation are used to detect BVDV.
•	Serology to identify antibodies in the dam or calf.

Prevention:

•	Vaccination: Prophylactic vaccination of breeding animals is essential to prevent vertical transmission and cerebellar hypoplasia in calves.

Treatment:

•	No specific antiviral treatment; focus on supportive care and prevention of secondary infections.
How well did you know this?
1
Not at all
2
3
4
5
Perfectly
57
Q

Maple syrup urine disease is a genetic disorder associated with spongiform changes in the brain and caused by a deficiency of this enzyme in Hereford and polled Shorthorn calves. It usually shows up at 2 to 3 days of age. Which of the following enzymes is the correct one?

  • Pyruvate kinase
  • Branched-chain ketoacid decarboxylase
  • Carnitine acyl translocase
  • Acetyl-CoA carboxylase
A

Answer: Branched-chain ketoacid decarboxylase

Explanation
The correct answer is branched-chain ketoacid decarboxylase, which will cause accumulation of 2-keto-3-methylvaleric, 2-ketoisocaproic, and 2-isovaleric acids along with their precursors isoleucine, leucine, and valine.

These are excreted in the urine and result in a burnt maple syrup smell (hence the name of the disease). This hereditary disease causes an encephalopathy. Acetyl-CoA carboxylase turns acetyl-CoA into malonyl CoA in fatty acid synthesis. Carnitine acyl translocase helps bring fatty acids across the inner mitochondrial membrane for degradation. Pyruvate kinase turns phosphoenol-pyruvate into pyruvate in the glycolysis pathway.

Maple syrup urine disease is caused by a deficiency in the enzyme branched-chain ketoacid decarboxylase, leading to the accumulation of 2-keto-3-methylvaleric, 2-ketoisocaproic, and 2-isovaleric acids, along with their precursors isoleucine, leucine, and valine. This results in the characteristic burnt maple syrup smell in the urine and causes encephalopathy in affected Hereford and polled Shorthorn calves.

How well did you know this?
1
Not at all
2
3
4
5
Perfectly
58
Q

You arrive at a beef ranch in a mountain meadow at 5000 feet elevation in the western US because the owner has called with the complaint of sudden death losses in adult cows, and another one staggering. You examine the sick animal and find fever of 105F (40.6 C), elevated respiratory rate, HR of 100/min, icteric mucous membranes, and weakness. You collect a urine sample and it appears dark red (see image). You also perform a post-mortem on one of the recently dead cows and find a hepatic infarct, icteric tissues and hemoglobinuria. The owner asks for a diagnosis, treatment, and prevention.

  • Blackleg; penicillin, vaccination
  • Lightning strike; provide shelters with lightning rods
  • Acute Fasciola hepatica infestation; penicillin, fluke control
  • Bacillary hemoglobinuria; penicillin, vaccination, and fluke control
  • Blue-green algae toxicity; no treatment, prevent via clean water source
A

Answer: Bacillary hemoglobinuria; penicillin, vaccination, and fluke control

Explanation
The diagnosis is bacillary hemoglobinuria caused by Clostridium novyi type D (formerly called Cl. hemolyticum).

Penicillin can be given to any animal showing early signs. Vaccination with Cl. novyi type D vaccine is effective, and prevention should also include fluke control. Migrating flukes cause hepatic damage and anaerobic areas which results in germination of the Cl. novyi spores in the liver, an infarct, and production of the hemotoxin.

Flukes are best controlled by controlling snails in wet areas, fencing off wet areas, and using routine fluke treatments.

The diagnosis is bacillary hemoglobinuria, caused by Clostridium novyi type D. Treatment includes administering penicillin to affected animals. Prevention involves vaccination with Cl. novyi type D and fluke control. Fluke migration creates anaerobic liver areas, allowing Cl. novyi spore germination, leading to hepatic infarcts and hemotoxin production. Controlling snails, fencing wet areas, and routine fluke treatments are essential preventive measures.

Bacillary Hemoglobinuria in Animals - Comprehensive Veterinary Information

Definitions and Terminology:

•	Bacillary Hemoglobinuria (Red Water Disease): Acute toxemia caused by Clostridium haemolyticum.

Causative Agents:

•	Pathogen: Clostridium haemolyticum (also known as Clostridium novyi Type D).

Physiopathology:

•	Transmission: Ingestion of spores, which remain latent in the liver and germinate under anaerobic conditions caused by liver damage (e.g., liver fluke infestation).
•	Pathogenesis: Spores germinate in the liver, producing beta toxin (phospholipase C) that causes intravascular hemolysis, leading to hemolytic anemia and hemoglobinuria.

Clinical Findings:

•	Symptoms: Sudden onset of severe depression, fever, abdominal pain, dyspnea, severe diarrhea, and hemoglobinuria. Anemia, jaundice, and edema of the sternum may also be present.
•	Lesions: Liver infarcts, bloody fluid in body cavities, hemorrhagic intestines, dark friable kidneys, and purplish red urine in the bladder.

Diagnosis:

•	Clinical Signs: Port wine-colored urine, severe depression.
•	Postmortem: Liver infarcts with diagnostic confirmation via PCR, IFAT, or other laboratory tests.

Treatment:

•	Antimicrobials: High-dose penicillin or tetracyclines.
•	Supportive Care: Blood transfusions and fluid therapy.

Control:

•	Vaccination: With C. haemolyticum bacterin, given once or twice yearly.
•	Management: Control of liver fluke infestation and improved pasture drainage.
How well did you know this?
1
Not at all
2
3
4
5
Perfectly
59
Q

Which of the following is NOT a benefit of providing a DCAD (dietary cation-anion difference) diet to cattle?

  • Cows absorb calcium more readily
  • There is a lower incidence of milk fever
  • Parathyroid hormone function is enhanced
  • Cows remain relatively more alkalotic
A

Answer: *Cows remain relatively more alkalotic

Explanation
DCAD is used to help prevent milk fever. Cows eating a DCAD diet are actually more acidotic which enhances parathyroid hormone function along with a better ability to utilize dietary calcium. The easy measure is to check urine pH (it should be acid) on cows to be sure they are ingesting the diet.

To review, DCAD is dietary cation-anion difference. A DCAD diet is enhanced with more anionic salts containing the strong ions chloride and sulfur, and has decreased amounts of strong cations such as sodium and potassium.

A benefit of providing a DCAD diet to cattle is not that they remain more alkalotic; in fact, DCAD diets make cows slightly more acidotic. This enhances parathyroid hormone function and improves calcium utilization, helping to prevent milk fever. Urine pH should be monitored to confirm proper ingestion of the DCAD diet.

How well did you know this?
1
Not at all
2
3
4
5
Perfectly
60
Q

You have diagnosed nitrate poisoning in a group of yearling cattle which were grazing Sudan grass (see photo). Which of the following is the most effective treatment?

  • Methylene blue
  • Digitalis
  • Vitamin B12
  • Sodium thiosulfate
  • Calcium gluconate
A

Answer: Methylene blue

Explanation
Nitrate toxicosis causes methemoglobinemia, in which the blood appears chocolate brown. As a result, the oxygen carrying capacity and delivery is greatly reduced and the animal may die. Methylene blue is usually administered IV as a 1% to 4% solution, with a total dose of 4 to 15 mg/kg body weight to cattle, for treating nitrate toxicosis.

Nitrates are found in high levels in many plants and some water sources. The nitrate is converted in the rumen to nitrite and then absorbed into the blood, which causes oxidation of the heme iron to the +3 ferric state (called methemoglobin). Methylene blue restores the iron in hemoglobin to its normal (reduced) oxygen-carrying state. This is achieved by providing an artificial electron acceptor for NADPH methemoglobin reductase (RBCs usually don’t have one; the presence of methylene blue allows the enzyme to function at normal levels).

The NADPH is generated via the hexose monophosphate shunt.

The most effective treatment for nitrate poisoning in cattle is intravenous administration of methylene blue. Nitrate toxicosis leads to methemoglobinemia, reducing oxygen-carrying capacity. Methylene blue, given as a 1-4% solution at 4-15 mg/kg, restores hemoglobin’s oxygen-carrying ability by acting as an artificial electron acceptor for NADPH methemoglobin reductase.

How well did you know this?
1
Not at all
2
3
4
5
Perfectly
61
Q

You have submitted tissues from a dead dairy cow near a sheep feedlot that died 7 days after exhibiting clinical signs of corneal opacity, high fever, lymphadenopathy, and CNS signs. The lab reports that the brain has lymphocytic perivasculitis. Which of the following disorders is the most likely diagnosis?

  • Bovine virus diarrhea virus (BVDV)
  • Polioencephalomalacia
  • Histophilus somni infection
  • Grain overload (rumen acidosis)
  • Malignant cataral fever (MCF)
A

Answer: Malignant cataral fever (MCF)

Explanation
Malignant catarrhal fever (MCF) in North America is caused by sheep-associated ovine herpesvirus-2. A very high percentage of sheep and goats in North America are infected with OHV-2 and can transmit the virus to susceptible cattle, bison, deer, and other related species, even if only in close proximity and not in direct contact. The virus causes lymphocytic vasculitis and can be fatal.

BVD does cause fever, but uncommonly results in corneal opacity. The marked lymphadenopathy and lymphocytic vasculitis are not consistent with BVD.

***PowerLecture: Malignant Catarrhal Fever - MCF

The most likely diagnosis is malignant catarrhal fever (MCF), caused by sheep-associated ovine herpesvirus-2 (OHV-2). This virus, common in sheep and goats, can transmit to cattle and cause fatal lymphocytic vasculitis. Key signs include corneal opacity, high fever, lymphadenopathy, and CNS symptoms. BVD is less likely as it rarely causes corneal opacity or marked lymphadenopathy.

Comprehensive Information on Malignant Catarrhal Fever (MCF) in Animals

  1. Definitions:• Malignant Catarrhal Fever (MCF): A severe, often fatal, lymphoproliferative disease in ruminants caused by ruminant gammaherpesviruses.
  2. Causative Agents:• Viruses: Ovine herpesvirus-2 (sheep), alcelaphine herpesvirus-1 (wildebeest), caprine herpesvirus-2 (goats).
  3. Epidemiology:• Hosts: Cattle, water buffalo, bison, deer.
    • Transmission: Direct contact or aerosolized virus from carriers like sheep and wildebeest.
    • Seasonality: Linked to lambing/calving periods; virus shedding is higher in young animals.
  4. Clinical Signs:• Acute Cases: Fever, depression, oral/nasal erosions, lymphadenopathy, corneal opacity.
    • Peracute Cases: Sudden death, hemorrhagic diarrhea, hematuria.
    • Chronic Cases: Alopecia, weight loss.
  5. Pathophysiology:• Infection Mechanism: Viral replication in lymphoid tissues leading to widespread vasculitis.
    • Lesions: Necrosis and inflammation of mucosal epithelium, generalized lymphoid proliferation, vasculitis.
  6. Diagnosis:• Clinical Signs: Fever, erosions, ocular changes, lymphadenopathy.
    • Laboratory Tests: PCR, immunohistochemistry, serology.
    • Differential Diagnoses: BVD, IBR, East Coast fever, rabies.
  7. Treatment:• Supportive Care: No specific antiviral treatment; manage stress and secondary infections.
    • Prognosis: Grave; high mortality rate.
  8. Control and Prevention:• Biosecurity: Separate carriers (sheep, wildebeest) from susceptible species.
    • No Vaccine: Early weaning and isolation of lambs.

Medications and Doses:

•	Supportive Care: Antipyretics, fluids, antibiotics for secondary bacterial infections.

Summary:

•	Causative Agents: Ruminant gammaherpesviruses.
•	Symptoms: Fever, nasal/oral erosions, lymphadenopathy, corneal opacity.
•	Clinical Changes: Vasculitis, necrosis, lymphoid proliferation.
•	Diagnosis: Clinical signs, PCR, serology.
•	Treatment: Supportive care.
•	Control: Biosecurity, no vaccine available.
How well did you know this?
1
Not at all
2
3
4
5
Perfectly
62
Q

You are presented with a 10-day old Holstein dairy calf weighing 40 Kg that is cold (Temperature is 97 degrees F, 36.1 degrees C) and nonresponsive. Her eyes appear sunken as in the photo. In addition to placing the calf on a warming pad, what is the best treatment?

  • 1 liter of subcutaneous fluids containing glucose, sodium, bicarbonate, chloride and lesser amounts of potassium
  • 4 liters fluid containing equal amounts of sodium and chloride
  • 4 liters of intravenous fluids containing glucose, sodium, bicarbonate, chloride and lesser amounts of potassium
  • Gentamicin intravenously at the label dosage
A

Answer: 4 liters of intravenous fluids containing glucose, sodium, bicarbonate, chloride and lesser amounts of potassium

Explanation
The calf is in metabolic acidosis (base deficit of about 15) and needs sodium containing fluids IV that contain bicarbonate or other base. 4 liters intravenously is a more appropriate fluid volume for a dehydrated 10-day old calf, calculated as 10% Of the 40 kg body weight. The sunken eye, as seen in the photo, is an important indicator of marked dehydration.

To treat metabolic acidosis caused by loss of sodium containing fluids you need to give more sodium than chloride, so sodium bicarbonate is the fluid of choice. Neonates such as this one tend to develop severe metabolic acidosis as they dehydrate due to absorption of acids from the gut and due to loss of renal compensatory mechanisms due to poor renal perfusion. The calf is also likely to be hypoglycemic, so adding glucose is essential. As you rehydrate and bring up blood glucose, the serum potassium will be driven back into cells and needs to be replaced with some potassium in the fluids.

***PowerLecture: Calf Diarrhea Part 2

The best treatment for this 10-day-old dehydrated Holstein calf is 4 liters of intravenous fluids containing glucose, sodium, bicarbonate, chloride, and potassium. The calf is in metabolic acidosis with a base deficit of about 15, requiring sodium bicarbonate to correct the acidosis. Adding glucose helps address likely hypoglycemia, and potassium replenishment is essential as rehydration drives potassium back into cells.

How well did you know this?
1
Not at all
2
3
4
5
Perfectly
63
Q

A 3-year old Holstein dairy cow, 3 weeks post partum, is presented with the complaint that she has dropped in milk production over the last several days. You perform a physical exam and the only abnormality you note is a large amount of ketone bodies in the milk and urine. You decide to administer glucose IV and insulin SQ. What else should this cow be administered?

  • IM tetracycline
  • Oral propylene glycol
  • IM parathyroid hormone
  • SQ bovine somatotropin
  • IV calcium gluconate
A

Answer: Oral propylene glycol

Explanation
The oral propylene glycol will serve as a precursor for glucose production, and will help with the demands of lactation that are causing increased fat mobilization and ketosis. A fourth treatment that is sometimes used is corticosteroids, as they decrease milk production and increase gluconeogenesis.

***PowerPage: Ketosis

In addition to IV glucose and SQ insulin, the cow should receive oral propylene glycol. This acts as a glucose precursor, helping to meet the energy demands of lactation, reducing fat mobilization, and combating ketosis. Corticosteroids can also be used to decrease milk production and increase gluconeogenesis.

How well did you know this?
1
Not at all
2
3
4
5
Perfectly
64
Q

Mucosal disease, or alternatively, chronic Bovine Virus Diarrhea (BVD) occurs in cattle when:

  • A 5-month old calf is infected with CPE biotype of BVD virus and then superinfected with a nonCPE biotype of BVD virus.
    A 5-month old calf is persistently infected with a strain of BVD virus which coats platelets; platelets are then removed by the RE system resulting in a bleeding diathesis.
    BVD type 2 infects a 5-month old calf.
    A 5-month old calf which was persistently infected as a fetus with a non cytopathic (nonCPE) biotype of BVD virus is superinfected with a cytopathic (CPE) biotype of BVD due to rearranging of the parent non-CPE viral RNA.
    A 5-month old calf which was persistently infected with the nonCPE biotype of BVD virus as a fetus forms antigen-antibody complexes which cause a fatal immune-mediated disorder.
A

Answer: A 5-month old calf which was persistently infected as a fetus with a non cytopathic (nonCPE) biotype of BVD virus is superinfected with a cytopathic (CPE) biotype of BVD due to rearranging of the parent non-CPE viral RNA.

Explanation
This is the correct answer because it describes the current understanding of the pathogenesis of both mucosal disease and chronic BVD.

A 5-month old calf is infected with CPE biotype of BVD virus and then superinfected with a nonCE biotype of BVD virus is incorrect because it does not say that there was persistent fetal infection; also, the order of the superinfection is the reverse of what causes mucosal disease or chronic BVD.

A 5-month old calf which was persistently infected with the nonCPE biotype of BVD virus as a fetus forms antigen-antibody complexes which cause a fatal immune-mediated disorder is incorrect because this is not the mechanism by which mucosal disease or chronic BVD occur.

All other answer choices are incorrect because they each describe one of the acute clinical syndromes from primary infection, rather than either mucosal disease or chronic BVD.

***PowerPage: Bovine Viral Diarrhea

Mucosal disease or chronic Bovine Virus Diarrhea (BVD) occurs when a 5-month-old calf, persistently infected as a fetus with the non-cytopathic (nonCPE) biotype of BVD virus, is superinfected with the cytopathic (CPE) biotype due to rearrangement of the parent non-CPE viral RNA. This describes the current pathogenesis of both mucosal disease and chronic BVD.

How well did you know this?
1
Not at all
2
3
4
5
Perfectly
65
Q

Several beef cows present with a history of decreased appetite and excessive salivation. On physical exam, their tongues are firm on palpation, nodular, and painful (see image). You diagnose actinobacillosis. What is your recommendation to the owner?

  • Begin therapy with an aminoglycosides
  • Isolate affected animals and submit one of them for necropsy
  • Isolate animals at once
  • Change feed and treat
  • Sell affected animals for meat
A

Answer: Change feed and treat.

The correct answer is to change feed and begin treatment of the animals affected with woody tongue, as the response is often good. Sodium iodide and antibiotics are effective. Given the presentation and clinical signs, these animals have probably begun to ingest very rough and stemmed (scabrous) feed items that have injured their mouths. Upon injury, the normal inhabitant Actinobacillus lignieresii invades the soft tissues and causes the characteristic woody tongue granulomatous inflammation.

These animals don’t have rabies, and there is no need to cull them. Change feed before additional animals are affected. Do not use aminoglycosides as they have an extremely long withdrawal period.

How well did you know this?
1
Not at all
2
3
4
5
Perfectly
66
Q

During a visit to a dairy, several cows are noted to have 1-3 cm cysts on their backs with small holes at the center resembling breathing pores created by larvae. If these are breathing pores, what organism are the cows infected with?

  • Simulium
  • Anopheles
  • Culicoides
  • Sarcophaga
  • Hypoderma
A

Answer: Hypoderma

Explanation
The correct answer is Hypoderma bovis or H lineatum. Hypoderma is also known as the cattle grub, heel fly, or warble fly. The life cycle begins with the female attaching up to 500 eggs to the hairs around the hocks and lower portions of the cow. First-stage larvae will hatch in just a few days and burrow into the skin. At this point, they begin their tour de cow and migrate towards the epidural fat in the spinal canal(H bovis) or via the esophagus (H lineatum). After several months in this beautiful location, they become L2s and migrate to the subcutaneous tissues of the back where they will molt once more, and become L3s. Once they are L3s, the swelling on the cows’ backs can be felt. After 5-11 weeks, the L3s mature and burst through the skin, dropping on the ground where they will become adults in another 1-3 months. Sarcophaga spp are known as the flesh flies. An adult female will deposit her eggs in wounds and ulcers; the larvae then feed off the wounds.

Eventually, they mature into L3s and fall off to pupate on the ground.

Simulium flies are also known as buffalo gnats or black flies. Adult females are the key problem with these flies because they suck blood! They prefer the legs, abdomen, head, and ears. Additionally, they will only eat during daytime. Female flies are an annoyance to cows and cause decreased productivity.

Anopheles are just mosquitoes; however, they are the most important vectors of human malaria and spread
West Nile Virus.

Culicoides are also known as no-see-ums, biting midges, and punkies. They are a great annoyance to cows and have the potential to transmit bluetongue and Onchocerca. In horses, they are thought to be the cause of sweet itch, a Type I hypersensitivity to their saliva.

How well did you know this?
1
Not at all
2
3
4
5
Perfectly
67
Q

You are in charge of a slaughter house and are asked about why so many of the culled dairy cattle in a recent shipment have liver abscesses (see photo) and what can be done about it. The abscesses result in a significant financial loss for the owner because the liver is condemned, and he is concerned. What is the cause?

  • Liver fluke migration
  • Rumen acidosis
  • Black disease
  • Foot abscesses showering bacteria to the liver
  • Traumatic reticuloperitonitis with hepatic involvement
A

Answer: Rumen Acidosis

Explanation
Dairy cattle which are fed a high concentrate diet should be gradually introduced to it. Even so, an additional dietary buffer may be needed. The herd veterinarian should check rumen pH on about 5 high-producing cows by needle stick of the rumen 2 to 3 hours after they are fed. If some fall below pH 5.5, additional sodium bicarbonate should be added to the ration.

The low rumen pH causes rumenitis allowing bacteria to translocate through the mucosa and be filtered from the hepatic portal circulation by the liver, resulting in one or more hepatic abscesses.

The key to picking rumenitis is that there are many culled dairy cattle coming in with this problem of hepatic abscess. If it were feet or hardware it would only occasionally lead to a liver abscess.

How well did you know this?
1
Not at all
2
3
4
5
Perfectly
68
Q

You examine a valuable beef bull which has been in a remote pasture unobserved for several months. The owner has noted that he has a swollen lower jaw. You note the hard, relatively non-painful swelling shown in the photo and diagnose:

  • Tooth root abscess
  • Osteosarcoma
  • Lymphoma
  • Actinobacillosis
  • Actinomycosis
A

Answer: Actinomycosis

Explanation
Also known as lumpy jaw, this condition results from the entry of the normal rumen inhabitant Actinomyces bovis into the bony mandible (usual site) or maxilla through a break in the mucous membranes or teeth. It may be arrested with therapy using sodium iodide, antimicrobials or even isoniazid off-label, but the bony swelling seldom changes much, even if arrested.

  • Actinomycosis: A chronic bacterial infection caused by gram-positive, anaerobic bacteria Actinomyces.
  • Causative Agents:
    • Actinomyces bovis: Lumpy jaw in cattle.
    • A. suis (A. denticolens): Pyogranulomatous mastitis in swine.
    • A. hordeovulneris: Localized abscesses and systemic infections in dogs.
    • A. viscosus: Cutaneous abscesses, pneumonia, pyothorax in dogs.
  • Cattle: Slow-growing, firm mass attached to the mandible, facial distortion, loose teeth, dyspnea, ulceration, and fistulous tracts.
  • Swine: Pyogranulomatous mastitis, subcutaneous granulomatous lesions, lung and organ abscesses.
  • Dogs: Localized abscesses, pyogranulomatous pleuritis, peritonitis, and visceral abscesses.
  • Methods: Clinical signs, culture (anaerobic conditions), Gram stain, radiology, cytology, and biopsy.
  • Ruminants: Intravenous sodium iodide (70 mg/kg of a 10%-20% solution, repeated at 7-10 day intervals), with antimicrobials like penicillin, florfenicol, or oxytetracycline.
  • Swine: Rarely successful; surgical removal may be required.
  • Dogs: Surgical debridement, penicillin, cephalosporins, sulfonamides, and repeated chest drainage for pyothorax.
  • Cattle: Avoid coarse, stemmy feeds or feeds with plant awns to prevent mucosal damage.

For more detailed information, visit the Merck Veterinary Manual - Actinomycosis in Cattle, Swine, and Other Animals.

How well did you know this?
1
Not at all
2
3
4
5
Perfectly
69
Q

Two 3 year-old bulls have died and another 5 are very ill with severe dyspnea and tachypnea. The affected animals are grunting, open-mouth breathing and frothing at the mouth. You are unable to catch one to check HR and rectal temperature or to listen to the chest.

The group of 20 animals was moved to a much better grass pasture 10 days ago. You perform a necropsy on one of the dead bulls and find that the lungs are edematous, emphysematous and full of bullae (see image). There is very little in the airways except froth, and no pleuritis is present. Based on the history and pathologic findings, you diagnose what disorder?

-Lungworm infestation
- Acute bovine pulmonary edema and emphysema
- Histophilus somni pneumonia
- Bovine respiratory syncitial virus
- Mannheimia hemolytica bronchopneumonia

A

Answer: Acute bovine pulmonary edema and emphysema

Explanation
This condition is an ARDS (acute respiratory distress syndrome) of cattle that usually occurs after animals are moved from poor quality feed to an improved lush pasture. The mechanism involves ingestion of L-tryptophan in lush feed being converted to 3-methylindole in the rumen. 3-MI is pneumotoxic and causes pulmonary edema, alveolar epithelial hyperplasia, hyaline membranes, and emphysema.

Lesions of the other choices are different from the ones described here. The two bacterial pneumonias usually occur in younger, often confined and stressed, animals. BRSV tends to affect young calves peracutely, and would not cause signs as described in these 3 year-old animals. Lungworm infestation could have similar clinical signs but is unlikely to be this acute and severe in 3 year-old animals, and parasitic pneumonia has a great deal of coughing associated with it. The pathology would be different with more consolidation and bronchial exudates.

Pulmonary Emphysema, Edema, and Interstitial Pneumonia in Cattle

Definitions and Causative Agents:

•	Acute Bovine Pulmonary Emphysema and Edema (ABPEE): Also known as “fog fever”, caused by metabolites of L-tryptophan in lush pasture.
•	Pathogenesis: L-tryptophan is converted by ruminal microorganisms to 3-methylindole, which is pneumotoxic.

Clinical Changes and Symptoms:

•	Symptoms: Mild to severe dyspnea, tachypnea, hyperpnea, mouth breathing, drooling, expiratory grunt, minimal coughing.
•	Lesions: Pulmonary edema, emphysema, alveolar epithelial hyperplasia, hyaline membrane formation.

Diagnosis:

•	Assessment: History of pasture change, clinical signs, necropsy findings.
•	Histopathology: Congestion, alveolar edema, epithelial hyperplasia.

Treatment:

•	No effective treatment: Manage affected animals with caution to prevent further stress and possible sudden death.

Prevention:

•	Pasture Management: Gradual introduction to lush pastures, use of hay before grazing, strip grazing, grazing with less susceptible animals first.
•	Medications: Monensin or lasalocid to inhibit bacteria converting L-tryptophan to 3-methylindole.

For detailed information, visit the Merck Veterinary Manual - Pulmonary Emphysema, Edema, and Interstitial Pneumonia in Cattle.

How well did you know this?
1
Not at all
2
3
4
5
Perfectly
70
Q

A dairyman brings his prize 6-month old bull calf to your clinic because he is not eating all his feed today and has diarrhea. He has just purchased this registered bull calf from a neighboring state, and it was shipped a week ago during some very cold weather. You examine the bull calf and find T=104 F (40 C), HR=90, and RR=35, with poor rumen motility. There are several ulcers on the dental pad, and the diarrhea is foul-smelling and contains flecks of blood. The calf is hemorrhaging from small scleral blood vessels (see photo). You quickly take a blood sample and have your assistant run it to the state diagnostic lab, which calls you less than an hour later to say that the thrombocyte count is 5000 (normal is 100,000-800,000/ul). You tell the owner that the calf appears to have bovine virus diarrhea infection with thrombocytopenia. What should your first line of therapy be if you are to save this calf?

  • Blood transfusion with fresh whole blood
  • High doses of IV ceftiofur
  • Immunostimulant drugs IV
  • Antiviral drug therapy
  • IV fluids to treat dehydration
A

Answer: Blood transfusion with fresh whole blood

Explanation
The thrombocytopenia is caused by BVD virus adhering to thrombocytes which are then removed by the reticuloendothelial (RE) system. Once the number gets this low, fatal hemorrhage is possible at any moment.
Although not usual, this form of BVD has been described. The other forms of therapy may be needed as well. If the calf survives, you will also need to determine whether or not he remains persistently infected with BVD… if he does, he should not be retained.

Comprehensive Information on Bovine Viral Diarrhea (BVD) for NAVLE Preparation

Definition & Etiology:

•	Bovine Viral Diarrhea (BVD): A disease caused by Bovine viral diarrhea virus (BVDV), a Pestivirus in the Flaviviridae family.
•	Transmission: Mainly through direct contact with infected animals or contaminated fomites. Vertical transmission can lead to persistently infected (PI) calves.

Pathophysiology:

•	BVDV can cause immunosuppression, leading to secondary infections.
•	Cerebellar Hypoplasia: In utero infection during mid-gestation can result in cerebellar hypoplasia in calves, characterized by ataxia, tremors, and inability to stand properly.

Clinical Signs:

•	Diarrhea, fever, nasal discharge, immunosuppression, and mucosal disease in PI animals.
•	Neurological signs in calves with cerebellar hypoplasia include ataxia, intention tremors, and hypermetric gait.

Diagnosis:

•	PCR and virus isolation are used to detect BVDV.
•	Serology to identify antibodies in the dam or calf.

Prevention:

•	Vaccination: Prophylactic vaccination of breeding animals is essential to prevent vertical transmission and cerebellar hypoplasia in calves.

Treatment:

•	No specific antiviral treatment; focus on supportive care and prevention of secondary infections.
How well did you know this?
1
Not at all
2
3
4
5
Perfectly
71
Q

You perform an exam on a calf that was born with a twin. The calf has an enlarged clitoris and an abnormally small ano-genital distance. What is your diagnosis?

  • Lupine toxicosis
  • Negative energy balance
  • Ponderosa pine needle toxicosi
  • Freemartin
A

Answer: Freemartin

Explanation
The correct answer is that this calf is a freemartin. This is evident by the clinical signs provided. A freemartin results from in utero exposure of the female to Mullerian-inhibiting hormone (also called anti-mullerian hormone or AMH) being secreted by the male. She is exposed via anastomosis between the two fetuses’ chorioallantoic vessels. Lupine causes fetal arthrogryposis if ingested, and ponderosa pine needles induce late abortion if consumed.

Freemartin Heifer and Blood Types

Freemartinism in Cattle

Freemartinism is a condition of sterility observed in female cattle (heifers) that are born as twins to male calves. Approximately 92% of such heifers are sterile. This condition arises due to the vascular connections between the placentas of the twin fetuses, which allows the transfer of anti-Müllerian hormone from the male to the female fetus. This hormone inhibits the development of the female reproductive tract, leading to varying degrees of masculinization of the female genitalia.

Characteristics of Freemartin Heifers

•	Genitalia: Affected heifers typically have a short vagina that ends blindly, without communication with the uterus. The cervix is often absent, and the ovaries usually remain underdeveloped and small.
•	Reproductive Organs: The tubular genital organs may range from cord-like structures to near-normal uterine horns, depending on the extent of masculinization.

Diagnostic Criteria

•	Vaginal Length: In calves 1-4 weeks old, the normal vaginal length is 13-15 cm, whereas in freemartin heifers, it is about 5-6 cm. This can be measured by gently inserting a lubricated probe with a blunt end into the vagina.
•	Cytogenetic Examination: Freemartins typically exhibit a mix of XX and XY chromosome patterns due to the interchange of cells in the placental circulation between the fetuses.
•	Blood Types: Freemartinism can also be indicated by the presence of two different blood types in a single animal, reflecting the cellular exchange between the twin fetuses.
How well did you know this?
1
Not at all
2
3
4
5
Perfectly
72
Q

A beef cow presents for palpation at approximately 30 days of gestation. Which positive sign of pregnancy will be present?

  • Chorioallantoic membrane slip
  • Palpable uterine artery fremitus
  • Placentomes are palpable
  • Fetus is palpable
A

Answer: Chorioallantoic membrane slip

Explanation
The correct answer is chorioallantoic membrane slip. Placentomes will be palpable starting between 75-90 days of gestation. The fetus itself will be palpable beginning at approximately 60 days, but may be out of reach between months 4-7 of gestation. Uterine artery fremitus will be evident on the ipsilateral pregnant horn at about 120 days of gestation. From 7 months on, fremitus is palpable bilaterally. Fremitus does not necessarily indicate a viable pregnancy.

At approximately 30 days of gestation, the positive sign of pregnancy in a beef cow is the chorioallantoic membrane slip. Placentomes are palpable at 75-90 days, the fetus at 60 days, and uterine artery fremitus appears around 120 days, but fremitus doesn’t always indicate a viable pregnancy.

How well did you know this?
1
Not at all
2
3
4
5
Perfectly
73
Q

A necropsy of an aborted bovine fetus shows enlarged lymph nodes and spleen, destructive lesions to the thymus, and evidence of chronic granulomatous infection. What is the most likely cause of this abortion?

  • Infectious bovine rhinotracheitis
  • Brucellosis
  • Tritrichomonas foetus
  • Epizotic bovine abortion
A

Answer: Epizotic bovine abortion

Explanation
The correct answer is epizootic bovine abortion (EBA), also called foothill abortion. EBA is an important disease for beef producers in the foothill and mountainous regions of California, Northern Nevada and Southern Oregon. The lesions described in the question are consistent with this diagnosis. The etiologic agent of EBA is a bacteria, Pajaroellobacter abortibovis and the vector is a tick, Ornithodoros coriaceus.

Tritrichomona foetus is involved with early embryonic death.

Brucellosis abortions are rare since it has been almost completely eradicated from the U.S. Lesions include autolysis, placentitis, and bronchopneumonia.

Infectious Bovine Rhinotracheitis (IBR) causes rapid fetal death; therefore, there is very little time for fetal response. Lesions you are likely to see include autolysis and focal necrosis of the organs.

The most likely cause of this abortion is epizootic bovine abortion (EBA), also known as foothill abortion. EBA occurs in foothill and mountainous regions of California, Northern Nevada, and Southern Oregon. The lesions described, such as enlarged lymph nodes, spleen, and chronic granulomatous infection, are consistent with EBA, caused by Pajaroellobacter abortibovis and transmitted by the tick Ornithodoros coriaceus.

How well did you know this?
1
Not at all
2
3
4
5
Perfectly
74
Q

A purebred Holstein breeder wishes to rid his dairy herd of Leptospira interrogans serovar Hardjo (type Hardjo-Bovis), which has been causing reproductive problems in his herd. Which steps are most likely to accomplish this goal?

  • Treat all animals with dihydrostreptomycin
  • Regularly vaccinate all cattle and eradicate rodents, as they are the maintenance hosts of this serovar
  • Regularly vaccinate all cattle and eliminate all dogs from the farm
  • Vaccinate all animals in the herd regularly
  • Treat all animals with tetracycline to eliminate carriers and regularly vaccinate all cattle against this serovar
A

Answer: Treat all animals with tetracycline to eliminate carriers and regularly vaccinate all cattle against this serovar

Explanation
Since Hardjo-Bovis is carried by cattle, he needs to both treat and vaccinate. Vaccination can prevent new carriers, but will not eliminate existing carriers. To be cost effective, he may need to mass treat dry cows and young stock using long acting tetracycline.

Swine (and opossums, skunks and raccoons) are carriers of Pomona, dogs of Canicola, rats of Icterohemorrhagiae, swine, mice and horses of Bratislava, and raccoons, muskrats and squirrels of Grippotyphosa.

Leptospirosis in Animals - Comprehensive Veterinary Information

Definitions and Terminology:

•	Leptospirosis: A zoonotic disease caused by pathogenic serovars of Leptospira.

Causative Agents:

•	Pathogens: Leptospira interrogans, Leptospira borgpetersenii.

Physiopathology:

•	Transmission: Direct or indirect contact with urine from infected animals, contaminated water, or soil.
•	Pathogenesis: Bacteria penetrate mucous membranes or broken skin, spread through the bloodstream, and localize in the kidneys, liver, lungs, and other organs.

Clinical Findings:

•	Symptoms: Fever, anorexia, vomiting, dehydration, icterus, renal failure, hemolytic anemia, uveitis, abortion, stillbirths.
•	Lesions: Interstitial nephritis, hepatitis, pulmonary hemorrhage.

Diagnosis:

•	Tests: Microscopic agglutination test (MAT), PCR, culture, ELISA.
•	Sample: Blood, urine, tissue samples.

Treatment:

•	Antibiotics: Doxycycline, amoxicillin, penicillin.
•	Supportive Care: Fluids, blood transfusions.

Prevention:

•	Vaccination: Polyvalent inactivated vaccines.
•	Hygiene: Avoid exposure to potentially contaminated environments.
How well did you know this?
1
Not at all
2
3
4
5
Perfectly
75
Q

A purebred Holstein breeder wishes to rid his dairy herd of Leptospira interrogans serovar Hardjo (type Hardjo-Bovis), which has been causing reproductive problems in his herd. Which steps are most likely to accomplish this goal?

  • Treat all animals with dihydrostreptomycin
  • Regularly vaccinate all cattle and eradicate rodents, as they are the maintenance hosts of this serovar
  • Regularly vaccinate all cattle and eliminate all dogs from the farm
  • Vaccinate all animals in the herd regularly
  • Treat all animals with tetracycline to eliminate carriers and regularly vaccinate all cattle against this serovar
A

Answer: Treat all animals with tetracycline to eliminate carriers and regularly vaccinate all cattle against this serovar

Explanation
Since Hardjo-Bovis is carried by cattle, he needs to both treat and vaccinate. Vaccination can prevent new carriers, but will not eliminate existing carriers. To be cost effective, he may need to mass treat dry cows and young stock using long acting tetracycline.

Swine (and opossums, skunks and raccoons) are carriers of Pomona, dogs of Canicola, rats of Icterohemorrhagiae, swine, mice and horses of Bratislava, and raccoons, muskrats and squirrels of Grippotyphosa.

Leptospirosis in Animals - Comprehensive Veterinary Information

Definitions and Terminology:

•	Leptospirosis: A zoonotic disease caused by pathogenic serovars of Leptospira.

Causative Agents:

•	Pathogens: Leptospira interrogans, Leptospira borgpetersenii.

Physiopathology:

•	Transmission: Direct or indirect contact with urine from infected animals, contaminated water, or soil.
•	Pathogenesis: Bacteria penetrate mucous membranes or broken skin, spread through the bloodstream, and localize in the kidneys, liver, lungs, and other organs.

Clinical Findings:

•	Symptoms: Fever, anorexia, vomiting, dehydration, icterus, renal failure, hemolytic anemia, uveitis, abortion, stillbirths.
•	Lesions: Interstitial nephritis, hepatitis, pulmonary hemorrhage.

Diagnosis:

•	Tests: Microscopic agglutination test (MAT), PCR, culture, ELISA.
•	Sample: Blood, urine, tissue samples.

Treatment:

•	Antibiotics: Doxycycline, amoxicillin, penicillin.
•	Supportive Care: Fluids, blood transfusions.

Prevention:

•	Vaccination: Polyvalent inactivated vaccines.
•	Hygiene: Avoid exposure to potentially contaminated environments.
How well did you know this?
1
Not at all
2
3
4
5
Perfectly
76
Q

What is the effect of PGF2-alpha during anestrus in cattle?

  • Induces an LH surge
  • There is no effect
  • Causes the lysis of the corpus luteum
  • Induces FSH release
A

Answer: There is no effect

Explanation
The correct answer is that there is no effect when the cow is in anestrus. That is the problem with using PGF2-alpha to synchronize estrus cycles in cows. A mature corpus luteum is needed so that PGF2-alpha can lyse it; the cow goes back into estrus in about 2-5 days.

How well did you know this?
1
Not at all
2
3
4
5
Perfectly
77
Q

You are called out early in the morning to examine a recumbent 7-year-old Friesian cow which calved at some stage during the night and is now unable to rise with the abnormality seen in the picture. What should you do?

  • Administer epidural anesthetic, remove attached fetal membranes, clean and lubricate the uterus and replace in normal position, then administer oxytocin and calcium gluconate
  • Administer epidural anesthetic and excise the extruded tissue
  • Administer epidural anesthetic, empty the bladder if necessary, lubricate the vagina and replace in normal position, place a Buhner suture around the vestibule at the point at which the initial eversion of the vaginal wall occurred
  • Administer epidural anesthetic, reduce and retain the rectum with a purse-string suture that is loose enough to allow 2 fingers into the opening, administer fecal softeners
A

Answer: Administer epidural anesthetic, remove attached fetal membranes, clean and lubricate the uterus and replace in normal position, then administer oxytocin and calcium gluconate

Explanation
This is a case of uterine prolapse. Because images may sometimes be difficult to differentiate uterine versus other prolapses, the clinical history can be useful in helping differentiate which type of prolapse is likely. Uterine prolapse occurs immediately or within hours of parturition. Vaginal prolapse occurs primarily in late pregnancy.

Rectal prolapse is typically associated with straining to defecate.

The uterus is more easily replaced if the cow is standing. If she remains recumbent it may help to put her sternal and pull both hind legs into a position behind her.

The oxytocin will help involute the uterus and expel fluids, and the calcium gluconate will help treat or prevent hypocalcemia. Do not give oxytocin before replacing the uterus because this makes it turgid and much more difficult to replace. Although not mentioned in any of the answer choices, administration of an antibiotic (i.e. oxytetracycline) may be useful to reduce metritis.

If the cow remains down and unable to rise, she should be lifted using a flotation tank. The sooner this is done, the more likely she is to recover.

How well did you know this?
1
Not at all
2
3
4
5
Perfectly
78
Q

You are performing a routine pregnancy examination via rectal palpation on an Angus beef cow and note that placentomes of variable size, some as big as a half-dollar coin are present and also note bilateral uterine artery fremitus. At what point in gestation is the fetus?

  • 60 days
  • 5 months
  • 35 days
  • At least 6 months
  • 4 months
A

Answer: At least 6 months

Explanation
The correct answer is at least 6 months. Placentomes (cotyledons on the placenta / caruncules on the uterus) will be palpable starting between 75-90 days of gestation and at 150 days can reach the size of a half-dollar coin. Three or more placentomes must be palpated to rule out that you are not palpating an ovary.

Uterine artery fremitus which is hypertrophy of the middle uterine artery with fluid turbulence that gives a “buzz” feeling to the artery will be evident on the ipsilateral pregnant horn at about 120 days of gestation. From 6-7 months on, the fremitus can be felt bilaterally. Fremitus is not a positive sign of pregnancy, but can be useful in staging.

How well did you know this?
1
Not at all
2
3
4
5
Perfectly
79
Q

You are asked to evaluate the breeding soundness of a 1-year old, 675 lb Brown Swiss bull. Vital parameters and your general physical examination are unremarkable. You collect semen via electro-ejaculation and note an acceptable semen quantity and quality. Scrotal circumference is 33 cm. You note 2 small penile warts near the tip of the penis that have a broad head and narrow stalk. There are no warts detected elsewhere on the bull and no other abnormalities associated with the penis. What should you recommend to the owner?

  • The bull should not be bred because he has not yet reached puberty. Warts should be re-evaluated when the bull reaches puberty.
  • You suggest vaccination with an autogenous vaccine
  • The bull should not be bred until at least 6 months after regression of all warts
  • Warts are not contagious but the bull has a very small scrotal circumference and is not considered a good breeding animal
  • You suggest surgical removal and sexual rest for 2-3 weeks
A

Answer: You suggest surgical removal and sexual rest for 2-3 weeks

Explanation
Penile warts in young bulls are quite common and are caused by bovine papilloma virus-1 (BPV-1) which can also cause warts on the nose or teats. Warts are very contagious and primarily spread by contamination of facility equipment but also spread directly from bull to bull. BPV-1 infection is not associated with other health problems. The treatment of choice for penile warts is surgical removal. Surgical damage to the warts frequently stimulates an immune response against BPV-1. Healing is usually complete within 2-3 weeks.

Autogenous vaccines have been used to treat penile warts but this is rarely practiced now due to cost and risk of severe reactions and relatively little benefit compared to surgical removal. It is not necessary to wait 6 months before breeding. Bulls typically reach puberty at 9-11 months of age depending on breed. Scrotal circumference of a 1 year-old bull also varies by breed but >30 cm is typically recommended for breeding at this age.

The best recommendation is surgical removal of the penile warts, followed by 2-3 weeks of sexual rest. Penile warts in young bulls, caused by bovine papilloma virus-1 (BPV-1), are common and highly contagious. Surgical removal typically triggers an immune response, leading to complete healing within 2-3 weeks. Autogenous vaccines are rarely used due to cost and risk, and it’s unnecessary to delay breeding for six months. The scrotal circumference of 33 cm is acceptable for a 1-year-old bull, indicating readiness for breeding.

How well did you know this?
1
Not at all
2
3
4
5
Perfectly
80
Q

Which of the following is not a cause of teat lesions?

  • Anaplasmosis
  • Pseudocowpox
  • Herpes mammillitis
  • Vesicular stomatitis
  • Bluetongue virus
A

Answer: Anaplasmosis

Explanation
The correct answer is Anaplasmosis, which is a cause of extravascular hemolysis. Pseudocowpox is a parapox virus and results in proliferative teat lesions. Vesicular stomatitis is a reportable disease caused by a rhabdovirus.

It is reportable because it is similar in presentation to foot and mouth disease. Clinical signs include ulceration of the teats and mouth. Bluetongue and herpes mammillitis also result in ulcerative lesions.

How well did you know this?
1
Not at all
2
3
4
5
Perfectly
81
Q

You are presented with a pair of aborted bovine twins and their fetal membranes, at about 8 months gestational age. The tissue is fresh and there are no gross or histologic lesions in any internal organs including the brain; likewise, the placentae are free of lesions. A test for serum immunoglobulins in both fetuses is negative. While a definitive diagnosis is probably not likely from this description, which of the following diagnoses is most compatible?

  • Abortion due to lethal white syndrome
  • Abortion due to Neosporum caninum
  • Abortion due to Brucella abortus
  • Abortion due to Listeria monocytogenes
  • Abortion due to Infectious Bovine Rhinotracheitis (IBR) virus
  • “Physiological abortion” attributable to twinning
A

Answer: “Physiological abortion” attributable to twinning

Explanation
In cattle, twinning is a significant risk factor for abortion.

Brucella abortus can cause abortion, but there are typically signs of severe inflammation, including placentitis.

IBR virus can quickly kill a bovine fetus, but as with Herpes viruses in many hosts, there are signs of severe inflammation throughout the fetus.

Listeria monocytogenes can cause abortion, but the fetus is usually autolyzed to a significant extent, and there are often brain micro-abscesses present.

Neosporum caninum can cause epidemic or enzootic patterns of abortion, but typically abortions occur earlier than 8 months and there is autolysis of tissues. Frequently, there are granulomas in the brain.

Lethal white svndrome is a cause of death in foals. not abortion in cattle.

The most compatible diagnosis for the twin bovine abortion with no gross or histological lesions and negative serum immunoglobulins is “physiological abortion” due to twinning. Twinning significantly increases the risk of abortion in cattle. Other causes like Brucella abortus, IBR virus, Listeria monocytogenes, and Neospora caninum typically present with inflammation, autolysis, or organ lesions, none of which were observed here. Lethal white syndrome affects foals, not cattle.

82
Q

What is the significance of low milk progesterone levels on day 0 of estrous and high on day 21 and 24 in a cow?

  • Likely not pregnant
  • Persistent corpus luteum
  • There is no correlation between milk progesterone levels and pregnancy
  • Likely pregnant
A

Answer: Likely pregnant

Explanation
The correct answer is that the cow is likely to be pregnant. If the progesterone levels were low on days 0, 21, and 24 of estrous, it is almost certain she is not pregnant. High progesterone levels throughout may indicate that she has been pregnant or that she has a persistent corpus luteum. In this scenario, with low levels initially at day O but high at day 21 and 24, this cow should be pregnant.

The cow is likely pregnant based on low milk progesterone levels on day 0 of estrous and high levels on days 21 and 24. Low progesterone throughout would indicate non-pregnancy, while high levels suggest pregnancy or a persistent corpus luteum. In this case, the hormonal pattern indicates pregnancy.

83
Q

A pregnant cow has a uterine torsion. Which of these therapeutic interventions should you attempt if the torsion is clockwise?

  • Push forward on the vagina while rolling the cow counterclockwise
  • Push forward on the vagina while rolling the cow clockwise
  • Administer oxytocin
  • Perform uterine surgery to relieve the torsion
A

Answer: Push forward on the vagina while rolling the cow clockwise

Explanation
The correct answer is push forward on the vagina while rolling the cow clockwise, which may seem counter-intuitive. By stabilizing the uterus and rotating the cow around it, you can attempt to relieve the torsion.

To address a clockwise uterine torsion, the recommended intervention is to push forward on the vagina while rolling the cow clockwise. This helps stabilize the uterus while rotating the cow, potentially relieving the torsion by allowing the uterus to untwist as the cow is rolled.

84
Q

What is the major mode of transmission of contagious mastitis causing organisms in cows?

  • Transmission by the milker’s hands and milking equipment
  • Transmission by injury to the teat
  • Transmission as a result of calves suckling
  • Transmission by contact with other infected individuals
A

Answer: Transmission by the milker’s hands and milking equipment

Explanation
The correct answer is transmission by milker’s hands and milking equipment. This is the main factor in the spread of mastitis, and you must work closely with your client in training personnel to apply proper milking techniques.

***PowerPage: Mastitis

85
Q

Pick the ideal suture pattern for a uterus after a Caesarian section in a cow.

  • Near-far-far-near
  • Utrecht pattern
  • Simple interrupted
  • Horizontal mattress
A

Answer: Utrecht pattern

Explanation
The correct answer is the Utrecht pattern. This pattern achieves a nice mucosal and serosal seal when done correctly. Additionally, the exposure of suture to the serosa is minimized with this type of pattern, so adhesions of uterus to surrounding structures is minimized

86
Q

The preferred site for PRACTICAL artificial insemination in the mare and cow is_________

  • Intrauterine
  • Intravestibular, i.e. just inside the vulva
  • Artificial insemination cannot be successfully performed in cows
  • Intracervical, into the external os of the cervix
  • Intra-abdominal, i.e. via the oviducts
  • Intravaginal
A

Answer: Intrauterine

Explanation
Artificial insemination (Al) with frozen or fresh semen is nearly always performed intrauterine in both the mare and the cow. Conception rates are optimal, compared with intravaginal or intracervical routes of Al. With the intracervical and intravaginal routes, sperm will have farther to go to get to the site of fertilization (ampulla of oviduct), and won’t have the advantage of being propelled towards the ampullae that sperm deposited intrauterine would have.

Intravaginal routes result in very low conception rates unless sperm numbers are greatly increased, in which case one of the main advantages of Al (ability to dilute semen from a superior male and use it on several females) is lost.

Given the same quality and quantity of semen/sperm, conception rates following intracervical routes of Al are higher than Al by intravaginal routes, but still much lower than when intrauterine routes are used.
Intravestibular (i.e., just inside the vulva) will result in very low conception rates. Many sperm will be killed by urine, and the surviving sperm will have much farther to go to get to the site of fertilization.

Al by the intra-abdominal route, i.e., via the oviducts, is highly impractical at this time, as the oviducts are not easily catheterized.

The preferred site for practical artificial insemination (AI) in both mares and cows is intrauterine. This method ensures optimal conception rates compared to intravaginal or intracervical routes, as it places sperm closer to the site of fertilization (ampulla of the oviduct). Intrauterine AI benefits from sperm being propelled towards the ampullae, increasing the chances of successful fertilization. Other methods, such as intravaginal or intravestibular, result in lower conception rates due to the distance sperm must travel and potential sperm damage.

87
Q

About 1 hour after a difficult labor where the calf had to be pulled out with force, the cow goes down. She is pale and her heart rate is 100/min. What happened?

  • Obturator paralysis
  • Uterine tear
  • Pelvic fracture
  • Hypocalcemia
A

Answer: Uterine tear

Explanation
The correct answer is uterine tear. With a traumatic fracture or damage to nerves, the cow would have gone down right away rather than an hour later. Hypocalcemia is possible but less likely in this case where the calf had to be forcibly extracted which can result in a uterine tear, and would be unlikely to make her pale and tachycardic.

88
Q

If trying to abort a cow between 5-8 months of gestation, what will work best?

  • PG2-alpha and dexamethasone
  • PGF2-alpha
  • GnRH
  • Dexamethasone
A

Answer: PG2-alpha and dexamethasone

Explanation
The correct answer is PGF2-alpha and dexamethasone. This is because at this point (5 to 8 months), both the CL and the fetus are involved in maintaining the pregnancy.

89
Q

You visit a dairy that has ongoing problems with mastitis. They ask you for advice on proper milking and maintenance of equipment. Which of the following statements is INCORRECT?

  • Fore strip each quarter
  • Replace the liner every 25 milkings
  • Shut off vacuum before removing the claw
  • Post dip all teats
  • Pre dip all teats
A

Answer: Replace the liner every 25 milkings

Explanation
The incorrect answer is to replace the liner after 25 milkings. This is the best answer because the liner will last for at least 500 milkings. There are three types of liners: synthetic rubber, natural rubber, and silicone. These are designed to last anywhere between 500 and 10,000 milkings. Post-milking teat disinfection is probably the most effective way of reducing the rate of contagious mastitis. Pre-milking dip will help control environmental mastitis.

The incorrect statement is “Replace the liner every 25 milkings.” Liners last for at least 500 milkings, with some types lasting up to 10,000 milkings, depending on the material. Post-milking teat disinfection is effective for reducing contagious mastitis, while pre-milking dips help control environmental mastitis.

90
Q

A necropsy of an aborted bovine fetus shows enlarged lymph nodes and spleen, destructive lesions to the thymus, and evidence of chronic granulomatous infection. What is the most likely cause of this abortion?

  • Brucellosis
  • Tritrichomonas foetus
  • Epizootic bovine abortion
  • Infectious bovine rhinotracheitis
A

Answer: Epizootic bovine abortion

Explanation
The correct answer is epizootic bovine abortion (EBA), also called foothill abortion. EBA is an important disease for beef producers in the foothill and mountainous regions of California, Northern Nevada and Southern Oregon. The lesions described in the question are consistent with this diagnosis. The etiologic agent of EBA is a bacteria, Pajaroellobacter abortibovis and the vector is a tick, Ornithodoros coriaceus.
Tritrichomona foetus is involved with early embryonic death.

Brucellosis abortions are rare since it has been almost completely eradicated from the U.S. Lesions include autolysis, placentitis, and bronchopneumonia.

Infectious Bovine Rhinotracheitis (IBR) causes rapid fetal death; therefore, there is very little time for fetal response. Lesions you are likely to see include autolysis and focal necrosis of the organs.

91
Q

A young bull presents for a breeding soundness exam. What is the minimum recommended percentage normal morphology of sperm?

  • 50% Normal
  • 60% Normal
  • 70% Normal
  • 90% Normal
  • 80% Normal
A

Answer: 70% Normal

Explanation
The correct answer is 70% normal. To be a satisfactory potential breeder, a bull must meet certain requirements including a normal physical exam, adequate scrotal circumference, sperm motility of at least 30% motile, and sperm morphology of at least 70% normal.

92
Q

What is the current recommendation for cows diagnosed with Mycoplasma bovis mastitis?

  • Systemic antimicrobials
  • Frequent milking (every 1-2 hours)
  • Anti-inflammatories
  • Cull
A

Answer: Cull

Explanation
The correct answer is cull. Unfortunately, there is very poor response to treatment and since this is a very contagious organism, it is best to cull. Other types of mastitis, such as Strep ag or coliform mastitis, may respond well to frequent milking, anti-inflammatories, and antibiotics.

93
Q

You recently helped deliver twins on a dairy farm where one of the twins was male and one was female. Which of the following is the common outcome of mixed sex twins in cattle?

  • Both animals are intersex, infertile, with non-functioning sexual organs; they are genetically distinct with the heifer XX and the bull XY
  • The heifer is intersex, infertile with masculinized behavior, non-functioning ovaries, and is genetically chimeric (XX/XY); the bull is phenotypically normal
  • The bull is intersex, infertile with feminized behavior and non-functioning testes, and is genetically male (XY); the heifer is phenotypically normal
  • The heifer is intersex, infertile with masculinized behavior, non-functioning ovaries, and is genetically female (XX); the bull is phenotypically normal
  • The bull is intersex, infertile with feminized behavior and non-functioning testes, and is genetically chimeric (XX/XY); the heifer is phenotypically normal
A

Answer: The heifer is intersex, infertile with masculinized behavior, non-functioning ovaries, and is genetically chimeric (XX/XY); the bull is phenotypically normal

Explanation
Freemartinism is the normal result of mixed-sex twins in cattle. This occurs due to shared circulation of chorionic blood vessels in utero. This allows antimullerian duct hormone and testosterone from the male fetus to inhibit development of the female tract resulting in a short vagina that ends blindly without communication to the uterus. The heifer also frequently shows masculinized behavior. Cytogenetic examination of freemartins reveals both XX and XY chromosome patterns.

Interestingly, although some bulls born as mixed-sex twins show the presence of XX cells, they have normal conformation and external genitalia and are fertile.

94
Q

Each year, hide damage due to this organism results in tremendous economic loss in cattle.

  • Hypoderma bovis
  • Ornithodoros coriaceus
  • Sarcoptes scabiei
  • Chochliomyia hominivorax
A

Answer: Hypoderma bovis

Explanation
The correct answer is Hypoderma bovis. H bovis is also known as the cattle grub, heel fly, or warble fly. The life cycle starts with the female attaching up to 500 eggs to the hairs around the hocks and lower portions of the cow. First-stage larvae will hatch in just a few days and burrow into the skin. At this point, they migrate towards the epidural fat in the spinal canal (H bovis) or via the esophagus (H lineatum). After several months, they become L2s and migrate to the subcutaneous tissues of the back where they will reach puberty, molt once more, and become L3s. Once they are L3s, swellings on the cows’ backs can be felt. After 5-11 weeks, the L3s mature and burst through the skin, dropping on the ground where they will become adults in another 1-3 months.

Cochliomyia hominivorax (aka screwworm) lays its eggs on skin wounds. The developing maggots feed on the flesh and can lead to multiple infestations from additional screwworms and other flies.

Ornithodoros coriaceus (aka Pajahuello tick) is a soft tick that feeds on many different mammals and birds. This tick has become somewhat famous because it appears to transmit epizootic bovine abortion.

Sarcoptes scabiei (aka scabies mange mite) causes mechanical and chemical irritation and intense pruritus, leading to loss of milk production and damage of hides due to scratching. Although this is a possible answer, the economic impact of Sarcoptes scabies is not as worrisome as with Hypoderma bovis.

95
Q

The 5-month old calf in the picture has diarrhea, fever and oral ulcers mainly on the dental pad. The calf entered a feedlot about 2 weeks before. Based on these clinical signs and history, the most likely diagnosis is ___________

  • Bovine viral diarrhea (BVD)
  • Infectious bovine rhinotracheitis (IBR)
  • Vesicular stomatitis
  • Heavy metal toxicity
  • Contagious ecthyma
A

Answer: Bovine viral diarrhea (BVD)

Explanation
BVD typically causes clinical illness in young susceptible animals exposed and stressed through transport or crowding and can result in oral ulcerations.

**PowerPage: Bovine Viral Diarrhea
**
PowerLecture: Bovine Viral Diarrhea

96
Q

A valuable bull that has been heavily grain fed for show for over 3 months is found bleeding from the nose. You examine the animal and note a fever of 104 F(40 C), rapid respiratory rate, and HR of 90 beats/min. You tell the owner that this bull may have developed rumen acidosis in the past, a resulting liver abscess and has now developed_______

  • Peracute Mannheimia hemolytica bronchopneumonia
  • Bleeding diathesis due to clotting defect
  • Bloody nose form trauma
  • Bovine respiratory syncytial virus pneumonia
  • Vena caval thrombosis and metastatic pneumonia
A

Answer: Vena caval thrombosis and metastatic pneumonia

Explanation
By chance a liver abscess in a few animals that develop rumen acidosis may form near the posterior vena cava and cause infected thrombi to break off and move down pulmonary arteries into the lung until they lodge and form an abscess. Pulmonary embolic abscesses then create tracts from affected arteries to an airway, which results in pulmonary bleeding. This condition is extremely difficult to treat successfully and animals usually die within months of a massive bleed.

97
Q

A 3-year old Holstein dairy cow is one month fresh and the milker has noted that her production has dropped in the last week. On physical exam you find normal TPR, a gaunt abdomen, scant but normal-appearing feces, and no abnormalities on rectal exam. She has a prominent ping with variable changing pitch on the left side of the abdomen between the 10th and 13th ribs when simultaneous auscultation and percussion are used. The image shows a model of her abdominal organs viewed from the left side. What is the correct diagnosis?

  • Free abdominal gas from uterine rupture
  • Abomasal volvulus
  • Gas in the rumen
  • Left displaced abomasum
  • Right displaced abomasum
A

Answer: Left displaced abomasum

Explanation
The gas-filled abomasum has moved from its normal position near the ventral midline to the left of the rumen, where it is trapped (see image). As gas bubbles in and out, it sounds like the end of a toilet flush. When percussed and auscultated, the changing gas pressures yield a variable high-pitched ping.

***PowerLecture: Left Displaced Abomasum

98
Q

On a pregnancy check of a cow, crepitus can be felt upon palpation of the uterus, and there are no positive signs of pregnancy. What is the most likely diagnosis?

  • Ruptured uterus
  • Macerated fetus
  • Mummified fetus
  • Pyometra
A

Answer: Macerated fetus

Explanation
The correct answer is macerated fetus. The key to making this diagnosis is in feeling the crepitus, which is caused by bacterial degradation of the fetus. A mummified fetus is a sterile fetus in which all fluids are resorbed, leaving a firm tarry mass. With pyometra, the uterus will be distended with pus. Pyometras are usually seen post-partum. If you see one post-coital, consider Tritrichomonas foetus.

99
Q

An outbreak of psoroptic mange is identified in a herd of cattle. What is the treatment of choice?

  • Pyrethrin
  • Ivermectin
  • Levamisole
  • Albendazole
A

Answer: Ivermectin

Explanation
The correct answer is ivermectin. Albendazole and levamisole are used to treat worms. Ivermectin is more effective than pyrethrin.

100
Q

A 3-week-old Holstein calf presents for diarrhea, fever, and anorexia. On physical exam, the calf appears to have a very stiff neck and hyperesthesia. Blood work shows an increased WBC and neutrophilia. A CSF tap is performed, which shows a turbid fluid, protein of 120 mg/dl, and 420 WBCs/uL which are mainly neutrophils. What is the most likely diagnosis?

  • Meningitis
  • Listeriosis
  • Polioencephalomalacia
  • Salt poisoning
A

Answer: Meningitis

Explanation
The correct answer is meningitis. This conclusion can be made by examining the CSF. An elevated CSF protein and CSF neutrophilia indicates a bacterial infection. Listeriosis tends to be associated with monocytosis of the CSF. The clinical signs and blood work are non-specific, and only the CSF results allow good differentiation between the other answer choices. With polioencephalomalacia the CSF changes in WBC count and protein will be much lower (5-50 WBCs/uL and protein slightly greater than 50mg/dl). Salt poisoning will not change the CSF protein and WBC count by much either.

101
Q

What gram positive organism is an obligate pathogen of the mammary gland of cows?

  • Corynebacterium bovis
  • Mycoplasma
  • Staphylococcus aureus
  • Streptococcus agalactiae
A

Answer: Streptococcus agalactiae

Explanation
The correct answer is Streptococcus agalactiae. This organism usually causes subclinical mastitis, but sometimes you may see high somatic cell counts and clinical signs with this organism. Perform a CAMP reaction to diagnose this organism. That is when you plate S. aureus and S. agalactiae and you see them act together to lyse red blood cells. This creates a clearing on your culture plate.

102
Q

At what day in gestation can pregnancy first be confirmed via ultrasound in cattle?

  • 15 days
  • 28 days
  • 60 days
  • 42 days
A

Answer: 28 days

Explanation
The correct answer is 28 days. Veterinarians can determine the fetal sex at approximately 58-90 days of gestation.

103
Q

A 4-year old dairy cow in her third lactation presents after calving normally about 95 days ago. No one has observed her in estrus since calving. On physical exam, you find a non-pregnant heavy, fluid-filled uterus, and an apparent corpus luteum on one ovary. Assuming treatment is successful, which of the following is the most significant therapeutic effect of a single intra-muscular injection of PGF2-alpha in this patient?

  • PGF2-alpha will stimulate the cow’s appetite
  • PGF2-alpha has no effect on a cow’s estrus cycle
  • PGF2-alpha will directly suppress uterine inflammation
  • PGF2-alpha will cause the maturation and ovulation of multiple follicles (i.e., superovulation)
  • PGF2-alpha will decrease uterine contractility
  • PGF2-alpha will lyse her corpus luteum, increase uterine motility, evacuate her uterus, and bring her into estrus
A

Answer: PGF2-alpha will lyse her corpus luteum, increase uterine motility, evacuate her uterus, and bring her into estrus

Explanation
PGF2-alpha has many effects, but chief among them is its luteolytic action. Lysis of the corpus luteum will immediately reduce circulating progesterone levels, allow the final development of an ovarian follicle, and bring the cow into estrus, which is accompanied by increased uterine motility, and in this case, drainage.

PGF2-alpha will not likely stimulate the cow’s appetite and would not do so by a direct therapeutic effect.

PGF2-alpha will not decrease uterine contractility. PGF2-alpha will probably increase uterine contractility, both directly (utero-tonic effects) and indirectly, through the rise of estrogen that follows PGF2-alpha-induced luteolysis.

PGF2-alpha is an inflammatory mediator. It will NOT directly suppress uterine inflammation. This cow needs an effective immune/inflammatory response to “clean up” her uterus. Anti-inflammatory therapy is not indicated.

PGF2-alpha will NOT cause the maturation and ovulation of multiple follicles (i.e., superovulation). That is achieved by an altogether different class of hormones, the gonadotropins.

104
Q

Mycobacterium avium ssp paratuberculosis is most commonly recognized in cattle at which age?

  • 6 weeks to 6 months of age
  • Greater than 5 years of age
  • 6 months to 2 years of age
  • Less than 6 weeks of age
  • 2-5 years of age
A

Answer: ** 2-5 years of age**

Explanation
The correct answer is 2-5 years of age, although younger and older animals can develop Johne’s disease.
Mycobacterium avium ssp paratuberculosis is the causative agent of Johne’s disease in cattle. It causes wasting, diarrhea, and decreased production. The disease is economically devastating. There is no recommended treatment for the disease. Animals testing positive should be culled.

***PowerLecture: Paratuberculosis - Johne’s Disease

105
Q

You visit a dairy that has ongoing problems with mastitis. They ask you for advice on proper milking and maintenance of equipment. Which of the following statements is INCORRECT?

  • Fore strip each quarter
  • Post dip all teats
  • Shut off vacuum before removing the claw
  • Pre dip all teats
  • Replace the liner every 25 milkings
A

Answer: Repleace the liner every 25 milkings

Explanation
The incorrect answer is to replace the liner after 25 milkings. This is the best answer because the liner will last for at least 500 milkings. There are three types of liners: synthetic rubber, natural rubber, and silicone. These are designed to last anywhere between 500 and 10,000 milkings. Post-milking teat disinfection is probably the most effective way of reducing the rate of contagious mastitis. Pre-milking dip will help control environmental mastitis.

106
Q

What cardiac abnormality is commonly associated with hypocalcemia and milk fever in the cow?

  • Atrial fibrillation
  • Ventricular premature contractions
  • Tachycardia
  • Sinoatrial node block
A

Answer: Tachycardia

Explanation
The correct answer is tachycardia. Calcium administration may cause all the other abnormalities if you give too much too fast. You may also see a sinus node arrest. Calcium administration will cause the heart to beat more slowly and more strongly.

107
Q

Which of the following is the best treatment for thelazia in cattle as shown in this image?

  • Piperazine
  • Remove manually
  • Thiabendazole
  • Fluconazole
A

Answer: Remove manually

Explanation
The correct answer is to remove them manually. After removal it is recommended to treat with 10% levamisole drops. This is actually a photo of thelazia in a canine, but the appearance is very similar in cattle. The eye-worm can also be treated with ivermectin.

108
Q

Which of the following medications has the shortest meat withdrawal time in beef cattle?

  • Ceftiofur sodium
  • Penicillin G
  • Sulfadimethoxine
  • Oxytetracycline
  • Chloramphenicol
A

Answer: Ceftiofur sodium

Explanation
The correct answer is ceftiofur sodium. Ceftiofur or Naxcel used as labelled has a 4 day withdrawal time in meat.

It has zero withdrawal time for milk. Chloramphenicol use is not allowed in food animals. Oxytetracline (LA200) has a 28 day meat withdrawal time. Sulfadimethoxine has a meat withdrawal time of 7 days. Penicillin G has a meat withdrawal time of 10 days.

109
Q

You are performing health checks on a beef ranch after an E. coli 0157:7 outbreak was detected due to contaminated crops in a nearby vegetable farm. Which of the following is an appropriate way to identify and test suspected carriers?

  • Send a lymph node biopsy to a diagnostic lab for histopathology and immunohistochemistry from cattle with lymphadenopathy
  • Send swab of sputum or respiratory secretions to a diagnostic lab for 0157:H7 PCR from cattle that are coughing or showing respiratory distresss
  • Send stool to diagnostic lab for 0157:7 PCR from cattle displaying signs of hematochezia
  • Send stool to a diagnostic lab for fecal flotation and Baermann examination in cattle that are emaciated or
    underweight
  • Send stool to diagnostic lab for 0157:7 specific fecal culture from all cattle.
A

Answer: Send stool to diagnostic lab for 0157:7 specific fecal culture from all cattle.

Explanation
E. coli 0157:H7 is an enterohemorrhagic strain of E. coli that produces “Shiga-like toxins” which cause illness in humans. The bacteria can exist in the intestines of healthy, normal cattle without causing signs or symptoms; therefore, there are no specific signs to look for when trying to identify suspects.

A stool culture can detect the bacterium, but must be specifically requested. The sample is usually cultured on sorbitol-MacConkey (SMAC) agar. On SMAC agar, 0157 colonies appear clear due to their inability (unlike other E. coli serotypes) to ferment sorbitol. Non-sorbitol fermenting colonies are tested for the somatic 0157 antigen before being confirmed as E. coli 0157.

Like all cultures, diagnosis is slow using this method, and swifter diagnosis is possible using PCR techniques. A PCR to detect Shiga-like toxin specific to 0157-H7 would be an equally good choice. You would pick culture or PCR based on speed and cost.

In humans, common signs of infection are severe, acute hemorrhagic diarrhea and abdominal cramps, usually without fever. In most cases, the illness resolves in 5 to 10 days.

110
Q

Nervous coccidiosis is a disease of cattle of what age?

  • Less than 1 year of age
  • Between 12-24 months of age
  • Greater than 24 months of age
  • Less than 1 month of age
A

Answer: Less than 1 year of age

Explanation
The correct answer is nervous coccidiosis is a disease of cattle less than one year of age. This disease is caused by an enteric infection of Eimeria spp. It is thought that the coccidian produces a neurotoxin which results in the neurologic form of the disease.

111
Q

In performing ocular ablation (removal) to treat severe cancer eye in a commercial beef cow, which of the following is the most appropriate anesthesia to use in the field?

  • General anesthesia using fluothane intubation following intramuscular xylazine
  • A five point orbital block using lidocaine
  • General anesthesia using sodium pentathol (barbiturate)
  • Sedation using xylazine and restraint in a squeeze chute
  • Topical anesthesia using proparacaine or tetracaine
A

In performing ocular ablation (removal) to treat severe cancer eye in a commercial beef cow, which of the following is the most appropriate anesthesia to use in the field?

  • General anesthesia using fluothane intubation following intramuscular xylazine
  • A five point orbital block using lidocaine
  • General anesthesia using sodium pentathol (barbiturate)
  • Sedation using xylazine and restraint in a squeeze chute
  • Topical anesthesia using proparacaine or tetracaine

Answer: A five point orbital block using lidocaine

Explanation
Squamous cell carcinoma is by far the most common ocular tumor in cattle and is referred to as “cancer eye”.

Treatment can vary from resection of part of an eyelid to complete enucleation including the eyelids.
Recurrence is common, as spread to the lymph node can occur if not caught early on.

The cow should be restrained in a squeeze chute and the head tied to one side so that the abnormal eye is accessible. The area should be closely clipped or shaved and disinfected, then generous amounts (10 ml per site, 50 ml total) of local anesthetic are injected using a 19 ga 2.5 inch (6 cm) needle. The 5 injection sites include the medial canthus, and then twice through each lid so that the tissues around the globe are flooded with local. Alternatively the Peterson ‘s orbital nerve block (another local block) can be utilized.

112
Q

This herd of cattle has been dealing with a Moraxella bovis outbreak in calves. In an effort to control the outbreak, it is important to stop the vector from transmitting the disease. Which one of the following is NOT a factor in the transmission of Moraxella bovis?

  • Musca autumnalis
  • Sarcoptes scabiei
  • Stomoxys calcitrans
  • Lyperosia irritans
A

Answer: Sarcoptes scabei

Explanation
The correct answer is Sarcoptes scabiei. Flies are the main vector of M. bovis, a.k.a. pink eye. Stomoxys calcitrans is the stable fly; Haematobia irritans is the horn fly; Musca autumnalis is the face fly. The face fly is one of the most commonly implicated vectors of all the flies. Sarcoptes scabiei is a reportable mite which induces severe pruritus and, as a result, may be economically devastating, but it is not involved in Moraxella transmission.

***PowerPage: Infectious Bovine Keratoconjunctivitis (Pinkeye)

113
Q

Several beef cows present with a history of decreased appetite and excessive salivation. On physical exam, their tongues are firm on palpation, nodular, and painful (see image). You diagnose actinobacillosis. What is your recommendation to the owner?

  • Isolate affected animals and submit one of them for necropsy
  • Sell affected animals for meat
  • Change feed and treat
  • Isolate animals at once
  • Begin therapy with an aminoglycoside
A

Answer: Change feed and treat

Explanation
The correct answer is to change feed and begin treatment of the animals affected with woody tongue, as the response is often good. Sodium iodide and antibiotics are effective. Given the presentation and clinical signs, these animals have probably begun to ingest very rough and stemmed (scabrous) feed items that have injured their mouths. Upon injury, the normal inhabitant Actinobacillus lignieresii invades the soft tissues and causes the characteristic woody tongue granulomatous inflammation.
These animals don’t have rabies, and there is no need to cull them. Change feed before additional animals are affected. Do not use aminoglycosides as they have an extremely long withdrawal period.

114
Q

A beef cattle farm in the mountains of California is experiencing an abortion storm affecting about 50% of heifers.
The abortions are occurring primarily in the last trimester. The cows appear healthy; some of the aborted fetuses have hepatosplenomegaly and generalized lymphadenopathy. A necropsy on one of the aborted fetuses shows lymphoid hyperplasia in the spleen and granulomatous inflammation in the liver. In the thymus, extensive macrophage infiltration into the medulla was seen as well as loss of cortical thymoctes. Fetal IgG was markedly elevated. Based on the likely diagnosis, which of the following measures would decrease the future incidence of this problem?

  • Control of Ponderosa pine trees
  • Control of mosquitoes
  • Vaccinate heifers against bovine herpesvirus
  • Expose heifers to endemic areas before breeding age
  • Avoid feeding silage to heifers
A

Answer: Expose heifers to endemic areas before breeding age

Explanation
This is the typical presentation for epizootic bovine abortion (EBA), also known as foothill abortion. The agent causing foothill abortion remains a subject of some investigation but a 2005 molecular study identified a novel deltaproteobacterium as being present in the Ornithodoros coriaceus tick vector and in DNA from fetal thymuses. With EBA, cows usually do not experience abortions again in later pregnancies and exposure to endemic areas before breeding age can also prevent abortions. Other control options may include control of the tick vector or prophylactic antibiotic use.

You can lower your index of suspicion for bovine herpesvirus (Infectious bovine rhinotracheitis virus) because there is no respiratory disease seen in this case; abortions from herpesvirus occur throughout pregnancy and fetuses are typically autolyzed with foci of necrosis in the liver or no gross lesions.
Ponderosa pine needle abortions occur in the last trimester but cows are often moribund after delivery and hemorrhage. There are usually no specific fetal lesions.

115
Q

What is the age of a beef cow which has only the first and second permanent incisors present?

  • Approximately 2.5 years
  • Approximately 6 months
  • Approximately 1 month
  • Approximately 1.5 years
A

Answer: Approximately 2.5 years

Explanation
The correct answer is approximately 2.5 years of age. A good rule of thumb is that the permanent incisors are in wear at 1.5, 2.5, 3.5 and 4.5 years of age (permanent incisors 1, 2, 3, and 4 respectively).

116
Q

A dairyman calls to say he has lost several 5 week old calves to acute respiratory signs in the last week. The calves have fever, depressed attitude, decreased appetite, cough and nasal discharge. They do not seem to respond to antibiotic therapy and may die within a few days. On postmortem of one, you find the lungs fail to collapse when the thorax is opened (see image), and they have interstitial emphysema and a rubbery consistency. You believe this is most likely caused by Bovine Respiratory Syncytial Virus (BRSV). What should now be recommended for the herd?

  • Vaccination of all calves against IBR
  • Metaphyllaxis of all calves from birth with tetracycline IM every 3 days
  • Intratracheal use of antiviral drugs in all affected calves
  • Vaccination of all calves against BVD
  • Vaccination of all calves against BRSV
A

Answer: Vaccination of all calves against BRSV

Explanation
Until the calves have time to develop immunity, new animals should be raised apart from and isolated from the current group of calves where the virus is circulating. Treatment of individual calves affected by BRSV may include antimicrobial drugs and NSAIDs; some veterinarians advocate the use of corticosteroids if pulmonary edema is severe.

117
Q

You are in the process of auscultating a cow that presented for respiratory distress and you hear no breath sounds in the dorsal chest. What is your tentative diagnosis?

  • Fibrinous pleuropneumonia
  • Aspiration pneumonia
  • Pulmonary contusion
  • Pneumothorax
A

Answer: Pneumothorax

Explanation
The correct answer is pneumothorax. Since there is free air in the chest, the lung has lost its vacuum and is collapsed. The free air moves dorsally and you are unable to hear any lung sounds. Fibrinous pleuropneumonia will have lung sounds associated with it. Aspiration pneumonia is usually cranioventral in location. Pulmonary contusion is a differential, but given the location, a pneumothorax is more likely.

118
Q

Mucosal disease, or alternatively, chronic Bovine Virus Diarrhea (BVD) occurs in cattle when:

  • A 5-month old calf which was persistently infected with the nonCE biotype of BVD virus as a fetus forms antigen-antibody complexes which cause a fatal immune-mediated disorder.
  • A 5-month old calf is infected with CPE biotype of BVD virus and then superinfected with a nonCE
    biotype of BVD virus.
  • A 5-month old calf is persistently infected with a strain of BVD virus which coats platelets; platelets are then removed by the RE system resulting in a bleeding diathesis.
  • A 5-month old calf which was persistently infected as a fetus with a non cytopathic (nonCPE) biotype of BVD virus is superinfected with a cytopathic (CPE) biotype of BVD due to rearranging of the parent non-CPE viral RNA.
  • BVD type 2 infects a 5-month old calf.
A

Answer: ** A 5-month old calf which was persistently infected as a fetus with a non cytopathic (nonCPE) biotype of BVD virus is superinfected with a cytopathic (CPE) biotype of BVD due to rearranging of the parent non-CPE viral RNA.**

Explanation
This is the correct answer because it describes the current understanding of the pathogenesis of both mucosal disease and chronic BVD.

A 5-month old calf is infected with CPE biotype of BVD virus and then superinfected with a nonCPE biotype of BVD virus is incorrect because it does not say that there was persistent fetal infection; also, the order of the superinfection is the reverse of what causes mucosal disease or chronic BVD.

A 5-month old calf which was persistently infected with the nonCE biotype of BVD virus as a fetus forms antigen-antibody complexes which cause a fatal immune-mediated disorder is incorrect because this is not the mechanism by which mucosal disease or chronic BVD occur.

All other answer choices are incorrect because they each describe one of the acute clinical syndromes from primary infection, rather than either mucosal disease or chronic BVD.

**PowerPage: Bovine Viral Diarrhea

119
Q

A rancher’s favorite bull has a lesion in the interdigital area of the left front foot (see image). The bull has recently become slightly lame on the left front leg. You tell the rancher that it is a corn. The owner wants you to fix it. What treatment should you undertake?

  • Tetracycline topically daily
  • Inject 10% formalin into the lesion
  • Procaine penicillin IM daily for 5 days
  • Surgical removal using sedation and local anesthesia
  • General anesthesia and careful dissection to surgically remove the mass
A

Answer: Surgical removal using sedation and local anesthesia

Explanation
The medical term for a corn is interdigital hyperplasia. Corns are thought to result from stretching of the insertions of the distal interphalangeal ligament in heavy beef breeds. In dairy breeds, corns can occur when cows are continuously standing in slurry, or have some other chronic source of irritation such as interdigital dermatitis.

Corns are best treated by surgical removal of the mass, being careful to avoid the interdigital fat pad. The area is then bandaged, and the claws can be temporarily wired together if necessary. Wiring the claws is often not necessary if the surgery is only skin deep.

120
Q

A 6-year old dairy cow presents with a 1-week history of progressive abdominal distension and loss of body condition. The cow has had a poor appetite and appears anxious. On rectal exam, there is only scant feces that are hard fecal balls coated in mucus. The abdomen is markedly distended from “10 to 4 o’clock” when viewed from behind (see image). Temperature is 100.9 F (38.3 C), heart rate is 36 beats per minute, and respiratory rate is 24 breaths per minute. There is decreased force of rumen contractions occurring four times per minute. You pinch the cow’s withers and she lowers her withers normally. Passage of a stomach tube yields a small amount of gas. Ultrasound of the anterior abdomen is within normal limits (no abscesses seen). Abdominocentesis yields a small quantity of straw-colored fluid with low protein concentration and low numbers of cells, mainly lymphocytes and is considered within normal limits. What should you tell the farmer?

  • The cow appears to have an impaction from Ostertagia or Haemonchus and should be treated with levamisole
  • The cow appears to have free gas bloat and requires trocarization of the rumen and possible rumenotomy
  • The cow appears to have vagal indigestion of undetermined cause and is unlikely to respond to medical management
  • The cow appears to have frothy bloat and requires antifoaming agents
A

Answer: ** The cow appears to have vagal indigestion of undetermined cause and is unlikely to respond to medical management**

Explanation
This cow likely has vagal indigestion based on the chronicity and clinical appearance. Frothy bloat is typically much more acute. Due to the severity and chronicity of this cow’s signs, its prognosis should be considered poor even without determination of the specific underlying cause. Most cases of vagal indigestion require surgery to identify the underlying cause and medical management alone is usually ineffective.

121
Q

Which of the following is most appropriate for increasing the rumen pH of a cow with rumen acidosis to the optimum range?

  • Vinegar
  • Bismuth subsalicylate
  • Magnesium oxide
  • Propylene glycol
A

Answer: Magnesium oxide

Explanation
The correct answer is magnesium oxide, which becomes Mg OH in the rumen. You may also use magnesium hydroxide or magnesium carbonate. Vinegar will cause acidification of the rumen and is therefore a poor choice.

Propylene glycol and bismuth subsalicylate will not affect rumen pH significantly.

122
Q

This dairy cow in the picture presents for a decrease in appetite and milk production. You systematically do your physical exam and as you percuss and auscultate you hear a monotone “boink” on the left side, dorsally, approximately between rib spaces 9-13 and extending to the hip. What gas-filled organ are you hearing?

  • Rumen
  • Cecum
  • Spiral colon
  • Abomasum
A

Answer: Rumen

Explanation
The correct answer is rumen. The location extending up to the hip and the monotone “boink” (not ping) are indicative of the rumen. A cow with LDA would have a variable-pitched ping heard to only just in back of the last rib (not to the hip). It is variable because the rumen is usually still contracting medial to the displaced abomasum, and the changing gas pressures result in a pitch that changes during percussion and auscultation.

123
Q

At what age will you see a permanent third incisor in wear (not age of eruption) in a cow?

  • Approximately 3.5 years
  • Approximately 1.5 years
  • Approximately 6 months
  • Approximately 2.5 years
  • Approximately 4.5 years
A

Answer: 3.5 y

Explanation
The correct answer is approximately 3.5 years of age. A good rule of thumb is that the permanent incisors are fully erupted and in wear at 1.5, 2.5, 3.5 and 4.5 years of age (permanent incisors 11, 12, 13 and 14 respectively).

The first permanent incisor erupts at 18 to 24 months, the second at 24 to 30 months, the third at 33 to 36 months, and the fourth erupts at 42 and 48 months. This means that the rough rule of thumb is a bit off for 11 in some animals, but it does help to remember these approximate times. There is some individual and some breed variability. After eruption, each tooth takes about 6 months before it is in wear.

124
Q

A 3-year old Holstein dairy cow is one month fresh and the milker has noted that her production has dropped in the last week. On physical exam you find normal TPR, a gaunt abdomen, scant but normal-appearing feces, and no abnormalities on rectal exam. She has a prominent ping with variable changing pitch on the left side of the abdomen between the 10th and 13th ribs when simultaneous auscultation and percussion are used. The image shows a model of her abdominal organs viewed from the left side. What is the correct diagnosis?

  • Abomasal volvulus
  • Gas in the rumen
  • Free abdominal gas from uterine rupture
  • Left displaced abomasum
  • Right displaced abomasum
A

Answer: Left displaced abomasum

Explanation
The gas-filled abomasum has moved from its normal position near the ventral midline to the left of the rumen, where it is trapped (see image). As gas bubbles in and out, it sounds like the end of a toilet flush. When percussed and auscultated, the changing gas pressures yield a variable high-pitched ping.

***PowerLecture: Left Displaced Abomasum

125
Q

A 10-day old commercial dairy calf has diarrhea that is white in color (see photo). The calf is dehydrated, hypovolemic, weak and unable to stand. T=10OF (37.8 C), HR=100, and RR=20. No other abnormalities are found on physical examination. Based on these findings, what is the treatment of choice?

  • IV fluids containing only saline
  • IV fluids with added sodium bicarbonate
  • Oral fluids containing high levels of both sodium and chloride
  • Oral fluids containing sodium bicarbonate
  • IV fluids containing 50 meq/L of potassium
A

Answer: IV fluids with added sodium bicarbonate

Explanation
This calf is typical of those suffering from nonspecific calf diarrhea, most often associated with enteropathogenic E coli, rotavirus, or cryptosporidium. The calf develops hypovolemia and metabolic acidosis and requires sodium-containing IV fluids which contain additional alkali such as sodium bicarbonate.

One can assess that the calf is severely acidotic given that it is lethargic and unable to stand. While oral fluids may also be useful, at this stage, the calf will require IV fluids.

***PowerLecture: Calf Diarrhea Part 2

126
Q

A 6-month old feedlot steer, which entered the feedlot 4 weeks ago, has lost weight and is now showing an enlarged left flank as shown in the image below. On exam, you find the left side of the abdomen to be gas-filled under moderate pressure, and the rumen to be otherwise poorly filled and with poor motility. Based on percussion and auscultation, the animal also appears to have chronic bronchopneumonia. What is the most likely cause of the rumen malfunction?

  • Left displaced abomasum
  • Frothy bloat
  • Cecal dilation
  • Type 3 vagal indigestion
  • Free gas bloat, failure to eructate
A

Answer: Free gas bloat, failure to eructate

Explanation
This is sometimes called Type 1 vagal indigestion, or free gas bloat. It is frequently associated with swollen mediastinal lymph nodes caused by pneumonia. The signals to or from dorsal rumen receptors, which detect gas pressure and open the cardia, are compromised such that eructation does not occur normally and free gas bloat occurs. This in turn causes the calf to feel full and it eats poorly and loses weight. One needs to treat the pneumonia and perhaps create a temporary rumen fistula to allow the escape of gas until eructation returns to normal.

***PowerLecture: Vagal Indigestion

127
Q

You are called out to see a 2-month old beef calf that collapsed this morning (see image). The farmer reports that the calf appeared well-nourished and was doing well a couple of days ago. You examine the calf and find temperature 100.6 F or 38.1 C, pulse 120 bpm, and respiratory rate 32 bpm with a slight expiratory grunt. The calf’s eyes are sunken and you estimate that it is 7-10% dehydrated based on decreased skin turgor. The ocular and oral mucous membranes appear congested. The abdomen is distended and palpation elicits a painful grunt. A quick field ultrasound shows several liters of peritoneal fluid and abdominocentesis yields blood tinged fluid. You quickly prepare the calf for a midline exploratory laparotomy and you identify a single, focal 1.5 cm perforating abomasal ulcer. With appropriate surgical and medical intervention, what is the calf’s prognosis?

  • Grave (‹10% chance of survival and recovery)
  • Good (80-90% chance of survival and recovery)
  • Guarded (20-40% chance of survival and recovery)
  • Fair (50-70% chance of survival and recovery)
  • Excellent (>95% chance of survival and recovery)
A

Answer: Grave (‹10% chance of survival and recovery)

Explanation
This calf has an abomasal perforation and acute septic peritonitis based on the description. The prognosis is grave, even with prompt veterinary attention. The calf’s signs of dehydration, weakness, and expiratory grunt are additional poor prognostic indicators. Very few animals recover from diffuse peritonitis and those that do will have massive abdominal adhesions. Acute abomasal perforation through a single 1-2 cm ulcer occurs sporadically in young (2-4 month old) beef calves and the cause is unknown.

128
Q

You have decided that you need to do an exploratory right flank surgery on a valuable cow because she has colic and abdominal distention, and is passing scant amounts of dark red feces. There are no obvious pings. She is dehydrated (skin turgor is abnormal and eyes sunken) and totally anorectic. The owner reports that she was normal two days ago when he last looked at her closely. On exploratory surgery under local anesthesia you find a hard 6- to 8- inch long by 4-inch diameter mass in the small intestine, as shown in the photo. It is painful to the cow when it is touched. The bowel proximal to the mass is distended and the bowel distal is empty. How would you treat this?

  • Perform a surgical bypass of the mass but do not remove it
  • Close the cow and give antimicrobial drugs, laxatives and IV fluids
  • Surgically resect the intussusception
  • Manually reduce the mass and close the cow
  • Euthanize the cow as this is an inoperable tumor
A

Answer: Surgically resect the intussusception

Explanation
Hopefully you will be able to recognize that the “mass” visualized in this image is an intussusception. After removal, an end-to-end anastomosis of the intestine is performed, and the cow is aggressively treated with antimicrobial drugs and IV fluids with a good content of chloride, such as saline, since these animals are usually suffering from hypochloremic hypokalemic metabolic alkalosis.

***PowerLecture: Colic

129
Q

A beef cattle farmer from California has had recurrent problems with roundworm infestations of Ostertagia ostertagi. The fall rains are approaching and, historically, this has been a time when he has had large numbers of cattle become ill. You randomly select 10 animals to perform a fecal egg count on and do not find elevated egg levels. What do you suspect is occurring and what will you recommend?

  • The cattle are unlikely to have an outbreak this year; no treatment is needed
  • The cattle are suffering from type Ill ostertagiasis; treat with fenbendazole
  • The cattle are suffering from type ll ostertagiasis; treat with ivermectin
  • The cattle are suffering from type I ostertagiasis; treat with fenbendazole
  • The cattle are suffering from type I ostertagiasis; treat with levamisole
A

Answer: The cattle are suffering from type ll ostertagiasis; treat with ivermectin

Explanation
Outbreaks in autumn into winter in climates like California with hot dry summers and cool wet fall and winter are associated with type Il ostertagiasis in the cool season. In this manifestation of disease, fecal egg counts are low and there are thousands of inhibited early fourth-stage larvae ready to exit the abomasal glands. Once a full blown type Il infection occurs, many larvae emerge, causing dysfunction of the abomasum and possibly leading to severe albumin loss, diarrhea, and even death.

In contrast, type I ostertagiasis in California climates occurs in winter and spring following rapid infection with large numbers of L3 larvae from heavily contaminated pastures (or on irrigated summer pasture). Egg counts in this scenario tend to be much higher and clinical disease is most common in younger animals after weaning.

However, in older animals, after invading the abomasal mucosa, many of the larvae have inhibited or delayed development at the L4 stage and wait for favorable climate conditions to emerge and cause type Il disease.

In climates with rainy summers and freezing winters the emergence will often be in spring. So climate affects the time that Type Il is seen.

As for treatment, benzimadazoles (i.e. fenbendazole) and ivermectin are generally more effective than levamisole but all are treatment options. Answer: The cattle are suffering from type ll ostertagiasis; treat with ivermectin

Explanation
Outbreaks in autumn into winter in climates like California with hot dry summers and cool wet fall and winter are associated with type Il ostertagiasis in the cool season. In this manifestation of disease, fecal egg counts are low and there are thousands of inhibited early fourth-stage larvae ready to exit the abomasal glands. Once a full blown type Il infection occurs, many larvae emerge, causing dysfunction of the abomasum and possibly leading to severe albumin loss, diarrhea, and even death.

In contrast, type I ostertagiasis in California climates occurs in winter and spring following rapid infection with large numbers of L3 larvae from heavily contaminated pastures (or on irrigated summer pasture). Egg counts in this scenario tend to be much higher and clinical disease is most common in younger animals after weaning.

However, in older animals, after invading the abomasal mucosa, many of the larvae have inhibited or delayed development at the L4 stage and wait for favorable climate conditions to emerge and cause type Il disease.

In climates with rainy summers and freezing winters the emergence will often be in spring. So climate affects the time that Type Il is seen.

As for treatment, benzimadazoles (i.e. fenbendazole) and ivermectin are generally more effective than levamisole but all are treatment options.

130
Q

You are called to a small beef and sheep ranch where the owner is concerned about a 1-year old steer which has cloudy eyes, mucoid nasal discharge, thickened and cracked skin, and diarrhea. You examine the animal and find T=107F or 41.7 C, HR=105, RR=40, corneal opacity and thick white nasal discharge (see photo), thick cracked skin all over the animal, enlarged prescapular lymph nodes, and diarrhea with small amounts of blood in it.
There are also some oral erosions. Based on the PE you tentatively diagnose this condition.

  • Malignant catarrhal fever (MCF)
  • Bovine papular stomatitis
  • Vesicular stomatitis
  • Gossypol toxicity
  • Bovine viral diarrhea (BVD)
A

Answer: Malignant catarrhal fever (MCF)

Explanation
MCF is caused by a herpes virus carried by sheep in North America and called ovine herpesvirus type 2.

The virus affects lymphocytes and allows the animal’s own killer cells to attack blood vessels, resulting in arteritis. Arteritis results in multisystemic signs, high fever, and enlarged lymph nodes. The acute severe form such as described here is usually fatal.

While a small number of cattle with BVD may develop corneal opacity, they do not usually have the enlarged lymph nodes and the combination of other signs seen with MCF.

***PowerLecture: Malignant Catarrhal Fever - MCF

131
Q

A very valuable dairy cow has died after a long illness with fever that the owner had treated with potent antimicrobial drugs for a week, then had given up. He asks you to perform a post-mortem to tell him the cause of death so he can collect insurance money. You find edematous lungs, edema in the ventral thoracic subcutaneous tissues, and this heart lesion. What is the diagnosis?

  • Pulmonary stenosis
  • Tetralogy of Fallot
  • Ventricular septal defect
  • Valvular endocarditis
  • White muscle disease
A

Answer: Valvular endocarditis

Explanation
Bacteria can lodge on heart valves and collect platelets and do damage to valves, causing murmurs and dysfunction and in some cases heart failure. The bacteria usually enter via a septic site such as a hoof abscess.

Valuable cows should be treated with penicillin or ceftiofur for several days if they develop hoof abscesses or similar problem.

132
Q

Three feedlot yearling steers in a small home ranch operation present with star-gazing, blindness, diarrhea, anisocoria, and strabismus. On physical exam, the steers are found to be underweight and in poor condition. The animals are being fed unsupplemented wheat and straw. Pupillary light response is absent in both eyes of all three. The operator reports that yesterday another steer with similar signs convulsed and died when being brought to the squeeze chute. Which of the following is the most likely diagnosis?

  • Vitamin C deficiency
  • Vitamin A deficiency
  • Lead poisoning
  • Salt poisoning
A

Answer: Vitamin A deficiency

Explanation
The correct answer is vitamin A deficiency. The clinical signs are very similar in these two other neurologic diseases. However, pupillary light reflex will stay intact with salt poisoning, and lead poisoning. PLRs disappear with vitamin A deficiency as a result of retinal degeneration and constriction of CN Il at the level of the optic foraminae. Blood work will usually be unremarkable. CS changes in Vit A deficiency include a mononuclear cell pleocytosis of 40-50 cell/dl and increased protein of approximately 140mg/dl. It is more common for unsupplemented feedlot animals to be deficient because they do not have access to vitamin A rich, green, succulent, plants. The vitamin A requirement for most species is between 40-80 IU/kg of body weight daily. Vit C deficiency will not cause CNS signs of this type.

133
Q

A dairy cow that freshened 3 weeks ago presents to you with little rumen motility, protrusion of the third eyelid, and a stiff stilted gait. You suspect that she has been affected by tetanus. The most likely source of the infection is

  • A nail imbedded in her hoof
  • Rumen atony allowing for an anaerobic
    environment
  • A recent metritis/uterine infection
  • A migrating foreign body
A

Answer: A recent metritis/uterine infection

Explanation
The correct answer is a recent uterine infection.
Cows presenting with clinical signs suggestive of tetanus infection often have had a recent uterine infection. Cattle tend to develop tetanus less commonly than horses, sheep, or goats, as they are less sensitive to the toxin. Calving or surgical castration are common in the history. Tetanus may develop from hoof wounds, as occurs in horses, but this is rare in cattle.

The tetanus toxin moves retrograde up the peripheral neuron into the spinal cord, where it prevents release of inhibitory neurotransmitters such as GABA. This inactivation of inhibitory neurotransmission causes spastic paralysis.
It is possible to develop tetanus from hoof wounds, as is common in horses, but this is very uncommon in cattle.

134
Q

A farmer was shipped ten 2-week old calves several days ago. They seemed initially normal but a few of the calves are now growing poorly and disinterested in eating the milk-replacer. One calf began vocalizing and arching her back and also developed white and sticky feces. She became very dehydrated and died before you arrived at the farm. Necropsy reveals mucosal inflammation of the rumen and parakeratosis of rumen epithelium. You advise the farmer that decreasing stress can reduce the incidence of this problem. Which of the following management aspects should you also carefully review?

  • Vaccination program
  • Feeding program
  • Quarantine program
  • Vector control program
  • Animal housing program
A

Answer: feeding program*

Explanation
The clinical description and pathologic findings are consistent with rumen acidosis, likely secondary to dysfunction of the esophageal groove (also known as reticular groove). To review normal physiology, the esophageal groove (also known as reticular groove) is a specialized part of the ruminant stomach that closes in response to drinking milk, guiding it directly to the abomasum. When dysfunctional, milk goes to the rumen where it is fermented by rumen microbes and converted to volatile fatty acids and lactic acid. The resulting rumen acidosis leads to the clinical signs described in this case. For this reason, this phenomenon is sometimes referred to as rumen drinking.

There are a number of predisposing factors to this problem. The major contributors are stress, irregular feeding, feeding poor quality milk replacer, feeding milk at too cold of a temperature (should be body temperature), tube feeding, diarrhea, and bucket drinking. Of the options in this question, the best choice is to review the feeding program because it involves so many of the predisposing factors.

135
Q

You are treating a very valuable 15-day old calf with total parenteral nutrition (TPN) because it had very severe diarrhea, weight loss, and anorexia (see image). You have already added glucose and amino acids to the TPN mixture. What other component must be added?

  • Lactose
  • Sucrose
  • Vitamins
  • Low-density lipoproteins
  • Lipids
A

Answer: Lipids

Explanation
A typical TPN mixture for a calf this size would be 1L of 10% amino acids, 1L of 50% glucose and 500 ml of 10% lipids per day.

136
Q

A beef ranch has suffered 12 acute deaths of 2-4 month old calves in the past six months. The owner of the ranch states that the animals appeared severely weak and depressed just before dying. On some of them, he noticed that they were having trouble breathing and had a frothy nasal discharge. You perform a necropsy on a calf that died yesterday. Findings included bilaterally symmetric muscular atrophy. The skeletal muscle appears pale and dry in appearance with white streaks running along muscle bundles. You also notice that there are several calves in the ranch which are having trouble rising, and their musculature appears swollen, hard, and painful. What is the most likely diagnosis?

  • Clostridial myositis
  • Oleander toxicity
  • Vitamin E and selenium deficiency
  • Septicemia
  • Gossypol toxicity
A

Answer: Vitamin E and selenium deficiency

Explanation
The correct answer is vitamin E and selenium deficiency (white muscle disease). The clinical signs are somewhat compatible with the other answers; however the necropsy results are diagnostic for white muscle disease. There is both a cardiac form, in which animals die acutely, and a skeletal muscle form, in which animals don’t die acutely but show clinical signs. In this particular question, the ranch was suffering from both forms. Remember that oleander and gossypol are both cardiotoxic and can cause acute death. Given the necropsy findings, clostridial myositis would be unlikely. Selenium is essential for glutathione peroxidase, deiodinase, and selenoprotein-P to work. Glutathione peroxidase breaks hydrogen peroxide and lipoperoxide into water or harmless alcohols.

137
Q

A 5-day old dairy calf has decreased appetite and is depressed. On physical exam, you note that she has a temperature of 104F (40 C), HR 120, scleral injection, and one eye has cloudy material in the anterior chamber. Pending lab results, you make a tentative diagnosis of what condition?

  • Malignant catarrhal fever (MCF)
  • Failure of passive transfer (FPT) and sepsis, with hypopyon
  • Chlamydia psittaci ocular infection and sepsis
  • Pinkeye caused by Moraxella bovis
  • Bovine viral diarrhea (BVD)
A

Answer: Failure of passive transfer (FPT) and sepsis, with hypopyon

Explanation
The history of a neonate with these signs and an elevated temperature, scleral injection and hypopyon are indicative of bacterial sepsis, a result of FPT.

138
Q

A dairy farmer in California calls you in distress. He has just found 16 dead yearling Holstein heifers in a pasture near his home. There are no survivors. All 16 were observed yesterday and appeared normal. You perform a postmortem on two animals and find the plant pictured in the rumen. You walk out to look at his pasture and note that this plant is growing in the cemetery adjacent to his pasture, and you find a pile of clippings containing grass and the plant near the fence. What is the plant?

  • Yew
  • Skunk cabbage
  • Oleander
  • Locoweed
  • Lupine
A

Answer: Yew

Explanation
Both Japanese yew (Taxus cuspidata) and English yew (T. baccata) contain alkaloids called taxines that are highly toxic and depress myocardial conduction by blocking sodium movement through membranes. Collapse and sudden death are commonly seen when this plant is ingested, often when clippings are consumed by livestock.

Oleander causes similar clinical scenarios.

139
Q

What gram positive organism is an obligate pathogen of the mammary gland of cows?

  • Staphylococcus aureus
  • Streptococcus agalactiae
  • Corynebacterium bovis
  • Mycoplasma
A

Answer: Streptococcus agalactiae

Explanation
The correct answer is Streptococcus agalactiae. This organism usually causes subclinical mastitis, but sometimes you may see high somatic cell counts and clinical signs with this organism. Perform a CAMP reaction to diagnose this organism. That is when you plate S. aureus and S. agalactiae and you see them act together to lyse red blood cells. This creates a clearing on your culture plate.

140
Q

A cow presents with a hard bony mass lesion at the ventral mandible (see image). Which treatment is known as being effective but is not recommended in pregnant animals as a result of concerns about it causing abortion and not for use in lactating dairy animals as a result of food safety issues?

  • Oxytetracycline
  • Intravenous sodium iodide
  • Ceftiofur
  • Corticosteroids
  • Penicillins
A

Answer: Intravenous sodium iodide

Explanation
The correct answer is intravenous iodine. It is commercially available as a 20% solution at a dose of 15 ml/100 Ibs (66 mg/Kg). The clinical signs are suggestive of lumpy jaw, which is caused by Actinomyces bovis. The prognosis is poor, so most animals should be culled. Penicillins are reportedly successful at arresting the lesion in very early cases. However, there may be a better chance with intravenous 20% sodium iodide. That being said, due to concerns that iodide may cause abortion and because of food safety concerns, the label warns not to use sodium iodide in pregnant or lactating cattle. Ceftofur may not be used in this extra label fashion.

141
Q

A purebred Holstein breeder wishes to rid his dairy herd of Leptospira interrogans serovar Hardjo (type Hardjo-Bovis), which has been causing reproductive problems in his herd. Which steps are most likely to accomplish this goal?

  • Treat all animals with dihydrostreptomycin
  • Regularly vaccinate all cattle and eradicate rodents, as they are the maintenance hosts of this serovar
  • Regularly vaccinate all cattle and eliminate all dogs from the farm
  • Treat all animals with tetracycline to eliminate carriers and regularly vaccinate all cattle against this serovar
  • Vaccinate all animals in the herd regularly
A

Answer: Treat all animals with tetracycline to eliminate carriers and regularly vaccinate all cattle against this serovar

Explanation
Since Hardjo-Bovis is carried by cattle, he needs to both treat and vaccinate. Vaccination can prevent new carriers, but will not eliminate existing carriers. To be cost effective, he may need to mass treat dry cows and young stock using long acting tetracycline.

Swine (and opossums, skunks and raccoons) are carriers of Pomona, dogs of Canicola, rats of Icterohemorrhagiae, swine, mice and horses of Bratislava, and raccoons, muskrats and squirrels of Grippotyphosa.

142
Q

Mycobacterium bovis is an important zoonotic disease spread from cattle to humans which can be controlled by testing and removal. Recently, new sources of infection for cattle which act as a reservoir have been discovered in the USA. Where are these reservoir sources of infection?

  • Humans with Mycobacterium tuberculosis
  • Sheep
  • Wildlife (deer)
  • Birds feeding on farms
  • Donkeys
A

Answer: ** Wildlife (deer)**

Explanation
In the USA, the most common reservoir infecting cattle are white tailed deer; badgers in Great Britain; brushtail possums in New Zealand. These reservoirs make control and eradication very difficult.

143
Q

A 6-year old Friesian cow presents to you with a mass on the left mandible (see image). The farmer reports that the mass has developed over the last several weeks and the cow has recently had some difficulty eating and lost weight. On examination, the mass is firm, immobile, and painful on manipulation. You note a thick discharge with small granular particles. Based on the most likely diagnosis, what should you tell the farmer?

  • The most effective treatment is oral sodium
    iodide
  • Treatment is unlikely to be successful and the cow should be culled
  • Treatment with sodium iodide intravenously is likely curative but the cow’s milk and meat will not be suitable for human consumption for 120 days
  • Intravenous penicillins are likely to be effective
A

Answer: Treatment is unlikely to be successful and the cow should be culled

Explanation
This is a case of Actinomyces bovis or “lumpy jaw”.

The keys to the diagnosis are the firm mass that is immobile over the mandible. The presence of
“sulfur granules” in the discharge is also a characteristic finding with this disease.

Unfortunately, simply making the diagnosis is not sufficient to answer this question correctly. All of the answer choices are potential treatments for lumpy jaw but this represents a moderate to severe case which is unlikely to respond to treatment due to difficulty in achieving the necessary antibiotic concentration over a sustained period. Therefore, the best answer choice in this case is to cull the cow.

144
Q

It is March and you are called to a beef ranch because several beef cows on lush pasture have been found dead and another is staggering, according to the rancher. By the time you arrive this cow is down on her side with her legs paddling. The HR=140 and is pounding, while T=103F (39.4 C). The eyelids are fluttering, there is nystagmus, and champing of the jaws. Which of the following disorders fits these signs best?

  • Copper deficiency
  • Hypocalcemia
  • Hypokalemia
  • Hypomagnesemia
  • Lead poisoning
A

Answer: Hypomagnesemia

Explanation
Hypomagnesemic tetany (grass tetany) can cause all of these signs. It usually occurs in cold or cool weather in pastured lactating beef or dairy cows. Lush pastures that are high in potassium and nitrogen and low in magnesium and sodium are most often involved. Hypocalcemia may cause similar signs in periparturient cows (milk fever).

145
Q

A 3-year old open beef cow presents as a result of lameness in her right hind limb. Upon physical examination, you notice an ulcerated necrotic lesion in the interdigital space. Which of the following is NOT a differential diagnosis?

  • Foot-and-Mouth disease
  • Interdigital necrobacillosis
  • Infectious bovine rhinotracheitis
  • Bovine viral diarrhea
  • Malignant catarrhal fever
A

Answer: Infectious bovine rhinotracheitis

Explanation
The correct answer is infectious bovine rhinotracheitis. There are three subtypes of infectious bovine rhinotracheitis identified. BHV-1.1 (Bovine herpes virus) is responsible for respiratory infections. BHV-1.2 may cause genital infections and respiratory disease. BHV-1.3 (reclassified recently and now called BHV 5) has been known to cause neurologic infections. Since most people are not aware of the reclassification, this aspect of the issue is unlikely to be questioned on a board examination. All of the other listed answer choices can cause the lesion described in the question.

146
Q

Young cattle 5 to 7 months of age introduced to a feedlot are at risk of experiencing respiratory disease. Which of the following describes the most effective method of decreasing the number of calves becoming ill in these circumstances?

  • Treating all calves with one injection of long-acting tetracycline upon entry
  • Preconditioning all calves prior to entry into the feedlot
  • Treating all calves with intranasal modified live virus BVD vaccine upon entry
  • Deworming with levamisole to act as an immunostimulant at entry
A

Answer: Preconditioning all calves prior to entry into the feedlot

Explanation
Preconditioning involves being sure that calves are weaned several weeks before entering the feedlot, vaccinated against infectious bovine rhinotracheitis (IBR), bovine viral diarrhea (BVD), Parainfluenza-3 (PI3), and Clostridial diseases. They should be dewormed, and dehorned and castrated if not already done. Ideally, the calves should also have learned to eat from a bunk and drink from a trough before entering the feedlot. Then, when the calves are mixed with calves from other ranches, they are less likely to succumb to respiratory disorders.

147
Q

You examine a Holstein steer with the complaint of poor appetite, depressed attitude, and ventral swelling. You note a normal TPR, but he has a large ventral swelling that pits on palpation (see image). On rectal exam, a small bladder is palpable. What is the correct diagnosis?

  • Corynebacterium pseudotuberculosis infection
  • Umbilical hernia
  • Penile hematoma
  • Ruptured bladder
    -Ruptured urethra
A

Answer: Ruptured urethra

Explanation
In the bovine, urolithiasis often results in rupture of the urethra. Urine then leaks into all the ventral tissues and causes this massive ventral edema, which progresses to necrosis and sometimes to gangrene.

***PowerLecture: Urolithiasis In Ruminants

148
Q

You are presented with a Holstein steer in a feedlot because he is not eating well. He has a normal temperature, HR=80 and RR=20. There is ventral abdominal edema and his breath smells like ammonia. Ultrasonography of the abdomen reveals free abdominal fluid and a collapsed urinary bladder.
You suspect a ruptured bladder and urethra with secondary uroperitoneum due to a urolith in the urethra. Which of the following would be the most cost effective treatment?

  • Surgically repair the bladder and urethral defects under general anesthesia
  • Treat with sodium containing IV fluids
  • Perineal urethrostomy
  • Place a Foley catheter in the bladder percutaneously
A

Answer: Perineal urethrostomy

Explanation
An exit for the urine is made surgically in the perineal area. This is a salvage procedure to get the animal to slaughter in one to 3 months when the tissue damage from any extravasated urine resolves sufficiently in the event that the urethral obstruction was more distal.

Treatment with sodium containing intravenous fluids would be ineffective as long as there is active leakage of urine into the abdomen. A surgical correction or diversion is necessary to stabilize this patient. Since this is a feedlot steer the most cost effective management decision is to perform a perineal urethrostomy proximal to the point of obstruction. This repair assumes that the ruptured bladder will be able to heal on its own.

***PowerLecture: Urolithiasis In Ruminants

149
Q

A 4 year old dairy cow presents for circling to the right, poor appetite and depressed attitude. On physical exam, the right side of the cow’s face and the right ear are drooping (facial paralysis). What is the most likely causative agent for these signs?

  • Listeria monocytogenes
  • Pasteurella multocida
  • Histophilus somni
  • Mycoplasma bovis
A

Answer: Listeria monocytogenes

Explanation
Listeria monocytogenes is the most common CNS infection in adult ruminants. Infection is often associated with feeding of silage. It causes microabscesses in the brain resulting most commonly in unilateral cranial nerve signs such as facial nerve paralysis, vestibular signs (circling), head tilt, and dysphagia. CN V-XII may be involved.

Head pressing may be seen in some cases. It is also a cause of abortions. Treatment is with tetracyclines or high levels of penicillin.

Mycoplasma bovis in young calves can affect some animals with otitis interna, so the calf shows (uni or bilateral) facial paralysis, ear droop, and vestibular signs such as head tilt, nystagmus, and ataxia. There may be purulent discharge from the aural canal. M bovis can also cause pneumonia, polyarthritis and tenosynovitis. This condition can be difficult to treat and result in many animals with chronic illness.

Histophilus somni can commonly be a cause of respiratory disease and septicemia, as well as TEME (thromboembolic meningoencephalitis) or otitis media /interna in young feedlot animals where they exhibit head tilt, nystagmus, circling and stumbling. They may also develop ipsilateral facial paralysis. So in this case the main differential is the age of the animal and the fact that this is often a feedlot animal, plus H somni tends to be a more fulminant systemic disorder.

***PowerLecture: Central Nervous System

150
Q

You are called out to examine a sick cow. The cow is dyspneic and has bright red mucous membranes. You suspect the cow has ingested which of the following cyanide-containing plants?

  • Fiddleneck
  • Ragwort
  • Common groundsel
  • Chokecherry
A

Answer: Chokecherry

Explanation
The correct answer is chokecherry. This is a cyanide-containing plant and can cause dyspnea, staggering, and death. Bright red mucous membranes are commonly seen due to the result of the cyanide interfering with the electron transport chain (binds to cytochrome oxidase); hemoglobin is unable to release oxygen. For some reason, the Gl tract has a bitter almond smell on necropsy. All other answer choices are pyrrolizidine alkaloid plants.

151
Q

Which of the following is NOT a dietary deficiency associated with a neurologic disorder in livestock?

  • Copper deficiency
  • Thiamine deficiency
  • Vitamin A deficiency
  • Vitamin D deficiency
A

Answer: Vitamin D deficiency

Explanation
The correct answer is vitamin D deficiency. Vitamin D deficiency will manifest with bone abnormalities (rickets).

Vitamin A deficiency may produce an encephalopathy in which convulsions and blindness are common clinical signs. Copper deficiency may result in demyelination and pathologic fractures of the vertebrae. Thiamine deficiency produces polioencephalomalacia.

152
Q

You are presented with a Holstein bull because he is not eating well. He has a normal temperature, HR=80 and RR=20. There is ventral abdominal edema (see image) and his breath smells like ammonia. Ultrasonography of the abdomen reveals free abdominal fluid and a collapsed urinary bladder. You suspect a ruptured bladder and urethra with secondary uroperitoneum due to a urolith in the urethra. Which of the following test results on blood are most useful in confirming your diagnosis?

  • Hyponatremia, hypochloremia, hyperphosphatemia
  • Azotemia, hypophosphatemia, hypernatremia
  • Hyponatremia, hypokalemia, hyperchloremia
  • Hypernatremia, hyperchloremia, hyperkalemia
A

Answer: Hyponatremia, hypochloremia, hyperphosphatemia

Explanation
These are the electrolyte abnormalities that should be expected in cases of uroabdomen. There would also be azotemia (elevated urea nitrogen), and possibly hyperkalemia in the blood.

***PowerLecture: Urolithiasis In Ruminants

153
Q

Which of the following is not a zoonotic pathogen shed in cow milk?

  • Salmonella dublin
  • Mycobacterium avium ssp paratuberculosis
  • Brucella abortus
  • Campylobacter fetus ssp venerealis
  • Listeria monocytogenes
A

Answer: Campylobacter fetus ssp venerealis

Explanation
The correct answer is Campylobacter fetus ssp. venerealis, which is the main cause of bovine campylobacteriosis but is not shed in milk. This organism is an obligate parasite of the bovine genital tract and causes abortion. Infection usually results in temporary infertility or early embryonic death. A different Campylobacter, C jejuni, is an emerging milk zoonotic pathogen.

M avium ssp paratuberculosis is believed by some individuals to be a possible human pathogen, perhaps responsible for some (but not all) cases of Crohn’s disease. The majority of evidence would disagree with that position, but agree that it should not be present in milk for human consumption.

154
Q

In an effort to control bovine virus diarrhea virus (BVD) in a herd of registered Holstein dairy cattle, the owner and veterinarian have worked to ensure that the herd is now BVD-free. Only a few heifer replacements are purchased, and all breeding is by artificial insemination from a commercial source. Which of the following is the most likely to achieve the goal of maintaining a BVD free herd in the future?

  • Before purchase test all heifer replacements for the presence of BVD antibody using the serum virus neutralization test
  • Vaccinate all heifers at 6 months of age with killed BVD vaccine
  • Be sure that a new clean rectal sleeve is used for each cow when doing rectal exams
  • Before purchase, test all heifer replacements for BVD virus using immunohistochemistry on a skin biopsy
  • Vaccinate all cows over 2 years old with modified live BVD vaccine
A

Answer: Before purchase, test all heifer replacements for BVD virus using immunohistochemistry on a skin biopsy

Explanation
BVD is brought into herds such as this via persistently infected animals that were congenitally infected with
BVD virus. These animals shed virus and infect herd mates. Thus it is necessary to check all purchased animals to be sure they are BVD free.

It would also be wise to also vaccinate all cattle on this dairy with killed BVD vaccine just to ensure that if BVD did appear, it would be less likely to cause a major problem. Initial vaccination requires two injections, followed by a yearly booster. Vaccination alone however, will not keep the herd BVD free if a BVD carrier heifer is brought

155
Q

The image demonstrates a sole abscess on the left rear medial claw of a commercial dairy cow which freshened one month previously. You plan to pare out the abscess and drain it. What additional step is most effective (and cost effective) to keep this cow in production while this problem resolves?

  • Give IM penicillin for 5 days
  • Apply wooden hoof block to affected left rear medial claw
  • Soak the foot twice daily in hot Epsom salts and water for one hour
  • Apply wooden hoof block to left rear lateral claw
  • Give long acting tetracycline injection IM
A

Answer: Apply wooden hoof block to left rear lateral claw

Explanation
This is easy and inexpensive and gets the cow’s weight off the sore medial claw so she will ambulate and eat relatively normally.

156
Q

You examine a valuable beef bull that was recently purchased in Texas and taken to the mountains of Colorado. The pasture is a mountain meadow at 10,000 feet elevation. The bull has developed edema of the brisket and ventral thorax, submandibular edema, dyspnea and tachypnea. Rectal temperature is normal. The HR is 90, and heart sounds are clearly audible without murmurs. What is your diagnosis?

  • Ventricular septal defect
  • Valvular endocarditis
  • Traumatic pericarditis
  • Cardiomyopathy
  • High mountain disease
A

Answer: High mountain disease

Explanation
The key to this question is the altitude mentioned in the question and accompanying clinical signs. Also called brisket disease and high altitude disease, this condition can be fatal and is brought on by elevations above 6000 feet. Hypoxic vasoconstriction (worse in some lines of cattle) causes pulmonary hypertension, which leads to cor pulmonale, which is secondary cardiac disease including right heart enlargement and failure.

157
Q

What is the etiologic agent of thromboembolic meningoencephalitis in cattle?

  • Listeria monocytogenes
  • Pseudomonas aeruginosa
  • Histophilus somni
  • Pasteurella multocida
A

Answer: Histophilus somni

Explanation
The correct answer is Histophilus somni (old name was Hemophilus somnus). TEME affects the cerebellum and brain stem, but H. somni can cause other signs of systemic disease such as pneumonia, laryngeal disorders, joint infections, metritis, orchitis, and conjunctivitis. Initial infection in cattle (usually 4-12 months old) probably occurs through the respiratory tract. Many times respiratory signs are seen before neurologic signs develop. The disease gets its name because endothelial cells become infected and then degenerate and expose the subendothelial collagen thus triggering the clotting cascade and resulting in thrombi (but no emboli). Lesions seen include multifocal hemorrhages and infarctions in the brain stem, cerebral cortex, and spinal cord. CSF fluid will be xanthochromic and cloudy due to increased numbers of neutrophils. This disease can be difficult to distinguish from polioencephalomalacia except that TEME has high fever.

***PowerLecture: Central Nervous System

158
Q

Post-milking teat dips are not effective at preventing this type of mastitis in cows.

  • Mycoplasma bovis
  • Staphylococcus aureus
  • Corynebacterium bovis
  • Streptococcus agalactiae
  • Escherichia coli
A

Answer: Escherichia coli

Explanation
The correct answer is Escherichia coli. This is an environmental contaminant and not a contagious organism. Pre-milking teat dips are useful for preventing such noncontagious diseases as E. coli, Enterococcus, and Streptococcus sp. other than S. agalactiae. Post-milking teat dips help prevent the spread of contagious pathogens such as Strep agalactiae from the skin into the streak canal and gland.

159
Q

A pregnant cow has a uterine torsion. Which of these therapeutic interventions should you attempt if the torsion is clockwise?

  • Administer oxytocin
  • Perform uterine surgery to relieve the torsion
  • Push forward on the vagina while rolling the cow clockwise
  • Push forward on the vagina while rolling the cow counterclockwise
A

Answer: Push forward on the vagina while rolling the cow clockwise

Explanation
The correct answer is push forward on the vagina while rolling the cow clockwise, which may seem counter-intuitive. By stabilizing the uterus and rotating the cow around it, you can attempt to relieve the torsion.

160
Q

A dairyman brings his prize 6-month old bull calf to your clinic because he is not eating all his feed today and has diarrhea. He has just purchased this registered bull calf from a neighboring state, and it was shipped a week ago during some very cold weather. You examine the bull calf and find T=104 F (40 C), HR=90, and RR=35, with poor rumen motility. There are several ulcers on the dental pad, and the diarrhea is foul-smelling and contains flecks of blood. The calf is hemorrhaging from small scleral blood vessels (see photo). You quickly take a blood sample and have your assistant run it to the state diagnostic lab, which calls you less than an hour later to say that the thrombocyte count is 5000 (normal is 100,000-800,000/ul). You tell the owner that the calf appears to have bovine virus diarrhea infection with thrombocytopenia. What should your first line of therapy be if you are to save this calf?

  • IV fluids to treat dehydration
  • Antiviral drug therapy
  • High doses of IV ceftiofur
  • Immunostimulant drugs IV
  • Blood transfusion with fresh whole blood
A

Answer: Blood transfusion with fresh whole blood

Explanation
The thrombocytopenia is caused by BVD virus adhering to thrombocytes which are then removed by the reticuloendothelial (RE) system.

Once the number gets this low, fatal hemorrhage is possible at any moment. Although not usual, this form of BVD has been described. The other forms of therapy may be needed as well. If the calf survives, you will also need to determine whether or not he remains persistently infected with BVD… if he does, he should not be retained.

161
Q

Post-milking teat dips are not effective at preventing this type of mastitis in cows.

  • Mycoplasma bovis
  • Staphylococcus aureus
  • Corynebacterium bovis
  • Streptococcus agalactiae
  • Escherichia coli
A

Answer: Escherichia coli

Explanation
The correct answer is Escherichia coli. This is an environmental contaminant and not a contagious organism. Pre-milking teat dips are useful for preventing such noncontagious diseases as E. coli, Enterococcus, and Streptococcus sp. other than S. agalactiae. Post-milking teat dips help prevent the spread of contagious pathogens such as Strep agalactiae from the skin into the streak canal and gland.

162
Q

What is the cause of alpha-Mannosidosis?

  • Genetic
  • Bacterial infiltration of the gyri
  • Fungal
  • Viral
A

Answer: Genetic

Explanation
The correct answer is genetic (autosomal-recessive). The condition is the result of a defect of the enzyme alpha-mannosidase and is seen in Murray gray, Simmental, Holstein, Galloway, and Angus cattle. The cleavage between N-acetyl glucosamine and mannose will not occur and the oligosaccharide will accumulate within the lysosomes of neurons, reticuloendothelial cells, and macrophages. The first sign at 1 month to 15 months of age will be a mild ataxia of the pelvic limbs after exercise. Other signs then develop, including hypermetria, aggressiveness, and intention head tremors. Most affected individuals will end up getting diarrhea and becoming recumbent and die within months after diagnosis.

163
Q

In January you examine a group of dairy calves which range in age from 2 to 7 months, with a complaint of hair loss and pruritus. The calves are thin and mucous membranes are pale. One has developed bronchopneumonia, and is also febrile and depressed. You do a skin scraping and find the parasite shown in the image, which your technician identifies as Solenopotes sp. The CBC shows the calves to be severely anemic. What treatment recommendation should you now make to the dairy owner?

  • Treat all calves for mange
  • Treat all calves with long acting tetracycline
  • Treat all calves for lice
  • Vaccinate all calves against Mannheimia hemolytica
  • Treat all calves with hematinics
A

Answer: Treat all calves for lice

Explanation
This is a blood sucking genus of louse, and can cause severe anemia. The anemic calves become thin and more susceptible to diseases like pneumonia. You can tell this is a louse and not a tick or a mite because lice are insects with 6 legs and ticks and mites are arachnids with 8 legs.

The three genera of blood sucking cattle lice are Solenopotes, Linognathus, and Hematopinus.

164
Q

You examine a 7-year-old dairy cow with the presenting complaint of poor milk production and weight loss. You find a normal TPR, but the cow is in poor body condition compared to others in the string, and has multiple enlarged lymph nodes at widely scattered locations (see image). You suspect bovine leukosis virus (BLV) has caused clinical lymphoma. Which of the following tests would be the most definitive in making the diagnosis of lymphoma?

  • AGID for BLV antibodies
  • Lymphocyte count
  • ELISA for BLV antibodies
  • WBC count
  • Aspirate lymph node and make slide
A

Answer: ELISA for BLV antibodies

Explanation
The best answer to this question is ELISA for BLV antibodies. A biopsy is definitive if you can demonstrate antigen with immunohistochemistry, but many diagnostic labs do not have this capability, so this answer option was not included for this question. The ELISA for BLV only reveals that the cow has antibodies to BLV virus, but not that she has clinical lymphoma. However, multiple enlarged lymph nodes and a positive BLV antibody test are strongly suggestive of lymphoma.

A negative ELISA test would rule out BLV and associated lymphoma.

Aspiration of lymph nodes can lead to false positives, as many inflamed reactive nodes in cattle can have cells that look neoplastic.

The AGID for BLV antibodies is less sensitive than ELISA. The WBC and lymphocyte count lacks sensitivity and specificity.

165
Q

A worried herdsman calls you out to assess one of his best cows that has developed a large 6-inch diameter sore on the flank with pus and blood associated with it. You suspect this lesion to be caused by Corynebacterium pseudotuberculosis. What should you tell the owner to do?

  • Surgical debridement and call the state authorities
  • Euthanize immediately and incinerate the body
  • Treat aggressively with systemic antibiotics
  • Flush the wound and let it heal
  • Slaughter immediately
A

Answer: Flush the wound and let it heal

Explanation
The correct answer is flush the wound and let it heal. C. pseudotuberculosis manifests itself as sores and abscesses in cattle. They are most common on the lateral thorax, neck, flank, or head. It is thought that an injury can result in inoculation of the organism. Cows are relatively unaffected systemically, and the lesions will resolve on their own in 2-4 weeks.

In sheep and goats the disease is called caseous lymphadenitis and the lesions seen can be external abscesses called “boils”, which are spread by shearing in sheep, or internal abscesses that result in weight loss and emaciation. There is a vaccine against this condition in sheep.

166
Q

A group of 3-4 month old dairy calves have oral ulcers and appear reluctant to eat hay because their mouths are sore. You examine 3 affected calves and find that they are of normal size and weight, have normal feces and normal TPR, but have ulcers around the dental pad and gums. The poor quality alfalfa hay contains some grass as shown in the photo. Based on these findings, what is the correct diagnosis?

  • Actinobacillosis
  • Bluetongue
  • Water grass poisoning
  • Bovine virus diarrhea
  • Bristle grass (Setaria lutescens) irritation
A

Answer: Bristle grass (Setaria lutescens) irritation

Explanation
Yellow bristle grass contains sharp miniature barbs that stick into the mucosa of young cattle and horses and cause ulcers. Simple removal from the diet results in healing.

167
Q

A 4 year old Ayrshire cow presents with a two day history of depression, anorexia, fever, conscious proprioceptive deficits, right sided head tilt, and head-pressing. A neurologic exam identified cranial nerve deficits. Particularly, the right ear is drooping, the right eye appears “dropped”, and she is drooling from the right side of her mouth. Where is the lesion?

  • Right side and intracranial
  • Right side and extracranial
  • Left side and intracranial
  • Left side and extracranial
A

Answer: Right side and intracranial

Explanation
The correct answer is right side and intracranial. This is evident because the head tilt and cranial nerve deficits are usually on the side of the lesion and all of these are occurring on the same side in this situation. If you have to guess go with ipsilateral to the lesion. This is an intracranial problem since you have conscious proprioceptive deficits, head pressing, and depression.

168
Q

Which type of urolith occurs most commonly in western U.S. range cattle grazing mature grasses as shown in the photo?**

  • Calcium oxalate
  • Calcium carbonate
  • Silicate
  • Struvite
A

Answer: Silicate

Explanation
Silicate stones are the result of high intakes of silica in mature range grasses, combined with other factors such as dehydration. Struvites (ammonium, magnesium, and phosphate) tend to be found in feedlot animals receiving large amounts of grain. Calcium carbonate stones are most common in sheep grazing pastures high in calcium and oxalates.

169
Q

This cow in the picture presents for chemosis and oculonasal mucopurulent discharge. On physical exam, you note her to be sneezing, febrile, and having small white fibrinonecrotic plaques on her conjunctival and nasal mucosa. What is your diagnosis?

  • Moraxella bovis
  • Infectious bovine rhinotracheitis
  • Calici virus
  • Mannheimia hemolytica
A

Answer: Infectious bovine rhinotracheitis

Explanation
The correct answer is infectious bovine rhinotracheitis. The key clinical sign to make this diagnosis is the white fibrinonecrotic plaques. None of the other answer choices cause white fibrinonecrotic plaques.

170
Q

A 4-year old Holstein dairy cow in mid-lactation has developed acute onset illness and is colicky (kicking at abdomen, switching her tail, repeatedly lying down and getting up). Her temperature is 102.0 F (38.9 C), HR is 100/min, her scant feces are sticky and dark blackberry jam-colored, her abdomen is distended, and there is no rumen motility. No pings are audible. Her PCV= 30% (24-46%). A blood gas and electrolyte panel reveals Na=140 mEq/L (136-144 mEq/L), K=2.5 meq/L (3.6-4.9 mEq/L), Cl=80meq/L (99-107 mEq/L), and HCO3= 40meq/L (20-30 mEq/L). Based on these clinical signs and lab values, which of the following is most likely to be the correct diagnosis?

  • Salmonellosis
  • Cecal dilation
  • LDA
  • Abomasal ulcer
  • Intussusception
A

Answer: Intussusception

Explanation
The only choices likely to cause colic are intussusception, cecal dilation, and abomasal ulcer. Of those 3, only intussusception would cause scant, sticky, dark feces. The intussusception usually occurs in the jejunum or ileum, and results in a backed-up Gl tract. This causes chloride to be sequestered in the abomasum, as well as internally vomited into the rumen, resulting in hypochloremia, hypokalemia, and metabolic alkalosis. A PCV of 30% is normal for a cow, so a bleeding ulcer is unlikely; the HR of 100/min in this case is likely due to pain rather than anemia.

***PowerLecture: Colic

171
Q

What is the effect of PGF2-alpha during anestrus in cattle?

  • Induces FSH release
  • Induces an LH surge
  • There is no effect
  • Causes the lysis of the corpus luteum
A

Answer: There is no effect

Explanation
The correct answer is that there is no effect when the cow is in anestrus. That is the problem with using PGF2-alpha to synchronize estrus cycles in cows. A mature corpus luteum is needed so that PGF2-alpha can lyse it; the cow goes back into estrus in about 2-5 days.

172
Q

You are called early one cold spring morning to see a 5-year old dairy cow which is down in the corral. She freshened three days ago and is fed TMR. You find her to be barely responsive and unable to assume or maintain sternal recumbency, T=98F or 36.7 C, Hr=100/min and weak, and there is no rumen motility. The uterine lochia is red and mucoid. On rectal exam you find normal feces and a full bladder. How should you treat this cow?

  • IV saline, large volumes
  • Blood transfusion
  • IV magnesium
  • Broad spectrum antibiotics
  • IV calcium
A

Answer: IV calcium

Explanation

This cow has the history and classic signs of hypocalcemia, also called milk fever. Skeletal, cardiac and smooth muscle weakness lead to the signs, and she is unable to maintain her body temperature in cold weather.

173
Q

You are at a dairy trying to solve a mastitis problem. You perform a physical exam on a cow with mastitis and notice serum-like secretion with clots from two of the teats, and the affected quarters are hot and swollen. The cow has a fever of 104 F, rapid HR of 100 bpm, and is not eating well. What can you tell the dairyman with a fair degree of confidence about the type of mastitis this is likely to be?

  • This cow is infected with a coliform
  • This cow is infected with Trueperella pyogenes
  • This cow is infected with Staphylococcus aureus
  • This cow is infected with Mycoplasma mycoides
A

Answer: This cow is infected with a coliform

Explanation
The correct answer is this cow is infected with a coliform. This is a typical secretion and other clinical signs with coliform mastitis, although coliform mastitis can also present with mild clinical signs.

Trueperella pyogenes mastitis is usually thick pus and the cow is not so ill.

Staph aureus can also cause systemic signs of illness, or gangrenous mastitis when severe. It is usually found in a herd without a comprehensive mastitis prevention and control program.

Mycoplasma bovis and a few other species can cause mastitis in cows, but M mycoides is mainly a problem of mastitis in does and systemic illness and arthritis in young goats.

174
Q

What is the most common supraventricular arrhythmia in cattle?

  • Atrial standstill
  • Ventricular fibrillation
  • Sinoatrial node arrest
  • Atrial fibrillation
A

Answer: Atrial fibrillation

Explanation
The correct answer is atrial fibrillation. This usually occurs as a result of digestive disturbances. It is associated with vagal tone, hypokalemia, hyperkalemia, and calcium therapy. You may be able to palpate a pulse deficit and/or pulsus alternans in these animals. On ECG, there will be no P waves because the sinoatrial node is not working. You will see random high frequency, low amplitude depolarizations called F waves. A QRS wave that occurs at irregular intervals will be present.

175
Q

A large flock of sheep pastured on western US mountain rangeland has had several deaths and continues to have sick animals. The animals initially are observed to have ataxia and tremors, which progresses to lethargy, salivation, respiratory distress, bloating, and death. You visit the pasture to see what is occurring and perform a post-mortem on one animal. No significant gross lesions are found. The pasture seems to contain a great deal of lupine (see image), many species of which are known to contain toxic nicotinic-acting alkaloids such as quinolizidine alkaloids. The adjacent pasture contains the same plants and pregnant beef cows belonging to the same rancher. What should you tell him with regard to the cows?

  • Pyrrolizidine alkaloids can damage the liver of cattle
  • Expect cattle to develop bone marrow suppression and hemorrhage
  • Lupine alkaloids can be teratogenic in cattle
  • Cattle are likely to develop diarrhea if they consume lupine, but no other problems
  • Cattle will show signs similar to the sheep, but have chocolate brown-colored blood
A

Answer: Lupine alkaloids can be teratogenic in cattle

Explanation
Different quinolizidine alkaloids in various species of lupine can cause either neurologic signs or teratogenesis.

Sheep are often more affected with neurologic signs by acute toxicity to lupine, although cattle and horses can also be affected. Cattle experience fetal teratogenesis more often than sheep. Classic lesions are arthrogryposis (crooked calf) or cleft palate. Some laboratories can assay for lupine alkaloids in urine or serum, so these can be collected to help confirm the diagnosis in the sheep.

Other plants with similar nicotinic-acting alkaloids include tobacco, Indian tobacco and tree tobacco, locoweeds, Scottish broom, mountain thermopsis, and poison hemlock. The best course of action is to move the animals to safer pastures.

Pyrrolizidine alkaloids are liver toxins and are not noted in this pasture. Bone marrow suppression occurs in cattle that consume large amounts of bracken fern. Brown-colored blood is associated with nitrate toxicity, which is associated with other plants.

176
Q

You examine a 4-year old Holstein dairy cow that was a high producer for the first 2 months of the current lactation but has suddenly developed poor appetite and a markedly decreased milk production. On physical exam, you find temperature is 103.5F (39.7 C), HR=55, there is no rumen motility palpable or audible, and feces are scant. The cow has a papple shape when viewed from the rear (see image), with a slight “boink” hollow sound over the left flank. What is your tentative diagnosis?

  • Cecal displacement
  • Grain overload
  • Abomasal torsion/volvulus
  • Vagal indigestion
  • Left displaced abomasum
A

Answer: Vagal indigestion

Explanation
The most common cause of vagal indigestion is a metallic foreign body penetrating the reticulum (hardware disease). The local peritonitis leads to poor motility of the forestomachs, so that the rumen fills with fluid and develops a gas cap, and the abomasum accumulates fluid; these give the shape from the rear of pear on the right and apple on the left (papple shape) as seen in the photo.

***PowerLecture: Vagal Indigestion

177
Q

At what period in production does displaced abomasum most commonly occur in dairy cows?

  • First two weeks after conception
  • First four weeks postpartum
  • First two weeks after being shipped
  • Mid- to late lactation
A

Answer: First four weeks postpartum

Explanation
This occurs for multiple reasons. Due to the increased metabolic demands from lactation, the diet changes drastically, and more gas is produced in the abomasum. Dairy cows often undergo a period of clinical or subclinical hypocalcemia. Hypocalcemia predisposes to decreased abomasal motility and may be a factor contributing to displacement (especially since there is more gas to move).

178
Q

Which of the following is a common means by which Bovine Virus Diarrhea (BVD) virus is maintained in a “closed” beef herd?

  • By ingestion of virus-containing animal neural tissue in concentrate mixes
  • By commingling of cattle and dogs
  • Transmission by a fly vector
  • By ingestion of the L-3 to L-4 stages of intermediate host, roundworms
  • By direct infection/re-infection of vaccinated cattle from outside sources
  • By birth of calves persistently infected with BVD due to early-gestation in utero infection with specific types of BVD virus
A

Answer: ** By birth of calves persistently infected with BVD due to early-gestation in utero infection with specific types of BVD virus**

Explanation
Bovine fetuses infected with so-called “non-CPE” BVD virus early in gestation can survive to term and beyond birth as persistently infected (PI) calves. They are thus a source of infection for the remainder of the herd.
By definition, a “closed herd” would have little opportunity for exposure to “outside” sources of BVD.

Ingestion of feed contaminated with bovine neural tissue is a likely means of spreading bovine spongiform encephalopathy (BSE), but is not known to be a means of transmitting BVD. No role for roundworms as intermediate hosts for BVD has been shown. Co-mingling of dogs and cattle has been shown to transmit Neosprum caninum, but not BVD.

179
Q

A 4-day old bull calf has been unable to bear weight on the right forelimb since birth which was an assisted delivery in anterior presentation using a calving aid. There is loss of muscle tone and markedly reduced reflexes in the right foreleg (see image). The right foreleg is non-painful on palpation. The left foreleg and the hindlimbs are weight bearing with normal reflexes. What should you tell the farmer?

  • The calf appears to have a fracture of the distal third metacarpal growth plate due to traction at birth and has an excellent prognosis to improve with supportive care
  • The calf appears to have a right brachial plexus avulsion due to traction at birth and it is unknown whether it will improve with time
  • The calf appears to have right sided radial nerve paralysis due to traction at birth and prognosis for recovery is grave; the calf should be euthanized
  • The calf appears to have a radius fracture from traction at birth and is likely to improve with splinting of the distal limb
A

Answer: The calf appears to have a right brachial plexus avulsion due to traction at birth and it is unknown whether it will improve with time

Explanation
The most likely conditions to consider for this calf are brachial plexus avulsion and radial nerve paralysis due to traction at birth. This injury results in an inability to extend the elbow, carpus and fetlock and bear weight on the affected limb with loss of muscle over the shoulder with resultant prominent spine of the shoulder blade. There is a dropped elbow, scuffing of the hooves as the leg is moved forward, and the foot is knuckled over at rest. Depending on the severity of injury, many calves can recover with supportive care. It is often recommended to carefully splint the distal limo to prevent contraction of the flexor tendons.

Other differentials could include spinal cord trauma or congenital sarcocystosis. Fractures are less likely given the presentation and lack of pain on palpation.

180
Q

Several steers in a feedlot have interdigital lesions as shown in this image. They are lame and the lesion appears to be painful. You diagnose interdigital necrobacillosis or infectious footrot. Which of the following is the most effective feedlot treatment?

  • Table all lame cattle and treat lesions with tetracycline bandages
  • Florenicol SQ, one dose
  • Chloramphenicol IM daily for 5 days
  • Walk all cattle in the feedlot through a formalin footbath daily for 5 days
  • Flunixin daily for 10 days
A

Answer: Florenicol SQ, one dose

Explanation
One dose of 40 mg/kg florenicol is reported to be an effective therapy and would be favored by a feedlot because the single treatment would avoid costly repeated handling. The use of
chloramphenicol is prohibited in cattle in the USA.

181
Q

Which of the following is true of bovine trichomoniasis?

  • Once infected, a cow typically becomes a permanent carrier
  • It is typically first suspected by the discovery of preputial discharges from infected bulls
  • It is carried by the male, especially young, immunologically naive bulls
  • It is caused by the flagellated protozoan, Trichinella spiralis
  • It is primarily a gastrointestinal pathogen
  • The mature bull is commonly the reservoir of infection for a herd
A

Answer: The mature bull is commonly the reservoir of infection for a herd

Explanation
Trichomoniasis, caused by the protozoan Tritrichomonas foetus, is typically a temporary infection in the female. She characteristically eliminates the infection in 2-4 months. The bull, however, becomes a chronic carrier as he ages, since aging is associated with increased depth of epithelial crypts of a bull’s glans penis and prepuce, thus increasing the availability of the “niche” where the organism thrives.

Once infected, a cow typically clears the infection from the reproductive tract in 2-4 months and regains her fertility.

Young bulls are rarely infected chronically, probably because they have shallow penile and preputial crypts, which are thought to provide poor niches for chronic infection.

Trichomoniasis typically manifests no visible signs in either the male or female.

182
Q

You examine a 4-year old Holstein dairy cow which freshened 2 weeks ago. She has a history of abrupt cessation of lactation and loss of interest in feed. T=105F, HR=90 and RR=30. The cow has an arched back and is treading and swishing her tail frequently. On rectal exam you find an enlarged and painful left kidney. You catch urine in a cup after stimulating her to urinate. Which description of the urine best fits what you would expect to find in this cow?

  • Ketonuria
  • Amyloid casts
  • Crystals present
  • Cloudy and bloody
  • Hemoglobinuria
A

Answer: Cloudy and bloody

Explanation
This cow is likely to have acute pyelonephritis. It is most commonly ascending and associated with E. coli, other coliforms, or Corynebacterium renale.

You could also use ultrasound to confirm the diagnosis, but finding neutrophils, RBCs and bacteria with these signs is indicative of acute pyelonephritis.

183
Q

What is the most common congenital abnormality associated with fetal infection with bovine viral diarrhea?

  • Renal dysmaturia
  • Arthrogryposis
  • Shortened long bones
  • Cerebellar hypoplasia
A

Answer: Cerebellar hypoplasia

Explanation
The correct answer is cerebellar hypoplasia. Microphthalmia and hydranencephaly are also lesions observed with fetal BVD infection, however not as common. Renal dysmaturia is not seen with fetal BVD infection.

There are two types of BVD (Type 1 and Type 2) with different surface proteins. BVD virus is also classified as being cytopathic or noncytopathic, based on their behavior in cell culture, but both can cause illness.

Interestingly, carrier cattle have been infected with the noncytopathic biotype of BVD as a fetus.

***PowerLecture: Bovine Viral Diarrhea

184
Q

The owner of a nearby dairy calls to have a wound on his heifer inspected. The wound is located ventrally, on the brisket, and there are larvae feeding on the tissue. The wound is approximately 5cm in diameter and has now become 6 cm deep as a result of larval feeding. As you approach the cow, you see a large, metallic blue-green colored fly leaving the wound. What is your diagnosis?

  • Chrysops
  • Cochliomyia hominivorax
  • Stomoxys calcitrans
  • Tabanus
A

Answer: Cochliomyia hominivorax

Explanation
The correct answer is Cochliomyia hominivorax. This fly is also known as the screwworm. Females lay hundreds of eggs at the edge of fresh wounds on the cow; the larvae hatch in approximately 24 hours. As the larvae eat the living tissue, they create a huge wound. Screwworm larvae then fall off and pupate, completing their life cycle in approximately 21 days. This fly is reportable!

185
Q

A 1-year old unvaccinated Angus steer presents as a result of acute lameness and depression. He has historically been healthy and one of the best animals in the group. On physical exam, there are no signs of trauma and the steer is febrile. A crepitant, edematous swelling is seen on the muscles of the left shoulder. It is hot and painful to the touch. Assuming there are no signs of trauma, what is the most likely diagnosis?

  • Clostridium hemolyticum
  • Clostridium chauvoei (blackleg)
  • Clostridioides difficile
  • Corynebacterium pseudotuberculosis
  • Mycoplasma bovis
A

Answer: Clostridium chauvoei (blackleg)

Explanation
The correct answer is Clostridium chauvoei (blackleg). Clostridium chauvoei results in infection after gaining access to the blood stream via the alimentary tract and then depositing in muscle. This is a serious disease, and animals will die if left untreated. Cattle should be vaccinated at 3 to 4 months of age against this and other Clostridial diseases. Clostridium hemolyticum causes liver lesions, and the toxin results in acute hemolysis and often in death. Clostridioides difficile usually results in gastrointestinal disease in horses.

Corynebacterium pseudotuberculosis results in large external bleeding sores on the skin of cattle and boils and internal abscesses in sheep and goats.

Mycoplasma bovis can cause sepsis, joint infections and mastitis.

186
Q

There is an appointment today for a work-up on a lame cow. “Playing the odds”, what digits are most likely to be involved, because these are the digits most often affected in dairy cattle?

  • Lateral front digits
  • Medial front digits
  • Lateral rear digits
  • Medial rear digits
A

Answer: Lateral rear digits

Explanation
The correct answer is lateral rear digits. Over 80% of foot disease involving the digits involves the rear digits.

Additionally, approximately 85% of these involve the lateral aspect. This is because they bear the majority of their rear limb weight on the lateral claw in the hind. In the front, cattle put most of their weight on their medial claw.

187
Q

A 7-day old calf presents for further evaluation and supportive care due to an acute onset of watery diarrhea. On physical examination, the calf is markedly depressed, floppy, nonresponsive, and 10% dehydrated. Which of the following best describes this calf’s state?

  • Normal acid-base status with severe hypocalcemia and sepsis
  • Hypovolemia, severe metabolic acidosis with hypokalemia and hypercalcemia
  • Metabolic acidosis, severe hyponatremia, hypokalemia and sepsis
  • Hypovolemia, metabolic alkalosis with severe hypokalemia
  • Hypovolemia, severe metabolic acidosis with hyperkalemia and possible sepsis
A

Answer: Hypovolemia, severe metabolic acidosis with hyperkalemia and possible sepsis

Explanation
No matter what the causative agent, calves of this age with severe diarrhea develop hypovolemia, severe metabolic acidosis, which results in potassium leaving cells, and the calf becomes hyperkalemic (in plasma). The floppy calf is typical of an animal with severe metabolic acidosis (base deficit of 15 meq or more) and may also be hypoglycemic. This calf is also possibly septic, as bacterial translocation across the gut often occurs in very ill calves.

188
Q

A recently freshened cow presents for decreased appetite and milk production. On physical exam, a left sided variable pitch ping is heard using simultaneous auscultation and percussion from the last rib and then diagonally downward to the 8th rib. Which is the most likely differential?

  • Left displaced abomasum
  • Free gas in the rumen
  • Right displaced abomasum
  • Hardware disease
A

Answer: Left displaced abomasum

Explanation
LDAs occur most commonly after parturition and should be considered. The ping described is classic for a left displaced abomasum. You can rule out a right displaced abomasum because the ping is described on the left side. Right displaced abomasum is much less common than LDA. There is no mention of stiff gait or xiphoid pain that would indicate a diagnosis of hardware disease. Free gas in the rumen is likely to be heard from the hip to the 8th rib and be a monotone ping.

***PowerLecture: Left Displaced Abomasum

189
Q

One cold December morning you are presented with a 5-year old Holstein dairy cow which freshened the day before. This morning she was found down in the corral and unable to rise. On physical examination you find T=98F, 36.7C, HR=90 (and the heart is difficult to hear), and RR=10. She has her neck turned back toward her thorax, and it has an “S” shaped curve in it. Her nose is dry, she is non-responsive to being handled, and her rumen motility is absent. The rectal exam shows that her bladder is full. The mammary glands and the uterine lochia appear normal. Based on these findings, which of the following disorders is most likely?

  • Milk fever
  • Left displaced abomasum (LDA)
  • Spinal lymphoma
  • Hypothyroidism
  • Coliform mastitis
A

Answer: Milk fever

Explanation
A cow with a history of recent parturition with clinical signs such as these should be considered as having hypocalcemia until proven otherwise. The cow needs treatment to restore normal calcium levels.

190
Q

Pick the ideal suture pattern for a uterus after a Caesarian section in a cow.

  • Utrecht pattern
  • Near-far-far-near
  • Simple interrupted
  • Horizontal mattress
A

Answer: Utrecht pattern

Explanation
The correct answer is the Utrecht pattern. This pattern achieves a nice mucosal and serosal seal when done correctly. Additionally, the exposure of suture to the serosa is minimized with this type of pattern, so adhesions of uterus to surrounding structures is minimized.

191
Q

The image shows an infarct in the liver discovered on post mortem exam of a mature beef cow which died one hour before in a western mountain pasture, after being observed to appear normal one day earlier. There is also dark red urine in the bladder. The pasture contains native plants, some pine trees, and a marshy area with water plants. The cows are unvaccinated and were never wormed. Given this history and the lesion found, the most likely cause of death is

  • Bacillary hemoglobinuria
  • Blue-green algae toxicity
  • Pine needle poisoning
  • Viral hepatitis
  • Death camas toxicity
A

Answer: Bacillary hemoglobinuria

Explanation
Also known as redwater, bacillary hemoglobinuria is caused by germination of Clostridium Novyi type D spores in the liver after anaerobic damage by migrating liver fluke larvae. Cl. Novyi was formerly called Cl. hemolyticum. Vaccination can prevent this disease.

***PowerLecture: Flukes and Bacillary Hemoglobinuria

192
Q

A client brings the one-half inch grub shown in the image to you one spring day, telling you it emerged from a hole in the back of one of his prize show cattle. He wants to know what to treat his cattle with and when.

  • Moxidectin now, in spring
  • Ivermectin in February
  • Ivermectin in early fall
  • Thiabendazole in early fall
  • Organophosphates in summer
A

Answer: Ivermectin in early fall

Explanation
This is a cattle warble called Hypoderma. The 2 species are H. bovis and H. lineatum. They undergo a long migration in tissues and only emerge from the back of the animal in spring. The crucial treatment time is early fall when larvae are just beginning to migrate in tissues. Organophosphates or one of the macrocyclic lactones (ivermectin, doramectin, eprinomectin or moxidectin) are effective.

193
Q

A herd of cattle present for an acute onset of a mild cough. Many of the cattle subsequently die and the others have improved over the course of several days. The cattle had a history of being moved to a lush pasture. Necropsy findings showed acute interstitial pneumonia and air filled bullae. What is the likely chemical responsible for this disease?

  • Magnesium
  • 3-Methylindole
  • 4-Ipomeanol
  • Methylmethacrylate
A

Answer: 3-Methylindole

Explanation
The correct answer is 3-methylindole. Cattle that are moved to lush forage that is high in tryptophan metabolize the tryptophan to 3-methylindole in the rumen. The 3-MI is toxic to the lung, resulting in acute interstitial pneumonia and emphysema. This syndrome is known as acute bovine pulmonary edema and emphysema (ABPEE), fog fever or grunts.

4- Ipomeanol is the chemical involved in moldy sweet potato poisoning, which looks clinically similar to this.

194
Q

A cow about one month from term goes off feed and rapidly develops an enlarged, rounded, abdominal shape.
She is now uncomfortable and reluctant to move. You examine the cow and note marked accumulation of watery fluid in the uterus. You perform a brief ultrasound and confirm the large volume of fluid in the uterus and a thickened placenta. What should you tell the farmer about this condition?

• Hydrops allantois (hydroallantois) is a disorder of the placenta and the prognosis for life of the fetus and fertility of the cow is poor
• Hydrops allantois (hydroallantois) is a disorder of the fetus and can be treated by induction of parturition or Caesarian section and in most cases if the fetus is close to term, the calf will survive and the cow will be fertile
• Hydrops amnion (hydramnios) is a disorder of the placenta and the fetus is likely to survive but the cow will likely be infertile
• Hydrops amnion (hydramnios) is a disorder of the fetus and although the fetus is unlikely to survive, the cow should be okay

A

Answer: Hydrops allantois (hydroallantois) is a disorder of the placenta and the prognosis for life of the fetus and fertility of the cow is poor

Explanation
The clinical description is most consistent with hydroallantois because of the rapid accumulation of fluid and the clinical signs of the cow.

Hydramnios is a reasonable differential although it more typically involves gradual accumulation of fluid and the cow is usually clinically unaffected.

Hydroallantois is a disorder of the placenta (chorio-allantois) that results in rapid accumulation of 100-200 liters of watery, clear fluid during the last trimester. The cow develops a rounded appearance to the abdomen and it is usually not possible to palpate the fetus or placentomes. The cow often becomes sick with anorexia, decreased rumen motility, dehydration and weakness. She may go down or rupture the prepubic tendon. The prognosis for the fetus is guarded and even with treatment, the cow will likely be infertile. Induction of parturition or C-section are treatment options but the majority of calves are not viable and the dam rarely has normal colostrum. If the pregnancy is terminated, it is unlikely that the cow will have a productive milk cycle. Usually salvage for slaughter is the preferred option unless the fetus is considered valuable and the cow is near term.

Hydramnios is a disorder of the fetus, and the placenta itself is normal. The condition is typically characterized by gradual accumulation of thick viscous fluid during the later part of gestation. The cow develops a pear shaped caudal view. The fetus and placentomes are usually palpable and the cow is usually clinically well. Pregnancy usually goes to term and a small, deformed fetus is delivered. The cow has a fair to good prognosis for life and fertility. The cow may be induced or allowed to go to term depending on her condition.

195
Q

What are the effects on chloride and acid-base balance when pyloric outflow is impeded in a ruminant?

  • Decreased serum chloride and metabolic alkalosis
  • Decreased serum chloride and metabolic acidosis
  • Increased serum chloride and metabolic acidosis
  • Increased serum chloride and metabolic alkalosis
A

Answer: Decreased serum chloride and metabolic alkalosis

Explanation
The correct answer is decreased serum chloride and metabolic alkalosis. This occurs as a result of the intestines not being able to reabsorb chloride back into the blood stream after it is secreted into the abomasum. Chloride is a strong anion and results in metabolic alkalosis when it is decreased.

***PowerLecture: Vagal Indigestion

196
Q

You perform a somatic cell count (SCC) on a string of cows that are down in milk production. Their count ranged from 100,000 to 200,000 cells/ml. What is your interpretation?

  • SCC is severely elevated
  • SCC is below normal
  • SCC is moderately elevated
  • SCC is normal
A

Answer: SCC is normal

Explanation
The correct answer is the somatic cell count is normal. The problem that is causing the decreased milk production does not appear to be associated with mammary gland infection.

Many studies suggest that cows with SCC of less than 200,000 are not likely to be infected with major mastitis pathogens, but cows with SCC above 300,000 are probably infected (Smith, 1996).

Herds with bulk tank SCC above 200,000 have varying degrees of subclinical mastitis. At 500,000 SCC, approximately 16% of the mammary quarters may be infected, resulting in a 6% reduction in milk production compared to a SCC of 200,000.

197
Q

On a visit to a feedlot, the rancher asks you what method she should use to euthanize sick cattle. Which of the following is considered an acceptable method of euthanasia for conscious cattle, according to the AVMA?

  • Intravenous injection of xylazine followed by intravenous injection of potassium chloride
  • Air embolism (intravascular injection of air)
  • Gunshot
  • Intravenous injection of bleach (sodium hypochlorite)
  • Exsanguination
A

Answer: Gunshot

Explanation
According to the AVMA Guidelines for the Euthanasia of Animals, gunshot is the most common method used for on-farm euthanasia of cattle. The anatomic landmarks used for the point of entry should be the intersection of two imaginary lines drawn from the outside corner of the eye to the center of the base of the opposite horn (or equivalent position). The person euthanizing the animal should have sufficient training and experience with the firearm for a safe and accurate procedure. Although one well-placed bullet usually results in immediate loss of consciousness with little likelihood of return to consciousness, one should always be prepared to deliver a second or even a third shot if necessary.

Unacceptable methods for euthanasia of conscious animals include injection of chemical agents (disinfectants, electrolytes including potassium chloride, and nonanesthetic pharmacologic agents), drowning, air embolism, electrocution, and exsanguination. Administration of xylazine or other alpha-2 agonists followed by intravenous potassium chloride or magnesium sulfate is also considered unacceptable because alpha-2 agonists alone are recognized as being unreliable for producing generalized anesthesia although they can produce a state resembling it.

Ref: AVMA Guidelines for the Euthanasia of Animals: 2013 Edition

198
Q

Just after Christmas, a dairy herd of 200 cows in the northeastern US has experienced an outbreak of diarrhea in 40% of the adult cows, with a resultant drop in appetite and milk production. The animals go to pasture in the spring, summer and fall, but in winter they are housed in a barn. The 2 year-old cows seem most severely affected. You examine two affected animals and find brownish fetid liquid feces full of bubbles. There are clots of blood in the feces of one of the animals.
Temperatures are normal, but HR=90 (60-70 beats/min), and they appear to be slightly dehydrated. Rumen motility is weak and the rumen is relatively empty. No other obvious abnormalities are noted. Yesterday the dairyman, at your instruction, submitted a CBC and feces to the diagnostic lab. They report that the CBC is normal in both animals, and both are negative for parasite ova. Given the history, age group affected, clinical signs and lab results, which of the following is the most likely diagnosis?

  • Salmonella typhimurium
  • Winter dysentery
  • Johne’s disease
  • Type 2 Ostertagia infestation
  • Clostridium perfringens type A hemorrhagic bowel syndrome
A

Answer: Winter dysentery

Explanation
Winter dysentery is an epidemic disorder of housed cattle, usually occurring in cold months and affecting 10-90% of adult cows. If fever occurs at all, it usually precedes the diarrhea phase. The disease is thought to be caused by coronavirus, and lesions involve the colon. In typical outbreaks, the disease runs its course in weeks and the herd becomes immune and recovers, although milk production may not recover to previous levels.

With Salmonella typhimurium, the cows would be sicker, febrile, have abnormalities in the CBC compatible with endotoxemia (neutropenia or neutrophilia), and an increased plasma fibrinogen. In a severe Salmonella typhimurium outbreak, some cows would likely die. The scenario described is not compatible with a diagnosis of Johne’s disease. Although a few individual animals in a herd might develop type 2 Ostertagia, a herd outbreak would be highly unlikely, especially involving older cows. With Clostridium perfringens type A, there is a low morbidity and high mortality; signs of small intestinal obstruction develop, and if feces are passed they are very dark and bloody.

199
Q

A 12 month old feedlot calf has inspiratory dyspnea, loud inspiratory noises, open mouth breathing, extended head and neck, and a moist painful cough. It is febrile, anorectic and depressed. On palpation of the larynx, the cough appears painful and the dyspnea worsens. What is the condition called?

  • 3-MI (3-methyl-indole) induced acute pulmonary edema and emphysema
  • Calf diphtheria
  • Pasteurella multocida bronchopneumonia
  • Acute BRSV (bovine respiratory syncytial virus) pneumonia
  • Mannheimia hemolytica bronchopneumonia
A

Answer: Calf diphtheria

Explanation
Also called necrotic laryngitis, or laryngeal necrobacillosis, this mainly feedlot condition strikes randomly but can result in death or a permanently misshapen larynx. The causes appear to be Histophilus somni and Fusobacterium necrophorum.

***PowerLecture: Upper Respiratory Disease

200
Q

Fluoride toxicity at a young age in cattle may result in permanent damage to which of the following?

  • Heart
  • Eyes
  • Teeth
  • Liver
A

Answer: Teeth

Explanation
The correct answer is teeth. Fluoride toxicity results in ameloblasts prematurely reducing in size. Eventually, the enamel epithelium forms an irregular matrix which does not calcify normally. Fluoride toxicity will cause bone lesions. Interestingly, the first palpable lesions are seen on the medial surface of the proximal third of the metatarsal bones. A key thing to know is that articular surfaces are not affected with osteofluorosis so this may help rule it in or out from other diseases such as osteomyelitis, septic arthritis, and osteoarthritis.